SlideShare a Scribd company logo
1 of 878
Download to read offline
Nelson Pediatrics Review(MCQs)19ed
‫ﺩ‬ ‫ﺘﻤﻨﻴﺎﺕ‬ ‫ﻤﻊ‬.‫ﺍﻟﺩﻋﺎﺀ‬ ‫ﺼﺎﻟﺢ‬ ‫ﻤﻥ‬ ‫ﺘﻨﺴﻭﻨﺎ‬ ‫ﻭﻻ‬ ، ‫ﺒﺎﻟﺘﻭﻓﻴﻕ‬ ‫ﺍﻟﺩﺠﺎﻨﻲ‬ ‫ﺯﻫﻴﺭ‬
--------------------------------------------------------------
1. Which of the following statements regarding foster care is true?
□A permanency plan must be made for a child in foster care no later than 12 mo from the child's entry into care
□A minority of children in foster care have a history of abuse or neglect
□The mission of foster care is to safely care for children while providing services to families to promote reunification
□Most (>70%) of children in foster care are reunited with their families
■A and C
description The mission of foster care is to provide for the health, safety, and well-being of children while assisting their
families with services to promote reunification. Children entering foster care have frequently experienced early childhood
trauma. More than 70% have a history of abuse, neglect, or both. Only about 50% of children achieve reunification. In the
USA, the Adoption and Safe Families Act (P.L. 105-89) passed in 1997 requires that a permanency plan be made for
each child no later than 12 mo after entry to foster care and that a petition to terminate parental rights typically must be filed
when a child has been in foster care for at least 15 of the previous 22 mo. (See Chapter 35, page 134, and e35-1.)
2. A 4 yr old girl is admitted to the hospital for her third evaluation for vaginal bleeding. The
mother noted bright red blood on the child's underwear. Previous examinations revealed a
normal 4 yr old girl, Tanner stage 1, with normal external genitalia. Pelvic ultrasound results
were normal, as was the serum estradiol level. The hemoglobin and platelet counts were
normal, as were the bleeding time and coagulation studies. Findings on pelvic examination
conducted under anesthesia also were normal. The next step in the examination is to:
■Determine the blood type of the blood on the underwear
□Interrogate the father
□Isolate the parents and child
□Determine von Willebrand factor levels
□Measure fibronectin in the vagina
description Consideration of factitious disorder by proxy should be triggered when the reported symptoms are repeatedly
noted by only one parent, appropriate testing fails to confirm a diagnosis, and seemingly appropriate treatment is ineffective.
At times, the child's symptoms, their course, or the response to treatment may be incompatible with any recognized disease.
Preverbal children are usually involved. Bleeding is a particularly common presentation. This may be caused by adding dyes
to samples, adding blood (e.g., from the mother) to the child's sample, or giving the child an anticoagulant (e.g., warfarin).
(See Chapter 37, page 146.)
3. Munchausen syndrome by proxy is characterized by all of the following EXCEPT:
□Mother who appears devoted and wins over members of care team
□Multiple hospitalizations and investigations without diagnosis
□Symptoms on history but not witnessed by medical team
■Symptoms occurring in presence of different caregivers (e.g., while mother is out of town)
□Use of medications or toxins
description Symptoms in young children are mostly associated with proximity of the offending caregiver to the child. The
mother may present as a devoted or even model parent who forms close relationships with members of the health care
team. While appearing very interested in her child's condition, she may be relatively distant emotionally. (See Chapter 37,
page 146.)
4. Which statement is false?
■Malnutrition is the second leading cause of acquired immune deficiency worldwide behind HIV infection
□Zinc is important in immune function and linear growth
□Kwashiorkor and marasmus are rare in developed countries
□The Western diet is associated with increased noncommunicable disease
description The significant global burden of malnutrition and undernutrition is the leading worldwide cause of acquired
immunodeficiency and the major underlying factor for morbidity and mortality globally for children <5 yr of age. Zinc is a
micronutrient that supports multiple metabolic functions in the body, is essential for normal immune functioning, and is
required to support linear growth; zinc deficiency is associated with impaired immune functioning and poor linear growth. In
parallel to the risk for nutrient and energy deficiencies, issues relating to excesses pose important challenges because of their
negative health effects, such as obesity or cardiovascular disease risk factors. The nutrition transition under way in the
developing world from traditional diets to the Western diet has been associated with increases in noncommunicable
diseases, often coexisting with undernutrition and malnutrition, observed sometimes in the same communities or even the
same families. (See e41-1.)
5. Components of energy expenditure in children include:
□Thermal effect of food
□Basal metabolic rate
□Energy for physical activity
□Energy to support growth
■All of the above
description The 3 components of energy expenditure in adults are the basal metabolic rate, the thermal effect of food
(energy required for digestion and absorption), and energy for physical activity. Additional energy intake and expenditure
are required to support growth and development for children. (See e41-4.)
6. Which of the following clinical scenarios increases the risk of vitamin A deficiency?
□Vegetarian diet
□Chronic intestinal disorders
□Zinc deficiency
■B and C
□All of the above
description Vitamin A is an essential micronutrient because it cannot be biogenerated de novo by animals. It must be
obtained from plants in the form of provitamin-A carotenoids. In the USA, grains and vegetables supply approximately
55% and dairy and meat products supply approximately 30% of vitamin A intake from food. Vitamin A and the
provitamins-A are fat soluble, and their absorption depends on the presence of adequate lipid and protein within the meal.
Chronic intestinal disorders or lipid malabsorption syndromes can result in vitamin A deficiency. In developing countries,
subclinical or clinical zinc deficiency can increase the risk of vitamin A deficiency. There is also some evidence of marginal
zinc intakes in children in the USA. (See Chapter 45, page 188.)
7. Which statement about vitamin A toxicity is NOT true?
□Excess vitamin A in utero can cause congenital malformations
□It may present as pseudotumor cerebri
■An infant with a preference for carrots and butternut squash may develop toxicity
□It may cause fissures at the corners of the mouth, pruritus, and alopecia
□Symptoms subside rapidly after withdrawing the source of the vitamin
description Excessive intake of carotenoids is not associated with toxicity but can cause yellow coloration of the skin that
disappears when intake is reduced; this disorder (carotenemia) is especially likely to occur in children with liver disease,
diabetes mellitus, or hypothyroidism and in those who do not have enzymes that metabolize carotenoids. (See Chapter 45,
page 191.)
8. Which statement about vitamin E is false?
□The most common form of vitamin E is tocopherol
■Premature infants given formula with a high content of polyunsaturated fatty acids and iron supplementation are protected
from deficiency
□Cholestatic liver disease increases the risk of deficiency
□Premature infants with vitamin E deficiency develop hemolysis, thrombocytosis, and edema
□Prolonged vitamin E deficiency causes a severe, progressive neurologic disorder
description Premature infants are particularly susceptible to vitamin E deficiency because there is significant transfer of
vitamin E during the last trimester of pregnancy. Vitamin E deficiency in premature infants causes thrombocytosis, edema,
and hemolysis, potentially causing anemia. The risk of symptomatic vitamin E deficiency was increased by the use of
formulas for premature infants that had a high content of polyunsaturated fatty acids (PUFAs). These formulas led to a high
content of PUFAs in red blood cell membranes, making them more susceptible to oxidative stress, which could be
ameliorated by vitamin E. Oxidative stress was augmented by aggressive use of iron supplementation; iron increases the
production of oxygen radicals. The incidence of hemolysis due to vitamin E deficiency in premature infants decreased
secondary to the use of formulas with a lower content of polyunsaturated fatty acids, less-aggressive use of iron, and
provision of adequate vitamin E. (See e49-1.)
9. Manifestations of hyperkalemia include all of the following EXCEPT:
□Paresthesias
□Weakness
□Paralysis
□Wide QRS complex
■Tetany
description The most important effects of hyperkalemia are due to the role of potassium in membrane polarization. The
cardiac conduction system is usually the dominant concern. Changes in the electrocardiogram (ECG) begin with peaking of
the T waves. This is followed, as the potassium level increases, by ST segment depression, an increased PR interval,
flattening of the P wave, and widening of the QRS complex. This process can eventually progress to ventricular fibrillation.
Asystole may also occur. Some patients have paresthesias, fasciculations, weakness, and even an ascending paralysis, but
cardiac toxicity usually precedes these clinical symptoms, emphasizing the danger of assuming that an absence of symptoms
implies an absence of danger. (See Chapter 52, page 221.)
10. Hyperkalemia may be associated with all of the following EXCEPT:
□Succinylcholine use
□Burns
□Trauma
□Chemotherapy
■Metabolic alkalosis
□Digitalis toxicity
□Uremia
description Many causes of hyperkalemia result in metabolic acidosis; a metabolic acidosis worsens hyperkalemia through
the transcellular shift of potassium out of cells. Renal insufficiency is a common cause of the combination of metabolic
acidosis and hyperkalemia. This association is also seen in diseases associated with aldosterone insufficiency or aldosterone
resistance. (See Chapter 52, page 221.)
11. The best method to reduce the potassium level during hyperkalemia, by reducing the body
burden of potassium, is:
□Sodium bicarbonate infusion
□Glucose and insulin infusion
□Calcium infusion
□Albuterol aerosol
■Kayexalate enema
description Treatment of hyperkalemia has 2 basic goals: (1) to stabilize the heart to prevent life-threatening arrhythmias
and (2) to remove potassium from the body. The treatments that acutely prevent arrhythmias all have the advantage of
working quickly (within minutes) but do not remove potassium from the body. Calcium stabilizes the cell membrane of heart
cells, preventing arrhythmias. It is given intravenously over a few minutes, and its action is almost immediate. Several
medications cause potassium to move intracellularly and thus rapidly reduce the plasma level to prevent arrhythmias. These
include bicarbonate, insulin and glucose, and nebulized albuterol. However, these medicines do not remove potassium from
the body. To reduce the total body potassium, 3 options are available. In patients who are not anuric, a loop diuretic
increases renal excretion of potassium. A high dose may be required in a patient with significant renal insufficiency. Sodium
polystyrene sulfonate (Kayexalate) is an exchange resin that is given either rectally or orally. Sodium in the resin is
exchanged for body potassium, and the potassium-containing resin is then excreted from the body. Some patients require
dialysis for acute removal of potassium. Dialysis is often necessary if the patient has either severe renal failure or an
especially high rate of endogenous potassium release, as is sometimes present with tumor lysis syndrome or
rhabdomyolysis. (See Chapter 52, page 222.)
12. Clinical manifestations of hypokalemia include all of the following EXCEPT:
□ECG changes
□Paralysis
□Urinary retention
□Constipation
□Muscle cramps
■Blurry vision
description The heart and skeletal muscle are especially vulnerable to hypokalemia. ECG changes include a flattened T
wave, a depressed ST segment, and the appearance of a U wave, which is located between the T wave (if still visible) and
the P wave. Ventricular fibrillation and torsades de pointes may occur, although usually only in the context of underlying
heart disease. The clinical consequences of hypokalemia in skeletal muscle include muscle weakness and cramps. Paralysis
is a possible complication, generally only at potassium levels <2.5 mEq/L. It usually starts in the legs and moves to the
arms. Respiratory paralysis may require mechanical ventilation. Some patients have rhabdomyolysis; the risk increases with
exercise. Hypokalemia slows gastrointestinal motility. This effect manifests as constipation; with potassium levels <2.5
mEq/L, an ileus may occur. Hypokalemia impairs bladder function, potentially leading to urinary retention. (See Chapter
52, page 224.)
13. From the following list, choose the route(s) by which insensible water loss may occur: 1. Sweat,
2. Fecal loss, 3. Evaporative loss from skin, 4. Respiratory water loss, 5. Obligate water for
urinary solute excretion
□1 and 3
□1, 2, and 3
□3 only
■3 and 4
□2 and 5
description Water is a crucial component of maintenance fluid therapy because of the obligatory daily water losses. These
losses are both measurable (urine, stool) and not measurable (insensible losses from the skin and lungs). Failure to replace
these losses leads to a child who is thirsty, uncomfortable, and, ultimately, dehydrated. (See Chapter 53, page 242.)
14. Which of the following is a goal of maintenance fluids?
□Diminish protein degradation
□Prevent dehydration
■Prevent hunger
□Prevent electrolyte derangements
□Prevent ketoacidosis
description The glucose in maintenance fluids provides approximately 20% of the normal caloric needs of the patient,
prevents the development of starvation ketoacidosis, and diminishes the protein degradation that would occur if the patient
received no calories. Glucose also provides added osmoles, thus avoiding the administration of hypotonic fluids that may
cause hemolysis. Maintenance fluids do not provide adequate calories, protein, fat, minerals, or vitamins. This fact is
typically not problematic for a patient receiving intravenous fluids for a few days. A patient receiving maintenance
intravenous fluids is receiving inadequate calories and will lose 0.5-1% of weight each day. Table 53-1 summarizes the
goals of maintenance fluids. (See Chapter 53, page 242.)
15. Which patient has an elevated risk of hyponatremia with standard maintenance fluid therapy
(D5 ½ NS if >10 kg, D5 ¼ NS if <10 kg)?
□6 mo old NPO for elective hernia repair
□4 month old with bronchiolitis and poor oral intake
□13 yr old status post motor vehicle accident with multiple fractures, requiring treatment with narcotics and antiemetics
□8 yr old with nephrotic syndrome
□A and D
■B, C, and D
description Patients who are producing antidiuretic hormone (ADH) may retain water, creating a risk of hyponatremia due
to water intoxication. Patients who may be producing ADH owing to subtle volume depletion or other mechanisms
(respiratory disease, stress, pain, nausea, medications such as narcotics) may be more safely treated with fluids that have a
higher sodium concentration, with a decrease in fluid rate, or with a combination of these strategies. Patients with persistent
ADH production due to an underlying disease process (syndrome of inappropriate ADH secretion [SIADH], congestive
heart failure, nephrotic syndrome, liver disease) should receive less than maintenance fluids. Treatment is individualized, and
careful monitoring is critical. Special caution is needed in patients who are known to have low-normal serum sodium
concentrations or hyponatremia. (See Chapter 52, page 243.)
16. In which patient is oral rehydration NOT indicated?
□2 yr old with moderate hypernatremic dehydration
□6 mo old with mild hyponatremic dehydration
■4 mo old with severe dehydration and normal serum sodium
□3 yr old with moderate dehydration and normal serum sodium
□A and C
description Dehydration, most often due to gastroenteritis, is a common problem in children. Most cases can be managed
with oral rehydration. Even children with mild to moderate hyponatremic or hypernatremic dehydration can be managed
with oral rehydration. The infant with severe dehydration is gravely ill. The decrease in blood pressure indicates that vital
organs may be receiving inadequate perfusion. Immediate and aggressive intervention is necessary. If possible, the child
with severe dehydration initially should receive intravenous therapy. (See Chapter 54, page 245.)
17. Which statement about pediatric poisoning is NOT true?
□Most poisonings among young children involve a single substance and are unintentional
□Poison prevention should be discussed at all well child visits beginning at 6 months
□Pediatric poisonings occur most frequently in the toddler and adolescent age ranges
■The toddler age group experiences the majority of poisoning deaths
□Poison control centers are available via phone, 24-7, toll free
description Although the majority of exposures are in children <6 yr, only 2% of the reported deaths occur in this age
group. In addition to the exploratory nature of ingestions in young children, product safety measures, poison prevention
education, early recognition of exposures, and around-the-clock access to regionally based poison control centers all
contribute to the favorable outcomes in this age group. Exposures in the adolescent age group are primarily intentional
(suicide or abuse or misuse of substances) and thus often result in more severe toxicity. Adolescents (ages 13-19 yr)
accounted for 74 of the 108 poison-related pediatric deaths in 2008 reported to the National Poison Data System.
Pediatricians should be aware of the signs of drug abuse or suicidal ideation in this population and should aggressively
intervene. (See Chapter 58, page 250.)
18. Quantitative levels of certain medications are helpful in the management of acute poisonings.
For which of the following medications is this NOT true?
□Salicylates
□Acetaminophen
□Iron
□Carbon monoxide
■Marijuana
description For select intoxications (salicylates, some anticonvulsants, acetaminophen, iron, digoxin, methanol, lithium,
theophylline, ethylene glycol, carbon monoxide), quantitative blood concentrations are integral to confirming the diagnosis
and formulating a treatment plan. For most exposures, quantitative measurement is not readily available and is not likely to
alter management. (See Chapter 58, page 253.)
19. The 4 principles of management of poisonings are what?
■Decontamination, enhanced elimination, antidote, supportive care
□Degradation, hydration, oxygenation, antidote
□Ipecac, clinical monitoring, dialysis, reversal
□Alkalinization, oxygenation, elimination, hydration
description The 4 principles of management of the poisoned patient are decontamination, enhanced elimination, antidote,
and supportive care. Few patients meet criteria for all of these interventions, although clinicians should consider each option
in every poisoned patient so as not to miss a potentially lifesaving therapy. Antidotes are available for relatively few poisons,
thus emphasizing the importance of meticulous supportive care and close clinical monitoring. (See Chapter 58, page 254.)
20. A 15 yr old boy is admitted to your care after the intentional ingestion of 2 g of his own
amitriptyline in a suicide gesture. He received activated charcoal for gastrointestinal
decontamination. The patient is placed in the ICU for monitoring and remains stable. He is
receiving only 0.45% normal saline. You observe a change in his cardiac monitor display with a
widening of his QRS complex to 0.12 second and occasional ectopic beats. The most
appropriate next step in management is to:
□Ignore these changes because they are still within normal limits
■Add sodium bicarbonate to his IV fluids to try to raise his serum pH above 7.4
□Repeat a dose of activated charcoal
□Begin a lidocaine infusion at an appropriate dose
□Order a chest radiograph
description Sodium bicarbonate is the antidote of choice for TCA toxicity and works via overcoming the sodium channel
blockade by providing a sodium load and via inducing an alkalosis to decrease drug binding to sodium channels. Indications
for sodium bicarbonate include a QRS duration >100 msec, ventricular dysrhythmias, and hypotension. An initial bolus of
1-2 mEq/kg of sodium bicarbonate is given followed by initiation of a continuous infusion. Additional boluses may be given
if the QRS duration continues to widen, with the goals of therapy being a serum pH of 7.45-7.55, improved hemodynamic
stability, and narrowing of the QRS complex. (See Chapter 58, page 264.)
21. A 2 yr old boy arrives in the emergency department after a seizure. On presentation his vital
signs are: temperature 40.2°C, heart rate 200 beats/min, respiratory rate 52 breaths/min. His
laboratory values are: arterial pH 7.2, serum bicarbonate 6 mmol/L, arterial PCO2 18 mm Hg,
sodium 148 mmol/L, potassium 3.1 mmol/L, WBC count 10,200/mm
3
. On the basis of the
history obtained and the presentation, you suspect that an accidental ingestion has occurred.
The most likely toxin is:
□A tricyclic antidepressant
□Acetaminophen
□Cocaine
□An organophosphate insecticide
■A salicylate
description Salicylate ingestions are classified as acute or chronic, and acute toxicity is far more common in pediatric
patients. Early signs of acute salicylism include nausea, vomiting, diaphoresis, and tinnitus. Moderate salicylate toxicity can
manifest as tachypnea and hyperpnea, tachycardia, and altered mental status. The tachycardia results in large part from
marked insensible losses from vomiting, tachypnea, diaphoresis, and uncoupling of oxidative phosphorylation. Thus, careful
attention should be paid to volume status and early volume resuscitation in the significantly poisoned patient. Signs of severe
salicylate toxicity include hyperthermia, coma, and seizures. Chronic salicylism can have a more insidious presentation, and
patients can show marked toxicity at significantly lower salicylate levels than in acute toxicity. The classic blood gas of
salicylate toxicity reveals a primary respiratory alkalosis and a primary, anion gap, metabolic acidosis. Hyperglycemia
(early) and hypoglycemia (late) have been described. Abnormal coagulation studies, clinically manifested as bleeding and
easy bruising, also may be seen. (See Chapter 58, page 260.)
22. A 2 yr old boy is noted to be drinking from a container filled with kerosene. He immediately
coughs, becomes tachypneic, and is brought to the hospital. The best approach to his treatment
is to:
□Induce emesis
□Perform nasogastric tube lavage
□Instill mineral oil
□Administer steroids
■None of the above
description The most important manifestation of hydrocarbon toxicity is aspiration pneumonitis. Aspiration usually occurs
during coughing and gagging at the time of ingestion or vomiting after the ingestion. The propensity of a hydrocarbon to
cause aspiration pneumonitis is inversely proportional to its viscosity. Compounds with low viscosity, such as mineral spirits,
naphtha, kerosene, gasoline, and lamp oil, spread rapidly across surfaces and cover large areas of the lungs when
aspirated. Only small quantities (<1 mL) of low-viscosity hydrocarbons need be aspirated to produce significant injury.
Respiratory symptoms can remain mild or progress rapidly to acute respiratory distress syndrome (ARDS) and respiratory
failure. Emesis and lavage are contraindicated given the risk of aspiration. Activated charcoal is not useful because it does
not bind the common hydrocarbons and can also induce vomiting. If hydrocarbon-induced pneumonitis develops,
respiratory treatment is supportive. Neither corticosteroids nor prophylactic antibiotics have shown any clear benefit. (See
Chapter 58, page 267.)
23. A 2 yr old child is found playing with a can of crystalline drain cleaner. The child's mother
telephones you for help. There are several crystals in the mouth, which you have the mother
wash out. The next step in treatment should be to:
□Have the mother administer lemon juice or orange juice to neutralize the alkaline crystals and come to your office
□Have the mother administer water or milk and call you back in 2 hr
■Have the mother administer water or milk and bring the child in for esophagoscopy
□Simply observe the child because the crystals are so bitter that the child was trying to spit them out when the mother
called, and therefore no problems should occur
□Administer ipecac at home and bring the child in to see you
description Alkalis produce a liquefaction necrosis, allowing further tissue penetration of the toxin and setting the stage for
possible perforation. Ingestion of caustic materials can produce injury to the oral mucosa, esophagus, and stomach. Patients
can have significant esophageal injury even in the absence of visible oral burns. Symptoms include pain, drooling, vomiting,
abdominal pain, and difficulty swallowing or refusal to swallow. Laryngeal injury can manifest as stridor and respiratory
distress, necessitating intubation. In the most severe cases, patients can present in shock after perforation of a hollow
viscus. Initial treatment of caustic exposures includes thorough removal of the product from the skin or eye by flushing with
water. Emesis and lavage are contraindicated. Activated charcoal should not be used because it does not bind these agents
and can predispose the patient to vomiting and subsequent aspiration. Endoscopy should be performed within 12-24 hr of
ingestion in symptomatic patients or those in whom injury is suspected on the basis of history and known characteristics of
the ingested product. (See Chapter 58, page 266.)
24. A 16 yr old, 165-lb patient reports consuming 20-40 325-mg capsules containing
acetaminophen 1 hr ago. The most appropriate approach to treatment is to:
□Measure the plasma level and determine potential toxicity from the level on the nomogram
□Wait until 4 hr after ingestion to measure the plasma level and do nothing else
■Administer activated charcoal immediately and measure the plasma level of acetaminophen 4 hr after ingestion
□Send the patient home because an ingestion of this magnitude is not toxic
□Administer N-acetylcysteine at a dose of 140 mg/kg
description Initial treatment should focus on the ABCs and consideration of decontamination with activated charcoal in
patients who present within 1-2 hr of ingestion. The antidote for acetaminophen poisoning is NAC, which works primarily
via replenishing hepatic glutathione stores. NAC therapy is most effective when initiated within 8 hr of ingestion. However,
there is no demonstrated benefit to giving NAC before the 4 hr postingestion mark. Thus, patients who present early after
ingestion should have a 4-hr level drawn and decision to initiate NAC should be based on this level. (See Chapter 58, page
259.)
25. A 2 yr old child presents in the emergency department after the reported ingestion of a
mouthful of lamp oil. The child reportedly vomited once at home. The child has a heart rate of
160 beats/min, a respiratory rate of 48 breaths/min, and a temperature of 37.2°C. A chest film
is read as normal. The most appropriate therapy for this child is to:
□Administer syrup of ipecac
□Administer activated charcoal
□Remove any ingested lamp oil by gastric lavage
■Admit the child for observation and supportive care
□Discharge the child home with a follow-up office visit in the morning
description The most important manifestation of hydrocarbon toxicity is aspiration pneumonitis. Aspiration usually occurs
during coughing and gagging at the time of ingestion or vomiting after the ingestion. The propensity of a hydrocarbon to
cause aspiration pneumonitis is inversely proportional to its viscosity. Compounds with low viscosity, such as mineral spirits,
naphtha, kerosene, gasoline, and lamp oil, spread rapidly across surfaces and cover large areas of the lungs when
aspirated. Only small quantities (<1 mL) of low-viscosity hydrocarbons need be aspirated to produce significant injury.
Chest radiographs may initially be normal, but they often show abnormalities within 6 hr of exposure in patients who have
aspirated. Respiratory symptoms can remain mild or progress rapidly to acute respiratory distress syndrome (ARDS) and
respiratory failure. Emesis and lavage are contraindicated given the risk of aspiration. Activated charcoal is not useful
because it does not bind the common hydrocarbons and can also induce vomiting. If hydrocarbon-induced pneumonitis
develops, respiratory treatment is supportive. (See Chapter 58, page 267.)
26. A teenage girl presents in the emergency department with the story that she got upset with her
boyfriend and swallowed a "handful of aspirin" about 4 hr previously. One hour afterward,
after she began vomiting, she confessed to her mother what she had done. On examination the
patient has normal vital signs and is asymptomatic except for the complaint of nausea. A
serum salicylate level is ordered, but the laboratory reports no salicylates in her blood. The
most appropriate next step in management is to:
□Discharge the patient home
■Order an acetaminophen level
□Request a psychiatric consultation
□Send a second sample for salicylate determination
□Order an abdominal radiograph to look for pills in the stomach
description Acetaminophen is a widely available medication and a commonly detected co-ingestant with the potential for
severe toxicity. Given that patients might initially be asymptomatic and might not report acetaminophen as a co-ingestant, an
acetaminophen level should be checked in all patients who present after an intentional exposure or ingestion. (See Chapter
58, page 253.)
27. Brain perfusion pressure generally equals:
■Mean arterial pressure minus intracranial pressure
□Diastolic blood pressure minus intracranial pressure
□Intracranial blood pressure minus systolic blood pressure
□Systolic blood pressure minus diastolic blood pressure
description The perfusion pressure of the brain (cerebral perfusion pressure [CPP]) is equal to the pressure of blood
entering the cranium (mean arterial pressure [MAP]) minus the ICP, in most cases. (See Chapter 63, page 296.)
28. Severe traumatic brain injury is characterized by a Glasgow Coma Score (GCS) of:
□0
□1-3
■3-8
□9-12
description The hallmark of severe TBI is coma (GCS score 3-8). Often, coma is seen immediately after the injury and is
sustained. In some cases, such as with an epidural hematoma, a child may be alert at presentation but the condition may
deteriorate after a period of hours. A similar picture can be seen in children with diffuse swelling, in whom a talk-and-die
scenario has been described. Clinicians also should not be lulled into underappreciating the potential for deterioration of a
child with moderate TBI (GCS score 9-12) with a significant contusion, because progressive swelling can potentially lead
to devastating complications. In the comatose child with severe TBI, the second key clinical manifestation is the
development of intracranial hypertension. Increased ICP can be appropriately managed only with continuous ICP
monitoring. The development of brain swelling is progressive. Significantly raised ICP (>20 mm Hg) can occur early after
severe TBI, but peak ICP generally is seen at 48-72 hr. Need for ICP-directed therapy may persist for longer than a
week. A few children have coma without increased ICP, resulting from axonal injury or brainstem injury. (See Chapter 63,
page 298.)
29. Children with moderate to severe traumatic brain injury require intracranial pressure
monitoring and treatment in the critical care unit to prevent progressive swelling. First-line
therapies include all of the following EXCEPT:
■Hypothermia
□Head in midline position
□Sedation and analgesia
□Controlled mechanical ventilation
□Mannitol therapy for ICP >20 mm Hg
description First-tier therapy includes elevation of the head of the bed, ensuring midline positioning of the head, controlled
mechanical ventilation, and sedation and analgesia (i.e., benzodiazepines and narcotics). If neuromuscular blockade is
needed, it may be desirable to monitor the EEG continuously because status epilepticus can occur; this complication will not
be recognized in a paralyzed patient and is associated with raised ICP and unfavorable outcome. If a ventricular rather than
parenchymal catheter is used to monitor ICP, therapeutic CSF drainage is available. Other first-tier therapies include the
osmolar agents mannitol and hypertonic saline given in response to ICP spikes. If ICP remains refractory to treatment,
careful reassessment of the patient is needed to rule out unrecognized hypercarbia, hypoxemia, fever, hypotension,
hypoglycemia, pain, and seizures. Repeat imaging should be considered to rule out a surgical lesion. Guidelines-based
second-tier therapies for refractory raised ICP are available, but evidence favoring a given second-tier therapy is limited.
In some centers, decompressive craniectomy is used. Others use a pentobarbital infusion. Mild hypothermia (32-34°C) to
control refractory ICP can be induced and maintained by means of surface cooling. Refractory raised ICP can also be
treated with hyperventilation (PaCO2 = 25-30 mm Hg). Other second-tier therapies such as lumbar CSF drainage are
options. (See Figure 63-12; also Chapter 63, page 300.)
30. Which of the following statements regarding cooling treatment for perinatal hypoxic ischemic
encephalopathy is not true?
□Candidates for cooling therapy have perinatal asphyxia, Apgar score of 0-3, acidosis, and neurologic dysfunction
□Coagulopathy is a contraindication for cooling therapy
□Goal core temperature is 32-34°C during cooling therapy.
□During cooling therapy, shivering should be prevented with sedation and neuromuscular blockade
■After 72 hours, the patient should be rapidly rewarmed to 36°C
description Similarly, in perinatal asphyxia, fetal acidosis, an Apgar score of 0-3 after 5 min, neurologic dysfunction,
and/or abnormal EEG findings are criteria for use of cooling therapy in term newborn infants. Exclusion criteria have
included coagulopathy, bleeding, and hemodynamic instability. According to the American Heart Association (AHA)
guidelines (predominantly for adults after a cardiac arrest when the initial event was associated with ventricular fibrillation),
cooling should be initiated as soon as possible after return of spontaneous circulation but may be beneficial even if delayed
(4-6 hr); it should be induced by means of surface cooling with cooling blankets; application of ice packs to the groin,
axillae, and neck; use of wet towels; and fanning. Infusion of 20 mL/kg IV of ice-cold saline over 30 min can be considered
in children and may reduce core temperature by approximately 2°C. If hypothermia is used in children, a temperature of
32-34°C should be used. The rewarming rate should be no greater than 1°C every 4-6 hr. In perinatal asphyxia, cooling
should be maintained for 72 hr. Shivering should be prevented with sedation and neuromuscular blockade. Temperature
should be continuously monitored. Hypothermia in children has been associated with an increased risk for neutropenia and
sepsis. (See Chapter 63, page 302.)
31. First-line medications for status epilepticus include all of the following, EXCEPT:
□Rectal diazepam
□Intravenous lorazepam
□Intravenous fosphenytoin
□Intravenous phenobarbital
■Intravenous valproic acid
description Several antiepileptic drugs have been advocated as first-line therapies for status epilepticus, including
benzodiazepines (rectal diazepam, IV midazolam or lorazepam), phenytoin (or fosphenytoin), and barbiturates
(phenobarbital). Therapy is adjusted for both symptoms and EEG evidence of seizures. In the PICU, for refractory cases, a
continuous infusion of barbiturates and/or benzodiazepines may be necessary. For refractory cases that progress to this
level of therapeutic intensity, respiratory support and hemodynamic monitoring and/or support should already be in place.
As therapy escalates, continuous EEG monitoring should be considered to help titrate therapy. For refractory status
epilepticus, newer therapies include mapping of the seizure focus followed by neurosurgical resection, IV lidocaine, or
levetiracetam. (See Chapter 63, page 303.)
32. The following statements about stroke in children are true EXCEPT:
□The most common causes of ischemic stroke in children are sickle cell disease and heart disease
■Atherosclerotic plaque migration is responsible for the majority of ischemic strokes
□Stroke presents clinically with focal neurologic deficits or coma in children
□A diagnosis of stroke requires CT or MRI findings
□Differential diagnosis of stroke in children includes complex migraine, seizures, and encephalitis
description The predominant causes of ischemic stroke in children are sickle cell disease and heart disease (either acquired
or congenital), which are responsible for approximately 50% of strokes after the neonatal period. Ischemic strokes in
children are generally not the result of atherosclerotic plaque migration, as they are in adults. Instead, damage to the intima
of cerebral arteries can form a thrombotic nidus. In sickle cell disease, chronic turbulent blood flow likely leads to vascular
damage. In intracerebral hemorrhage, blood vessel wall integrity is compromised, leading to extravasation of blood into the
parenchyma or dural spaces. The usual pathology in children with heart disease is embolism from diseased valves (or
intracardiac devices) and right-to-left shunts that leads to cerebrovascular occlusion. (See Chapter 63, page 303.)
33. Which of the following statements about ventilator-associated pneumonia (VAP) is NOT true?
□VAP is multifactorial and causes include endotracheal tube colonization, aspiration of gastric secretions, and suppression
of cough reflexes
■VAP is largely unpreventable
□Fever, leukocytosis, and infiltrate on chest radiography support a diagnosis of VAP
□Empirical treatment of VAP should include nosocomial pathogens
description Elevation of the head of the bed to 30 degrees after initiation of mechanical ventilation and use of a protocol
for oral decontamination during mechanical ventilation are two means of reducing the risk for VAP. The most effective
strategy to minimize any ventilator complication is regular assessment of extubation readiness and liberation from mechanical
ventilation as soon as clinically possible. (See Chapter 65, page 328.)
34. The initial ventilator settings are determined by:
■The patient's underlying disease
□The patient's preferences
□Standard order sets
□Attempts to normalize the blood gases
description The choice of mode of ventilation depends on how much ventilator-patient interaction is desired and the
disease entity that is being treated. (See Chapter 65, page 327.)
35. Patients with severe forms of reactive airways disease (e.g., asthma) who require mechanical
ventilation may benefit from which of the following initial ventilator parameters?
□Rapid rates, short inspiratory times, and tidal volumes <6 mL/kg
□Low rates, prolonged inspiratory/expiratory times, and low tidal volumes (<6 mL/kg)
■Low rates, prolonged inspiratory/expiratory times, and moderate tidal volumes (10-12 mL/kg)
□Low rates, prolonged inspiratory/expiratory times, and high tidal volumes (>15 mL/kg)
description In a patient with relatively normal lungs, an age-appropriate ventilator rate and a tidal volume of 7-10 mL/kg
would be appropriate initial settings. Diseases associated with decreased time constants (decreased static compliance, e.g.,
ARDS, pneumonia, pulmonary edema) are best treated with small (6 mL/kg) tidal volume and relatively rapid rates (25-40
breaths/min). Diseases associated with prolonged time constants (increased airway resistance, e.g., asthma, bronchiolitis)
are best treated with relatively slow rates and higher (10-12 mL/kg) tidal volume. (See Chapter 65, page 327.)
36. Per the IOM, high-quality health care by definition must be:
□Timely and effective
□Equitable and efficient
□Safe and patient centered
□A and C
■All of the above
description To measure health care quality, the IOM has identified Six Dimensions of Quality, all of which relate to quality
of care. The Six Dimensions of Quality are effectiveness, efficiency, equity, timeliness, patient safety, and patient-centered
care. The IOM emphasizes the concept that all Six Dimensions of Quality need to be met for the provision of high-quality
health care. Health care that maximizes outcomes but is not efficient (i.e., not cost-effective) is not quality care. Health care
that is highly efficient but limits access also is not high quality. These concepts can be viewed as the overall value
proposition—that is, the value created for a patient. (See e2-1.)
37. Which of the following is a step in the rapid cycle of improvement (PDSA)?
□Perform
□Define
■Study
□Attend
description The Model for Improvement can be implemented using a framework of rapid cycle of improvement also
known as the Plan-Do-Study-Act (PDSA) cycle (Fig. 2-1). The PDSA cycle is typically aimed at testing small changes
and then studying the results to plan and implement the next cycle of change (i.e., multiple PDSA cycles build on previous
learning from PDSAs). Valuable information can be obtained from PDSA cycles that are successful and those that are not,
to help plan the next iteration of the PDSA cycle. The PDSA cycle specifically requires that improvements be data driven.
This is important because many clinicians attempt to make changes for improvement in their practice but do not emphasize
the importance of data collection. (See e2-3.)
38. Developing a culture of safety relies on all of the following concepts EXCEPT:
□De-emphasizing hierarchy and encouraging contributions of all members of the health care team
□Excellent communication including patient hand-outs
□Application of human factors engineering to systems and processes
■Discouraging anonymity in the reporting of adverse events
description The biggest challenge in making the health system safer is changing the culture from one of treating errors as
personal failures to one of treating errors as opportunities to improve the system. Organizations need to foster a culture of
learning in which each individual will feel accountable for ensuring a safe and quality program, communication is open, and
teamwork is valued. Reporting of errors should be valued, reports of adverse events should be handled confidentially, and
those who report errors should be protected from discovery. Developing a culture of learning involves the compassionate
and appropriate disclosure of system failures and medical errors to patients and families. (See e2-6).
39. A 4 yr old girl sustained a 40% 2nd- and 3rd-degree total body surface area (BSA) burn from
scalding hot water. Of the following, the most therapeutic approach is:
□Aggressive use of topical antibacterial agents with frequent dressing changes
□Use of enzymatic debridement ointment
■Excision of the burn wounds and grafting
□Use of topical analgesics
description Deep 2nd-degree burns of >10% of BSA benefit from early excision and grafting. To improve outcome,
sequential excision and grafting of 3rd-degree and deep 2nd-degree burns is required in children with large burns. Prompt
excision with immediate wound closure is achieved with autografts, which are often meshed to increase the efficiency of
coverings. (See Chapter 68, page 353.)
40. A 10 yr old boy sustained 30% BSA burns and had been requiring dressing changes for
physical therapy. Which of the following regimens will provide the best pain management?
□IV morphine bolus
□Oral ibuprofen
□Oral acetaminophen
■Oral morphine and oral lorazepam
description From the onset of treatment, preemptive pain control during dressing changes is of paramount importance.
Opiate analgesia, prescribed in an adequate dose and timed to cover dressing changes, is essential to comfort management.
Anxiolytic medication added to the analgesic is usually helpful and has more than a synergistic effect. (See Chapter 68,
page 355.)
41. Indications for admission to the hospital after a burn injury may include all of the following
EXCEPT:
□Suspected child abuse
□Electric burns through an extremity
□Perineal burns
□Poor follow-up
■No tetanus immunization
□Inhalation injury
description Burns covering >10-15% of total BSA, burns associated with smoke inhalation, burns resulting from high-
tension (voltage) electrical injuries, and burns associated with suspected child abuse or neglect should be treated as
emergencies, and the child hospitalized. Small 1st- and 2nd-degree burns of the hands, feet, face, perineum, and joint
surfaces also require admission if close follow-up care is difficult to provide. Children who have been in enclosed-space
fires and those who have face and neck burns should be hospitalized for at least 24 hr for observation for signs of central
nervous system (CNS) effects of anoxia from carbon monoxide poisoning and pulmonary effects from smoke inhalation.
(See Table 68-2, Chapter 68, page 349.)
42. A burn wound characterized by the absence of painful sensation, bleeding, or capillary refilling
is best classified as:
□First degree
□Moderate to severe
□Second degree
□Midlevel
■Full thickness
description Full-thickness, or 3rd-degree, burns involve destruction of the entire epidermis and dermis, leaving no residual
epidermal cells to repopulate the damaged area. The absence of painful sensation and capillary filling demonstrates the loss
of nerve and capillary elements. The wound cannot epithelialize and can heal only by wound contraction or skin grafting.
(See Table 68-5, Chapter 68, page 351.)
43. Which of the following statements regarding predictive genetic testing is true?
□Predictive testing is genetic testing done in a person who is symptomatic for a genetic disorder
□Predictive testing in children is ethically acceptable if the parents desire it
□Federal law prevents companies from denying disability insurance based on a positive genetic test
■If a person has a positive predictive genetic test for a disease, he/she may not ever develop that disease
□None of the above
description Predictive genetic testing involves performing a test in a person who is at risk for developing a genetic disorder
(presymptomatic), usually on the basis of family history, yet who does not manifest signs or symptoms. A major caution
with predictive testing is that the presence of a gene mutation does not necessarily mean that the disease will develop. Many
of the disorders with age-dependent penetrance display incomplete penetrance. A person who inherits a mutation might
never develop signs of the disorder. In the USA, the Genetic Information Nondiscrimination Act of 2008 protects
individuals from genetic discrimination at the hands of health insurers and employers but does not extend protection against
discrimination from providers of life, disability, or long-term care insurance. (See Chapter 72, page 377.)
44. Genetic counseling is indicated in which of the following clinical scenarios?
□Abnormal prenatal quad screen
□Infant born with hypoplastic left heart disease
□History of multiple miscarriages
□Two cousins planning to marry
□Child diagnosed with cystic fibrosis
■All of the above
description See Table 72-2, Indications for Genetic Counseling. (See Chapter 72, page 378.)
45. Which statement regarding treatment of genetic disorders is NOT true?
■Physiologic therapy for genetic disease such as PKU is curative if started early
□Newborn screening is important because early identification of genetic disorders allows early treatment
□Enzyme replacement is available for Gaucher disease and Pompe disease
□A bone marrow transplant may potentially cure thalassemia major
□Gene-transfer vehicles include viruses
description Physiologic therapies attempt to ameliorate the phenotype of a genetic disorder by modifying the physiology of
the affected individual. The underlying defect itself is not altered by treatment. Physiologic therapies are used in the
treatment of inborn errors of metabolism. These include dietary manipulation, such as avoiding phenylalanine by persons
with phenylketonuria; coenzyme supplementation for some patients with methylmalonic acidemia and mitochondrial
diseases; stimulation of alternative pathways to excrete ammonia for those with urea cycle disorders; bisphosphonate
treatment for those with osteogenesis imperfecta to reduce bone fractures; and avoiding cigarette smoking by persons with
α1-antitrypsin deficiency. Physiologic treatments can be highly effective, but they usually need to be maintained for a
lifetime because they do not affect the underlying genetic disorder. Many of these treatments are most effective when begun
early in life before irreversible damage has occurred. This is the rationale for comprehensive newborn screening for inborn
errors of metabolism. (See Chapter 72, page 379.)
46. Which statement regarding genetic disorders of metabolism is NOT true?
■In severe disorders, the affected infant may be sick at birth
□Most genetic metabolic diseases are treatable
□The majority of genetic metabolic disease have autosomal recessive inheritance
□Early diagnosis is crucial to good prognosis for most disorders
□Tandem mass spectrometry may identify a large number of disorders with just a few drops of blood
description In genetic disorders of metabolism, the affected infant is normal at birth and becomes symptomatic later in life.
This differentiates these infants from those who appear sick at birth due to birth trauma, intrauterine insults, chromosomal
abnormalities, or other genetic diseases. Severe forms of genetic disorders usually become clinically apparent in the
newborn period or shortly thereafter. The majority of conditions are inherited as autosomal recessive traits. Most of the
genetic metabolic conditions can be controlled successfully by some form of therapy, and a few can be potentially cured by
the use of bone marrow or liver transplants. This underlines the importance of early diagnosis, which can be achieved
through screening of all newborn infants. Tandem mass spectrometry (MS/MS) requires a few drops of blood to be placed
on a filter paper and mailed to a central laboratory for assay. A large number of genetic conditions can be identified by this
method. (See Chapter 78, page 416).
47. Signs and symptoms of inborn errors of metabolism include all of the following EXCEPT:
□Vomiting
■Diarrhea
□Lethargy
□Poor feeding
□Seizures
description Pediatricians should familiarize themselves with early manifestations of genetic metabolic disorders, because
(1) severe forms of some of these conditions may cause symptoms before the results of screening studies become available
and (2) the current screening methods, although quite extensive, identify a small number of all inherited metabolic
conditions. In the newborn period, the clinical findings are usually nonspecific and similar to those seen in infants with sepsis.
A genetic disorder of metabolism should be considered in the differential diagnosis of a severely ill newborn infant, and
special studies should be undertaken if the index of suspicion is high. Signs and symptoms such as lethargy, poor feeding,
convulsions, and vomiting may develop as early as a few hours after birth. (See Chapter 78, page 416 and Fig. 78-1.)
48. Initial laboratory studies to investigate for metabolic disease in an ill infant should include:
□Lactate, glucose, bicarbonate
□Glucose, calcium, pH
□Sodium, glucose, bicarbonate
■pH, bicarbonate, ammonia
□Complete blood cell count, sodium, potassium
description Measurements of serum concentrations of ammonia, bicarbonate, and pH are often very helpful initially in
differentiating major causes of genetic metabolic disorders (see Fig. 78-1). An elevated blood ammonia level is usually
caused by defects of urea cycle enzymes. Infants with elevated blood ammonia levels from urea cycle defects commonly
have normal serum pH and bicarbonate values; without measurement of the blood ammonia level their defect may remain
undiagnosed and they may succumb to their disease. Elevation of serum ammonia levels also is observed in some infants
with certain organic acidemias. These infants are severely acidotic because of accumulation of organic acids in body fluids.
When blood ammonia, pH, and bicarbonate values are normal, other aminoacidopathies (e.g., hyperglycinemia) or
galactosemia should be considered; galactosemic infants also may manifest cataracts, hepatomegaly, ascites, and jaundice.
(See Chapter 78, page 417).
49. A 2 day old boy manifests poor feeding, vomiting, and lethargy leading to coma. Laboratory
data reveal respiratory alkalosis and hyperammonemia. The urine orotic acid level is also
elevated. The most likely diagnosis is:
□Methylmalonic acidemia
□Carbamoylphosphate synthase deficiency
■Ornithine transcarbamylase (OTC) deficiency
□Galactosemia
□Reye syndrome
description Many genetic disorders of metabolism increase plasma ammonia. In neonatal hyperammonemia, most of the
symptoms are related to brain dysfunction due to the elevated ammonia. The affected infant is normal at birth but becomes
symptomatic within a few days of protein feeding. Refusal to eat, vomiting, tachypnea, and lethargy can quickly progress to
a deep coma. Convulsions are common. See Figure 79-13 for an algorithm to diagnose the cause of hyperammonemia. In
the case of OTC deficiency, laboratory studies will not demonstrate acidosis. A marked increase in urinary orotic acid
distinguishes OTC deficiency from other disorders. (See Figure 79-13 in Chapter 79, page 449.)
50. For most lysosomal storage disorders, carrier identification and prenatal diagnosis are
available; a specific diagnosis is essential to permit genetic counseling. Which of the following
disorders is X-linked?
□Niemann-Pick disease
■Fabry disease
□Tay-Sachs disease
□Krabbe disease
□Gaucher disease
description For all of the lysosomal storage disorders, inheritance is autosomal recessive except for X-linked Fabry
disease. (See Chapter 80, page 483.)
51. GM1 gangliosidosis is characterized by all of the following EXCEPT:
□Psychomotor retardation
□Angiokeratomas
□Hepatosplenomegaly
□Frontal bossing
■Peripheral neuropathy
description The clinical manifestations of the infantile form of GM1 gangliosidosis may be evident in the newborn infant as
hepatosplenomegaly, edema, and skin eruptions (angiokeratomas). It most frequently presents in the first 6 mo of life with
developmental delay followed by progressive psychomotor retardation and the onset of tonic-clonic seizures. A typical
facies is characterized by low-set ears, frontal bossing, a depressed nasal bridge, and an abnormally long philtrum. Up to
50% of patients have a macular cherry-red spot. Hepatosplenomegaly and skeletal abnormalities similar to those of the
mucopolysaccharidoses, including anterior beaking of the vertebrae, enlargement of the sella turcica, and thickening of the
calvarium, are present. (see Chapter 80, page 483.)
52. A 4 mo old girl presents with developmental delay, an exaggerated startle response to loud
noise, and macrocephaly. On physical examination, the child has decreased eye contact and a
cherry-red spot in each retina. The most likely diagnosis is:
■GM2 gangliosidosis
□Gaucher disease
□Fabry disease
□Galactosemia
□Glycogen storage disease, type I
description The GM2 gangliosidoses include Tay-Sachs disease and Sandhoff disease; each results from the deficiency of
β-hexosaminidase activity and the lysosomal accumulation of GM2 gangliosides, particularly in the central nervous system.
The clinical manifestations of Sandhoff disease are similar to those for Tay-Sachs disease. The diagnosis of infantile Tay-
Sachs disease and Sandhoff disease is usually suspected in an infant with neurologic features and a cherry-red spot.
Affected infants usually develop normally until 4 to 5 mo of age when decreased eye contact and an exaggerated startle
response to noise (hyperacusis) are noted. Macrocephaly, not associated with hydrocephalus, may develop. In the 2nd yr
of life, seizures develop that may be refractory to anticonvulsant therapy. Neurodegeneration is relentless, with death
occurring by the age of 4 or 5 yr. (See Chapter 80, page 482.)
53. A 15 yr old presents with chronic fatigue and severe bone pain of 1 year's duration. He has
hepatosplenomegaly and a normal retinal examination. Laboratory studies reveal normocytic
anemia and thrombocytopenia. Radiographs of the distal femur reveal Erlenmeyer flask
deformities. The most likely diagnosis is:
□Lymphoma
■Gaucher disease
□Sandhoff disease
□GM3 gangliosidosis
□Sickle cell anemia
description Gaucher disease is a multisystemic lipidosis characterized by hematologic abnormalities, organomegaly, and
skeletal involvement, the latter usually manifesting as bone pain and pathologic fractures. Clinical manifestations of type 1
Gaucher disease have a variable age at onset, from early childhood to late adulthood, with most symptomatic patients
presenting by adolescence. At presentation, patients may have bruising from thrombocytopenia, chronic fatigue secondary
to anemia, hepatomegaly with or without elevated liver function test results, splenomegaly, and bone pain. Most patients
develop radiologic evidence of skeletal involvement, including an Erlenmeyer flask deformity of the distal femur. (See
Chapter 80, page 487.)
54. Fabry disease is characterized by all of the following EXCEPT:
□Acroparesthesias
□Corneal opacities
□Cardiomyopathy
■Mental retardation
□Angiokeratomas
description Fabry disease is an X-linked inborn error of glycosphingolipid metabolism characterized by angiokeratomas
(telangiectatic skin lesions), hypohidrosis, corneal and lenticular opacities, acroparesthesias, and vascular disease of the
kidney, heart, and/or brain. (See Chapter 80, page 489.)
55. A 14 mo old girl presents with irritability, poor walking, genu recurvatum, and hypotonia.
Physical examination reveals absent deep tendon reflexes. Laboratory data reveal an increased
cerebrospinal fluid protein level and decreased nerve conduction velocities. The most likely
diagnosis is:
■Metachromatic leukodystrophy
□I cell disease
□GM3 gangliosidosis
□Fabry disease
□Neuromyopathic Gaucher disease
description The clinical manifestations of the late infantile form of metachromatic leukodystrophy, which is most common,
usually present between 12 and 18 mo of age as irritability, inability to walk, and hyperextension of the knee, causing genu
recurvatum. The clinical progression of the disease relates to the pathologic involvement of both central and peripheral
nervous system, giving a mixture of upper and lower motor neuron and cognitive and psychiatric signs. Deep tendon
reflexes are diminished or absent. Gradual muscle wasting, weakness, and hypotonia become evident and lead to a
debilitated state. As the disease progresses, nystagmus, myoclonic seizures, optic atrophy, and quadriparesis appear, with
death in the 1st decade of life. On evaluation, decreased nerve conduction velocities, increased cerebrospinal fluid protein,
metachromatic deposits in sampled segments of sural nerve, and metachromatic granules in urinary sediment are all
suggestive of the disorder. (See Chapter 80, page 490.)
56. A 15 yr old Ashkenazi Jewish girl is seen because of chronic fatigue. On examination, she
seems pale and thin and has a somewhat large abdomen. Her spleen is felt in the iliac fossa. She
is mentally alert and has a history of normal development and normal school performance. Her
blood cell count shows hemoglobin, 9.0 g/dL and a white blood cell count of 3,000/mm
3
, with
normal differential cell count and no abnormal cells. Platelet count is 60,000/μL. The most
likely diagnosis is:
□Tay-Sachs disease
□Niemann-Pick disease type A
■Gaucher disease type I
□Mucolipidosis type IV
□Canavan disease
description Clinical manifestations of type I Gaucher disease have a variable age at onset, from early childhood to late
adulthood, with most symptomatic patients presenting by adolescence. Type I, which accounts for 99% of cases, has a
striking predilection for Ashkenazi Jews. Gaucher disease should be considered in the differential diagnosis of patients with
unexplained organomegaly, who bruise easily, have bone pain, or have a combination of these conditions. Evaluation may
show bruising from thrombocytopenia, chronic fatigue secondary to anemia, hepatomegaly with or without elevated liver
function test results, splenomegaly, and bone pain. (See Chapter 80, page 487.)
57. A 7 mo old boy has been healthy and developing normally since birth. His mother now reports
that he has decreased eye contact with her, even during feedings. The infant also startles very
easily when there is a loud noise in the house. Of the following, the most appropriate diagnostic
test to confirm the etiology of these findings is the measurement of:
■Leukocyte β-hexosaminidase A activity
□Serum concentration of amino acids
□Serum concentration of ammonia
□Urinary mucopolysaccharides
□Urinary organic acids
description Patients with the infantile form of GM2 gangliosidoses have clinical manifestations in infancy including loss of
motor skills, increased startle reaction, and macular pallor and retinal cherry-red spots. Affected infants usually develop
normally until 4 to 5 mo of age when decreased eye contact and an exaggerated startle response to noise (hyperacusis) are
noted. Macrocephaly, not associated with hydrocephalus, may develop. Definitive diagnosis is made by determination of
β-hexosaminidase A and B activities in peripheral leukocytes. The two GM2 gangliosidoses are distinguished by the
enzymatic assay, because in Tay-Sachs disease only the β-hexosaminidase A isozyme is deficient, whereas in Sandhoff
disease both the β-hexosaminidase A and B isozymes are deficient. (See Chapter 80, page 485.)
58. A 3 yr old boy was normal at birth but developed progressive coarsening of the face and
developmental delay. In addition to coarse facies and mental retardation, the physical
examination reveals a cardiac systolic regurgitant murmur, hepatomegaly, joint stiffness, and
short stature. CT scan reveals hydrocephalus. There is no corneal clouding. All affected family
members have been boys. The most likely diagnosis is:
□Hurler syndrome
□Galactosemia
□Metachromatic leukodystrophy
■Hunter syndrome
□Sanfilippo syndrome
description Hunter disease (MPS II) is an X-linked disorder and manifests almost exclusively in males. It has been
observed in a few females and this is explained by skewed inactivation of the X chromosome carrying the normal gene.
Patients with severe MPS II have features similar to those of Hurler disease except for the lack of corneal clouding and the
somewhat slower progression of somatic and central nervous system deterioration. Coarse facial features, short stature,
dysostosis multiplex, joint stiffness, and mental retardation manifest between 2 and 4 yr of age. (See Chapter 82, page
511.)
59. Hurler disease is characterized by all of the following EXCEPT:
□Hepatosplenomegaly
□Coarse facies
□Dysostosis multiplex
■X-linked inheritance
□Corneal clouding
description Diagnosis of Hurler disease is usually made between 6 and 24 mo of age with evidence of
hepatosplenomegaly, coarse facial features, corneal clouding, large tongue, prominent forehead, joint stiffness, short stature,
and skeletal dysplasia known as dysostosis multiplex. Acute cardiomyopathy has been found in some infants younger than 1
yr of age. Inheritance is autosomal recessive. (See Chapter 82, page 511.)
60. According to the International Classification of Diseases, major causes of U.S. infant mortality
include all of the following EXCEPT:
□Sudden infant death syndrome
□Disorders relating to prematurity
□Congenital malformations
□Disorders relating to maternal complications of pregnancy
■Nonaccidental trauma
description In the USA, –50% of infant deaths in 2008 were due to 4 conditions (classified according to the International
Classification of Diseases, 10th revision): congenital malformations (20.1%), disorders relating to prematurity and
unspecified LBW (16.9%), sudden infant death syndrome (SIDS) (8.2%), and newborns affected by maternal
complications of pregnancy (6.3%). LBW (as a result of preterm delivery and/or IUGR) is a major determinant of both
neonatal and infant mortality rates and, together with congenital anomalies (cardiac, central nervous system, respiratory),
contributes significantly to childhood morbidity. In developing countries, LBW/prematurity, birth asphyxia, and infections
are the major causes of infant deaths. (See e87-2.)
61. All of the following increase the risk of pregnancy EXCEPT:
□Monochorionic twins
□Teenage mother
□Mother > 40 years of age
■Long interpregnancy interval
□Cigarette smoking
description See Table 89-1 for Factors Associated with High-Risk Pregnancy. The lowest neonatal mortality rate occurs
in infants of mothers who receive adequate prenatal care and who are 20-30 yr of age. Pregnancies in both teenagers and
women older than 40 yr, particularly primiparous women, are at increased risk for intrauterine growth restriction, fetal
distress, and intrauterine death. A short interpregnancy interval is associated with increased risk. (See e89-1.)
62. Reproductive technologies are associated with all of the following pregnancy-related risks
EXCEPT:
□Higher risk for multiple fetuses
■Higher risk for group B streptococcal sepsis
□Higher risk for low and very low birthweight
□Higher risk for prematurity
□Higher risk for congenital anomalies
description The use of assisted reproductive technology (in vitro fertilization, intracytoplasmic sperm injection) increases
the risk of perinatal mortality, infant morbidity, prematurity, low and very low birthweight, and cerebral palsy, largely
because of the increase in multiple-fetus pregnancies with such technology; the risks for birth defects are also increased, in
part, because of epigenetic effects on gene expression. (See e89-1.)
63. Oligohydramnios is associated with all of the following EXCEPT:
■Esophageal atresia
□Pulmonary hypoplasia
□Potter syndrome
□Posterior urethral valves
□Skeletal contractures
description Oligohydramnios is associated with congenital anomalies; intrauterine growth restriction; severe renal, bladder,
or urethral anomalies; and drugs that interfere with fetal urination. Oligohydramnios becomes most evident after 20 wk of
gestation, when fetal urination is the major source of amniotic fluid. Rupture of the membranes is the most common cause of
oligohydramnios and must be ruled out if oligohydramnios is suspected, especially if a normal-sized bladder is seen on fetal
ultrasound evaluation. Oligohydramnios causes fetal compression abnormalities such as fetal distress, clubfoot, spadelike
hands, and a flattened nasal bridge. The most serious complication of chronic oligohydramnios is pulmonary hypoplasia.
Polyhydramnios is associated with premature labor, abruptio placentae, multiple congenital anomalies, and fetal
neuromuscular dysfunction or obstruction of the gastrointestinal tract that interferes with reabsorption of the amniotic fluid
that is normally swallowed by the fetus. Increased fetal urination or edema formation is also associated with excessive
amniotic fluid volume. (See Table 89-4 and e89-2.)
64. Amnion nodosum is associated with all of the following EXCEPT:
□Oligohydramnios
■Fetal Candida infection
□Renal agenesis
□Pulmonary hypoplasia
□Flattened facies
description Examination of the fresh placenta, cord, and membranes may alert the physician to a newborn infant at high
risk and may help confirm a diagnosis in a sick infant. Amnion nodosum (granules on the amnion) and oligohydramnios are
associated with pulmonary hypoplasia and renal agenesis, whereas small whitish nodules on the cord suggest a candidal
infection. (See Chapter 90, page 552.)
65. Which statement about twins is not true?
■Monovular twinning appears to be an inherited tendency
□Most twins are born prematurely
□Ninety percent of twins are detected before delivery
□Monochorionic twins may be presumed to be monovular
□The second born twin is at increased risk for respiratory distress syndrome and asphyxia compared with the first born
description The occurrence of monovular twins appears to be independent of genetic influence. Polyovular pregnancies
are more frequent beyond the 2nd pregnancy, in older women, and in families with a history of polyovular twins. (See
Chapter 91, page 553.)
66. Fetal transfusion syndrome results in which of the following?
□Donor twin large and plethoric
□Recipient twin small and anemic
□Donor twin small and anemic
□Recipient twin large and plethoric
□A and B
■C and D
description In the fetal transfusion syndrome, an artery from one twin acutely or chronically delivers blood that is drained
into the vein of the other. The latter becomes plethoric and large, and the former is anemic and small. Generally, with
chronicity, 5 g/dL hemoglobin and 20% body weight differences can be noted in this syndrome. (See Table 91-2, Chapter
91, page 554.)
67. Very-low-birthweight (VLBW) infants are best described as:
□Predominantly growth restricted
■Predominantly premature
□Predominantly post dates
□The result of maternal illness
□The result of placental infarction
description VLBW infants weigh <1500 g and are predominantly premature. In the USA in 2008 the VLBW rates were
approximately 1.46% overall, 3.01% among blacks, and 1.18% among whites. The VLBW rate is an accurate predictor of
the infant mortality rate. VLBW infants account for over 50% of neonatal deaths and 50% of handicapped infants; their
survival is directly related to birthweight, with approximately 20% of those between 500 and 600 g and > 90% of those
between 1250 and 1500 g surviving. The VLBW rate has remained unchanged for black Americans but has increased
among whites, perhaps because of a rise in multiple births among whites. Perinatal care has improved the rate of survival of
VLBW infants. When compared with term infants, VLBW neonates have a higher incidence of rehospitalization during the
1st yr of life for sequelae of prematurity, infections, neurologic complications, and psychosocial disorders. (See Chapter 91,
page 556.)
68. Which of the following statements regarding the assessment of gestational age at birth is NOT
true?
□In the absence of asphyxia, neurologic maturity correlates with gestational age
■The Ballard scoring system is accurate to within 2 days
□If a discrepancy exists between the estimation of gestational age by physical examination and fetal ultrasonographic
evaluation, the infant is at high risk for morbidity and mortality
□Physical features used to assess gestational age include breast buds, ear development, and presence of lanugo
description Neurologic maturity (nerve conduction velocity), in the absence of asphyxia, correlates with gestational age
despite reduced fetal weight. Physical signs may be useful in estimating gestational age at birth. Commonly used, the Ballard
scoring system is accurate to ±2 wk. An infant should be presumed to be at high risk for mortality or morbidity if a
discrepancy exists between the estimation of gestational age by physical examination, the mother's estimated date of her last
menstrual period, and fetal ultrasonographic evaluation. (See Figure 91-6, Chapter 91, page 557.)
69. A term female is born by spontaneous vaginal delivery to a primiparous woman who received 2
doses of meperidine 30 min and 2 hr before an abrupt delivery. The baby is apneic and limp.
The most important, immediate management is to:
□Administer naloxone in the umbilical vein
■Perform bag-mask ventilation
□Administer naloxone in the endotracheal tube
□Begin chest compressions
□Obtain a cord pH level
description Narcosis results from administration of morphine, meperidine, fentanyl, barbiturates, or tranquilizers to the
mother shortly before delivery or from maternal anesthesia given during the 2nd stage of labor. This sequela should be
avoided by the use of appropriate analgesic and anesthetic practices. Treatment includes initial physical stimulation and
securing of a patent airway. If effective ventilation is not initiated, artificial breathing with a bag and mask must be instituted.
At the same time, if the respiratory depression is due to an opiate, naloxone hydrochloride (Narcan), 0.1 mg/kg, should be
given intravenously or intramuscularly. Naloxone is contraindicated in infants born to mothers with opiate addiction because
it precipitates acute neonatal withdrawal with severe seizures. (See Chapter 94, page 575.)
70. Successful ventilation is determined by all of the following EXCEPT:
■Zero reading of end-tidal CO2 measurement
□Pink color
□Rising heart rate
□Symmetric breath sounds
□Good chest rise
description Successful ventilation is signified by adequate chest rise, symmetric breath sounds, improved pink color, heart
rate > 100 beats/min, spontaneous respirations, presence of end-tidal CO2, and improved tone. (See Chapter 94, page
576.)
71. The biopsychosocial model of development, when applied to the child's height, includes all of
the following EXCEPT:
□Genetic endowment
□Personal eating habits
□Access to food
□Parents' beliefs
■Differences between breast milk and formula
description The biopsychosocial model of medicine recognizes biologic factors, social factors, and psychologic factors that
contribute to health and disease. Genetics, environment, economics (cost/access), personal habits, and family beliefs all
contribute in this model. (See e6-1.)
72. All of the following statements regarding a child's temperament are true EXCEPT:
■Temperament is absolute and stable throughout the life span
□Biology influences temperament
□It is a pattern of the child's responses
□It is relatively resistant to parents' attempts to modify
□It helps parents understand the child's behavior without guilt
description Temperament describes the stable, early-appearing individual variations in behavioral dimensions, including
emotionality (crying, laughing, sulking), activity level, attention, sociability, and persistence. Temperament has long been
described as biologic or "inherited," largely based on parent reports of twins. We now know that genes are dynamic,
changing in the quantity and quality of their effects as a child ages and thus, like environment, may continue to change.
Longitudinal twin studies of adult personality indicate that personality changes largely result from non-shared environmental
influences, whereas stability of temperament appears to result from genetic factors. The concept of temperament can help
parents understand and accept the characteristics of their children without feeling responsible for having caused them. (see
e6-1).
73. The parents of a 3 yr old girl report that "she ran before she walked," "she is never hungry at
the same time," and "she goes from toy to toy." This child is best described as:
□Autistic
□Having a specific temperament
□Having attention-deficit/hyperactivity disorder
■Having developmental pervasive disorder
□Being deaf
description The classic theory of Thomas and Chess proposes 9 dimensions of temperament (Table 6-1). These
characteristics lead to 3 common constellations: (1) the easy, highly adaptable child, who has regular biologic cycles; (2) the
difficult child, who withdraws from new stimuli and is easily frustrated; and (3) the slow-to-warm-up child, who needs extra
time to adapt to new circumstances. Various combinations of these clusters also occur. (See e6-1, including Table 6-1.)
74. A term baby girl has 2 episodes of bile-stained emesis at 24 hr after birth. There is a history of
excessive amniotic fluid volume. The most appropriate diagnostic test is:
□Blood culture
■Barium upper gastrointestinal x-ray series with small bowel follow-through
□Barium enema
□CT scan
□Head ultrasound study
description Bile-stained emesis suggests intestinal obstruction beyond the duodenum but also may be idiopathic.
Abdominal radiographs (kidney-ureter-bladder [KUB] and cross-table lateral views) should be performed in neonates with
persistent emesis and in all infants with bile-stained emesis to detect air-fluid levels, distended bowel loops, characteristic
patterns of obstruction (double bubble: duodenal atresia), and pneumoperitoneum (intestinal perforation). A contrast
swallow radiograph with small bowel follow-through is indicated in the presence of bilious emesis. (See Chapter 96, page
600.)
75. Meconium plug is associated with all of the following EXCEPT:
□Hypermagnesemia
□Infants of diabetic mothers
□Cystic fibrosis
□Hirschsprung disease
■Prematurity
description Meconium plugs are associated with small left colon syndrome in infants of diabetic mothers and with cystic
fibrosis, rectal aganglionosis, maternal opiate use, and magnesium sulfate therapy for preeclampsia. (See Chapter 96, page
600.)
76. Which statement about hemolytic disease of the newborn is NOT true?
□RhoGAM administration should be given to the mother within 72 hours of the delivery of an Rh-negative infant; it should
also be given after abortion of amniocentesis
□ABO incompatibility is the most common cause of hemolytic disease of the newborn
■Most cases of ABO incompatibility require transfusion
□Rh and ABO incompatibility account for approximately 95% of cases of hemolytic disease of the newborn
description Most cases of ABO incompatibility are mild, with jaundice being the only clinical manifestation. The infant is
not generally affected at birth; pallor is not present, and hydrops fetalis is extremely rare. The liver and spleen are not
greatly enlarged, if at all. Jaundice usually appears during the 1st 24 hr. Rarely, it may become severe, and symptoms and
signs of kernicterus develop rapidly. Treatment of ABO incompatibility relies on phototherapy to lower serum bilirubin
levels. In severe cases, IVIG administration can reduce the rate of hemolysis and the need for exchange transfusion.
Exchange transfusions with type O blood of the same Rh type as the infant may be needed in some cases to correct
dangerous degrees of anemia or hyperbilirubinemia. (See Chapter 97, page 619.)
77. The best approach to prevent congenital anomalies in infants of diabetic mothers is to:
□Discontinue insulin and begin glyburide
□Provide continuous insulin infusion during labor and delivery
□Switch from an oral hypoglycemic agent to insulin until 36-wk of gestation
□Do serial glucose tolerance tests
■Maintain periconceptional tight control of maternal blood glucose levels
description Periconception glucose control reduces the risk of anomalies and other adverse outcomes, and glucose control
during labor reduces the incidence of neonatal hypoglycemia. Women with type 1 diabetes who have tight glucose control
during pregnancy (average daily glucose levels < 95 mg/dL) are delivered of infants with birthweights and anthropomorphic
features similar to those of infants of nondiabetic mothers. (See Chapter 101, page 629.)
78. Which statement regarding adolescent development is NOT true?
□In girls, the first sign of puberty is breast bud development
□Adolescents experience asymmetric growth beginning with enlargement of the hands and feet, followed by the arms and
legs, and finally, the trunk and chest
□Some degree of breast hypertrophy, typically bilateral, occurs in ~50% of boys during Tanner stages 2-3
■It is abnormal for early adolescents to be preoccupied with their body changes and feel that everyone is staring at them
□Adolescents are able to make more complex decisions, but these decisions are highly susceptible to emotion
description Self-consciousness increases exponentially in response to the somatic transformations of puberty. Self-
awareness at this age centers on external characteristics, in contrast to the introspection of later adolescence. It is normal
for early adolescents to be preoccupied with their body changes, scrutinize their appearance, and feel that everyone else is
staring at them. (See Chapter 104, page 651.)
79. Papanicolaou smears are indicated in:
□Women aged 16 and older
□Women aged 21 and older
■All women beginning 3 years after onset of sexual activity or age 21 years, whichever is earlier
□All sexually active teenage girls
description For sexually active females, the guidelines for Pap smears for cervical cancer screening suggest that annual
screening can be delayed safely up to 3 yr after the onset of sexual activity or age 21 yr, whichever is earlier; HPV DNA
testing should not be used routinely for females 29 yr and younger since the results would not influence management. (See
Chapter 106, page 667.)
80. All of the following may cause gynecomastia EXCEPT:
□Klinefelter syndrome
□Phenothiazines
□Normal puberty
□Heroin
■Albuterol
□Anabolic steroids
description Pubertal gynecomastia, occurring in up to 60% of normal adolescent males, has long been attributed to a
transient imbalance of estrogen and androgen concentrations. With typical onset between chronological ages of 10 and 13
yr, it usually regresses over several months Nonpubertal gynecomastia with hypogonadism is associated with Klinefelter
syndrome and places a patient at a higher risk of breast cancer. Other conditions associated with nonpubertal gynecomastia
are secondary to endocrine disorders, neoplasms, chronic disease, trauma, and medications as well as drugs of abuse. See
Table 109-1, Drugs causing Gynecomastia. (See e109-1.)
81. Common presenting complaints of pregnancy include all of the following EXCEPT:
□Fatigue
□Breast tenderness
□Nausea
□Irregular menses
■Dysuria
description Adolescents may experience the traditional symptoms of pregnancy: morning sickness (vomiting, nausea that
may also occur any time of the day), swollen tender breasts, weight gain, and amenorrhea. Often the presentation is less
classic. Headache, fatigue, abdominal pain, dizziness, and scanty or irregular menses are common presenting complaints.
(See Chapter 112, page 700.)
82. Teenage pregnancy carries increased risk of all of the following EXCEPT:
□Prematurity
□Low birth weight infant
□Gastroschisis
■Multiple birth (twins)
□Maternal eclampsia
description As expected, teen mothers have low rates of age-related chronic disease (diabetes or hypertension) that might
affect the outcomes of a pregnancy. They also have lower rates of twin pregnancies than older women. They tolerate
childbirth well with few operative interventions. However, as compared with 20-39 yr old mothers, teens have higher
incidences of low birthweight infants, preterm infants, neonatal deaths, passage of moderate to heavy fetal meconium during
parturition, and infant deaths within 1 yr after birth. The highest rates of these poor outcomes occur in the youngest and
most economically deprived mothers. Gastroschisis, although very rare, has a markedly higher incidence in infants of teen
mothers for reasons that are not yet clear. Teen mothers also have higher rates of anemia, pregnancy-associated
hypertension, and eclampsia, with the youngest teens having rates of pregnancy-associated hypertension 40% higher than
the rates of women in their 20s and 30s. The youngest teens also have a higher incidence of poor weight gain (<16 lb)
during their pregnancy. This correlates with a decrease in the birthweights of their infants. (See Chapter 112, page 701.)
83. The X-linked lymphoproliferative (XLP) syndrome is classically associated with overwhelming
infection by which of the following agents?
■Epstein-Barr virus
□Enteroviruses
□Catalase-positive bacteria
□Neisseria meningitidis
□Pneumocystis jiroveci
description X-linked lymphoproliferative (XLP) disease is an X-linked recessive trait characterized by an inadequate
immune response to infection with Epstein-Barr virus (EBV). Affected males are usually healthy until they acquire EBV
infection. The mean age of presentation is < 5 yr. There are 3 major clinical phenotypes: (1) fulminant, often fatal, infectious
mononucleosis (50% of cases); (2) lymphomas, predominantly involving B-lineage cells (25%); or (3) acquired
hypogammaglobulinemia (25%). There is a marked impairment in production of antibodies to the EBV nuclear antigen
(EBNA), whereas titers of antibodies to the viral capsid antigen (VCA) have ranged from absent to markedly elevated.
XLP has an unfavorable prognosis; 70% of affected boys die by age 10 yr. (See Chapter 118, page 727.)
84. A 1 yr old child with recurrent sinusitis is found to have a normal WBC count but no
circulating B cells, small tonsils, and no palpable lymph nodes. Serum concentrations of IgG,
IgA, IgM, and IgE are below the 95% lower limits for age. The most likely diagnosis is:
□Ataxia-telangiectasia
□X-linked lymphoproliferative syndrome
□DiGeorge syndrome
□Common variable immunodeficiency
■X-linked agammaglobulinemia (XLA)
description Most boys afflicted with XLA remain well during the 1st 6-9 mo of life by virtue of maternally transmitted IgG
antibodies. Thereafter, they acquire infections with extracellular pyogenic organisms, such as Streptococcus pneumoniae
and Haemophilus influenzae, unless they are given prophylactic antibiotics or immunoglobulin therapy. Infections include
sinusitis, otitis media, pneumonia, or, less often, sepsis or meningitis. Infections with Mycoplasma also are particularly
problematic. The diagnosis of XLA should be suspected if lymphoid hypoplasia is found on physical examination (minimal
or no tonsillar tissue and no palpable lymph nodes), and serum concentrations of IgG, IgA, IgM, and IgE are far below the
95% confidence limits for appropriate age- and race-matched controls, usually with total immunoglobulins < 100 mg/dL.
(See Chapter 118, page 124.)
85. All of the following are prominent features of Wiskott-Aldrich syndrome EXCEPT:
□Atopic dermatitis
□Thrombocytopenia
□Recurrent infections with encapsulated bacteria
■Autosomal dominant inheritance
□More frequent occurrence in males
description Wiskott-Aldrich syndrome, an X-linked recessive syndrome, is characterized by atopic dermatitis,
thrombocytopenic purpura with normal-appearing megakaryocytes but small defective platelets, and undue susceptibility to
infection. Patients often have prolonged bleeding from the circumcision site or bloody diarrhea during infancy. The
thrombocytopenia is not initially due to antiplatelet antibodies. Atopic dermatitis and recurrent infections usually develop
during the 1st yr of life. Streptococcus pneumoniae and other bacteria having polysaccharide capsules cause otitis media,
pneumonia, meningitis, and sepsis. Later, infections with agents such as P. jiroveci and the herpesviruses become more
frequent. Survival beyond the teens is rare; infections, bleeding, and EBV-associated malignancies are major causes of
death. (See Chapter 120, page 734.)
86. All of the following are typically associated with an eosinophilic response and eosinophilia
EXCEPT:
□Allergic rhinitis
□Hypersensitivity drug reactions
□Trichinosis (Trichinella spiralis)
■Pinworms (Enterobius vermicularis)
□Wiskott-Aldrich syndrome
description Many diseases are associated with moderate (1,500-5,000 cells/μL) or severe (>5,000 cells/μL) eosinophilia
(see Table 123-1). Allergy is the most common cause of eosinophilia in children in the USA. Eosinophilia is often
associated with infection with multicellular helminthic parasites, which are the most common cause in developing countries.
Severe eosinophilia in children is most commonly due to visceral larva migrans. The level of eosinophilia tends to parallel the
magnitude and extent of tissue invasion, especially by larvae. Eosinophilia often does not occur in established parasitic
infections that are well contained within tissues or are solely intraluminal in the gastrointestinal tract, such as Giardia lamblia
and Enterobius vermicularis infection. Eosinophilia is observed in many patients with primary immunodeficiency syndromes,
especially hyper-IgE syndrome, Wiskott-Aldrich syndrome, and Omenn syndrome. (See Chapter 123, page 740.)
87. A 2 yr old boy refuses to hold his mother's hand when walking across the parking lot of a
shopping mall. He attempts to run away from her, but she quickly grabs his hand before he
darts out in front of a car. She is exasperated and scared by his actions. Of the following, which
technique is most effective in addressing the behavior?
□Verbal reprimand
□Spanking
□Scolding
■Time-out
□Slapping his hand
description Discipline should be immediate, specific to the behavior, and time limited. Time-out for approximately 1 min/yr
of age is very effective. A kitchen timer allows the parent to step back from the situation; the child is free when the timer
rings. (See Chapter 10, page 36.)
88. To be effective, time-outs should:
□Be accompanied by verbal reprimands
□Be accompanied by punishment
□Last at least 5-10 min
■Last 1 min/yr of age
□Be done at a point in time removed from the incident
description Discipline should be immediate, specific to the behavior, and time limited. Time-out for approximately 1 min/yr
of age is very effective. A kitchen timer allows the parent to step back from the situation; the child is free when the timer
rings. (See Chapter 10 page 36.)
89. Handedness is usually determined by age:
□2-4 mo
□6-12 mo
□15-18 mo
□20-24 mo
■36-48 mo
description Handedness is usually established by the 3rd yr. Frustration may result from attempts to change children's hand
preference. Variations in fine motor development reflect both individual proclivities and different opportunities for learning.
Children who are seldom allowed to use crayons, for example, develop a mature pencil grasp later. (See Chapter 10, page
33.)
90. The best approach for parents to help a preschool child overcome monster fears is to:
□Rationalize that monsters do not exist
□Read books that do not have monsters in them
□Have the pediatrician explain that monsters are make-believe
■Use "great power" like monster spray to keep monsters away
□None of the above
description The active imagination that fuels play and the magical, animist thinking characteristic of preoperational cognition
can also generate intense fears. More than 80% of parents report at least 1 fear in their preschool children. Refusal to take
baths or to sit on the toilet may arise from the fear of being washed or flushed away, reflecting a child's immature
appreciation of relative size. Attempts to demonstrate rationally that there are no monsters in the closet often fail, inasmuch
as the fear arises from prerational thinking. However, this same thinking allows parents to be endowed with magical powers
that can banish the monsters with "monster spray" or a night light. Parents should acknowledge the fears, offer reassurance
and a sense of security, and give the child some sense of control over the situation. (See Chapter 10, page 35.)
91. Between 2 and 5 yr of age, language increases; as a rule, the number of words in a sentence is:
□Based on knowledge of numbers
■Equal to the age of the child in years
□Independent of the environment
□Independent of the number of questions asked the child by adults
□Based on the ABCs
description Language development occurs most rapidly between 2 and 5 yr of age. Vocabulary increases from 50-100
words to more than 2,000. Sentence structure advances from telegraphic phrases ("Baby cry") to sentences incorporating
all of the major grammatical components. As a rule of thumb, between the ages of 2 and 5 yr, the number of words in a
typical sentence equals the child's age (2 by age 2 yr, 3 by age 3 yr, and so on). By 21 mo to 2 yr, most children are using
possessives ("My ball"), progressives (the "-ing" construction, as in "I playing"), questions, and negatives. (See Chapter 10,
page 35.)
92. All of the following statements regarding language development are true EXCEPT:
□Deaf children may create their own language
□The basics for language may be "hard-wired" in the brain
■Environment and experience have minimal impact on the rate of language development
□Delayed language may signify deafness
□Delayed language may signify mental retardation
description It is important to distinguish between speech (the production of intelligible sounds) and language, which refers
to the underlying mental act. Language includes both expressive and receptive functions. Receptive language
(understanding) varies less in its rate of acquisition than does expressive language; therefore, it has greater prognostic
importance. (See Chapters 14 and 32.)
Language acquisition depends critically on environmental input. Key determinants include the amount and variety of speech
directed toward children and the frequency with which adults ask questions and encourage verbalization. Children raised in
poverty typically perform lower on measures of language development compared with children from economically
advantaged families.
Although experience influences the rate of language development, many linguists believe that the basic mechanism for
language learning is "hard-wired" in the brain. Children do not simply imitate adult speech; they abstract the complex rules
of grammar from the ambient language, generating implicit hypotheses. Evidence for the existence of such implicit rules
comes from analysis of grammatical errors, such as the overgeneralized use of "-s" to signify the plural and "-ed" to signify
the past ("We seed lots of mouses.").
Language is linked to both cognitive and emotional development. Language delays may be the first indication that a child
has mental retardation, has an autism spectrum disorder, or has been maltreated. Language plays a critical part in the
regulation of behavior through internalized "private speech" in which a child repeats adult prohibitions, first audibly and then
mentally. Language also allows children to express feelings, such as anger or frustration, without acting them out;
consequently, language-delayed children show higher rates of tantrums and other externalizing behaviors. (See Chapter 10,
page 35.)
93. Repeated meningococcal infections suggest which of the following types of immune disorder?
□B-cell defect
□T-cell defect
□Combined B- and T-cell defect
□Phagocyte function defect
■Complement component deficiency
description A defect of complement function should be considered in any patient with recurrent pyogenic infections,
disseminated meningococcal or gonococcal infection, or a second episode of bacteremia at any age. A previously well
adolescent or young adult with meningococcal meningitis due to an uncommon serotype (not A, B, or C) should undergo
screening for a late-component or alternative pathway deficiency with CH50 and AP50 assays. (See Chapter 128, page
753.)
94. A complement defect should be suspected in which disorder?
□Recurrent angioedema
□Chronic nephritis
□Recurrent hemolytic-uremic syndrome
□A and B
■All of the above
description A defect of complement function should be considered in any patient with recurrent angioedema, autoimmune
disease, chronic nephritis, hemolytic-uremic syndrome (HUS), or partial lipodystrophy, or with recurrent pyogenic
infections, disseminated meningococcal or gonococcal infection, or a second episode of bacteremia at any age. A
previously well adolescent or young adult with meningococcal meningitis due to an uncommon serotype (not A, B, or C)
should undergo screening for a late-component or alternative pathway deficiency with CH50 and AP50 assays. (See
Chapter 128, page 753.)
95. Signs of allergy on physical examination include all of the following EXCEPT:
□Dennie lines (Dennie-Morgan folds)
□Purple discolorations beneath the lower eyelids
■Erythematous nasal mucosa with thick yellow secretions
□Xerosis
□Keratoconus
description Parents of allergic children are often concerned about blue-gray to purple discolorations beneath the lower
eyelids, attributed to venous stasis and referred to as allergic shiners. They are found in up to 60% of allergic patients and
almost 40% of patients without allergic disease. They are often accompanied by Dennie lines (Dennie-Morgan folds), which
are prominent symmetric skin folds that extend in an arc from the inner canthus beneath and parallel to the lower lid margin.
The nasal mucosa in allergic rhinitis is classically described as pale to purple in comparison with the beefy red mucosa of
patients with nonallergic rhinitis. Allergic nasal secretions are typically thin and clear. Purulent secretions suggest another
cause of rhinitis. Keratoconus, or protrusion of the cornea, may occur in patients with atopic dermatitis as a result of
repeated trauma produced by persistent rubbing of the eyes. (See Chapter 135, page 765.)
96. A 7 yr old boy with asthma has roughness over the extensor surfaces of the upper arms and
thighs, which is caused by keratin plugs lodged in the openings of hair follicles. This physical
finding is termed:
■Keratosis pilaris
□Fibroepitheliosis
□Hidradenitis
□Xerosis
□Acrochordon
description Keratosis pilaris, often found on the extensor surfaces of the upper arms and thighs, is characterized by
roughness of the skin caused by keratin plugs lodged in the openings of hair follicles. Examination of the skin of the palms
and feet reveals exaggerated palmar and plantar creases in some allergic children. (See Chapter 135, page 765.)
Nelson pediatrics review (mcqs) 19ed
Nelson pediatrics review (mcqs) 19ed
Nelson pediatrics review (mcqs) 19ed
Nelson pediatrics review (mcqs) 19ed
Nelson pediatrics review (mcqs) 19ed
Nelson pediatrics review (mcqs) 19ed
Nelson pediatrics review (mcqs) 19ed
Nelson pediatrics review (mcqs) 19ed
Nelson pediatrics review (mcqs) 19ed
Nelson pediatrics review (mcqs) 19ed
Nelson pediatrics review (mcqs) 19ed
Nelson pediatrics review (mcqs) 19ed
Nelson pediatrics review (mcqs) 19ed
Nelson pediatrics review (mcqs) 19ed
Nelson pediatrics review (mcqs) 19ed
Nelson pediatrics review (mcqs) 19ed
Nelson pediatrics review (mcqs) 19ed
Nelson pediatrics review (mcqs) 19ed
Nelson pediatrics review (mcqs) 19ed
Nelson pediatrics review (mcqs) 19ed
Nelson pediatrics review (mcqs) 19ed
Nelson pediatrics review (mcqs) 19ed
Nelson pediatrics review (mcqs) 19ed
Nelson pediatrics review (mcqs) 19ed
Nelson pediatrics review (mcqs) 19ed
Nelson pediatrics review (mcqs) 19ed
Nelson pediatrics review (mcqs) 19ed
Nelson pediatrics review (mcqs) 19ed
Nelson pediatrics review (mcqs) 19ed
Nelson pediatrics review (mcqs) 19ed
Nelson pediatrics review (mcqs) 19ed
Nelson pediatrics review (mcqs) 19ed
Nelson pediatrics review (mcqs) 19ed
Nelson pediatrics review (mcqs) 19ed
Nelson pediatrics review (mcqs) 19ed
Nelson pediatrics review (mcqs) 19ed
Nelson pediatrics review (mcqs) 19ed
Nelson pediatrics review (mcqs) 19ed
Nelson pediatrics review (mcqs) 19ed
Nelson pediatrics review (mcqs) 19ed
Nelson pediatrics review (mcqs) 19ed
Nelson pediatrics review (mcqs) 19ed
Nelson pediatrics review (mcqs) 19ed
Nelson pediatrics review (mcqs) 19ed
Nelson pediatrics review (mcqs) 19ed
Nelson pediatrics review (mcqs) 19ed
Nelson pediatrics review (mcqs) 19ed
Nelson pediatrics review (mcqs) 19ed
Nelson pediatrics review (mcqs) 19ed
Nelson pediatrics review (mcqs) 19ed
Nelson pediatrics review (mcqs) 19ed
Nelson pediatrics review (mcqs) 19ed
Nelson pediatrics review (mcqs) 19ed
Nelson pediatrics review (mcqs) 19ed
Nelson pediatrics review (mcqs) 19ed
Nelson pediatrics review (mcqs) 19ed
Nelson pediatrics review (mcqs) 19ed
Nelson pediatrics review (mcqs) 19ed
Nelson pediatrics review (mcqs) 19ed
Nelson pediatrics review (mcqs) 19ed
Nelson pediatrics review (mcqs) 19ed
Nelson pediatrics review (mcqs) 19ed
Nelson pediatrics review (mcqs) 19ed
Nelson pediatrics review (mcqs) 19ed
Nelson pediatrics review (mcqs) 19ed
Nelson pediatrics review (mcqs) 19ed
Nelson pediatrics review (mcqs) 19ed
Nelson pediatrics review (mcqs) 19ed
Nelson pediatrics review (mcqs) 19ed
Nelson pediatrics review (mcqs) 19ed
Nelson pediatrics review (mcqs) 19ed
Nelson pediatrics review (mcqs) 19ed
Nelson pediatrics review (mcqs) 19ed
Nelson pediatrics review (mcqs) 19ed
Nelson pediatrics review (mcqs) 19ed
Nelson pediatrics review (mcqs) 19ed
Nelson pediatrics review (mcqs) 19ed
Nelson pediatrics review (mcqs) 19ed
Nelson pediatrics review (mcqs) 19ed
Nelson pediatrics review (mcqs) 19ed
Nelson pediatrics review (mcqs) 19ed
Nelson pediatrics review (mcqs) 19ed
Nelson pediatrics review (mcqs) 19ed
Nelson pediatrics review (mcqs) 19ed
Nelson pediatrics review (mcqs) 19ed
Nelson pediatrics review (mcqs) 19ed
Nelson pediatrics review (mcqs) 19ed
Nelson pediatrics review (mcqs) 19ed
Nelson pediatrics review (mcqs) 19ed
Nelson pediatrics review (mcqs) 19ed
Nelson pediatrics review (mcqs) 19ed
Nelson pediatrics review (mcqs) 19ed
Nelson pediatrics review (mcqs) 19ed
Nelson pediatrics review (mcqs) 19ed
Nelson pediatrics review (mcqs) 19ed
Nelson pediatrics review (mcqs) 19ed
Nelson pediatrics review (mcqs) 19ed
Nelson pediatrics review (mcqs) 19ed
Nelson pediatrics review (mcqs) 19ed
Nelson pediatrics review (mcqs) 19ed
Nelson pediatrics review (mcqs) 19ed
Nelson pediatrics review (mcqs) 19ed
Nelson pediatrics review (mcqs) 19ed
Nelson pediatrics review (mcqs) 19ed
Nelson pediatrics review (mcqs) 19ed
Nelson pediatrics review (mcqs) 19ed
Nelson pediatrics review (mcqs) 19ed
Nelson pediatrics review (mcqs) 19ed
Nelson pediatrics review (mcqs) 19ed
Nelson pediatrics review (mcqs) 19ed
Nelson pediatrics review (mcqs) 19ed
Nelson pediatrics review (mcqs) 19ed
Nelson pediatrics review (mcqs) 19ed
Nelson pediatrics review (mcqs) 19ed
Nelson pediatrics review (mcqs) 19ed
Nelson pediatrics review (mcqs) 19ed
Nelson pediatrics review (mcqs) 19ed
Nelson pediatrics review (mcqs) 19ed
Nelson pediatrics review (mcqs) 19ed
Nelson pediatrics review (mcqs) 19ed
Nelson pediatrics review (mcqs) 19ed
Nelson pediatrics review (mcqs) 19ed
Nelson pediatrics review (mcqs) 19ed
Nelson pediatrics review (mcqs) 19ed
Nelson pediatrics review (mcqs) 19ed
Nelson pediatrics review (mcqs) 19ed
Nelson pediatrics review (mcqs) 19ed
Nelson pediatrics review (mcqs) 19ed
Nelson pediatrics review (mcqs) 19ed
Nelson pediatrics review (mcqs) 19ed
Nelson pediatrics review (mcqs) 19ed
Nelson pediatrics review (mcqs) 19ed
Nelson pediatrics review (mcqs) 19ed
Nelson pediatrics review (mcqs) 19ed
Nelson pediatrics review (mcqs) 19ed
Nelson pediatrics review (mcqs) 19ed
Nelson pediatrics review (mcqs) 19ed
Nelson pediatrics review (mcqs) 19ed
Nelson pediatrics review (mcqs) 19ed
Nelson pediatrics review (mcqs) 19ed
Nelson pediatrics review (mcqs) 19ed
Nelson pediatrics review (mcqs) 19ed
Nelson pediatrics review (mcqs) 19ed
Nelson pediatrics review (mcqs) 19ed
Nelson pediatrics review (mcqs) 19ed
Nelson pediatrics review (mcqs) 19ed
Nelson pediatrics review (mcqs) 19ed
Nelson pediatrics review (mcqs) 19ed
Nelson pediatrics review (mcqs) 19ed
Nelson pediatrics review (mcqs) 19ed
Nelson pediatrics review (mcqs) 19ed
Nelson pediatrics review (mcqs) 19ed
Nelson pediatrics review (mcqs) 19ed
Nelson pediatrics review (mcqs) 19ed
Nelson pediatrics review (mcqs) 19ed
Nelson pediatrics review (mcqs) 19ed
Nelson pediatrics review (mcqs) 19ed
Nelson pediatrics review (mcqs) 19ed
Nelson pediatrics review (mcqs) 19ed
Nelson pediatrics review (mcqs) 19ed
Nelson pediatrics review (mcqs) 19ed
Nelson pediatrics review (mcqs) 19ed
Nelson pediatrics review (mcqs) 19ed
Nelson pediatrics review (mcqs) 19ed
Nelson pediatrics review (mcqs) 19ed
Nelson pediatrics review (mcqs) 19ed
Nelson pediatrics review (mcqs) 19ed
Nelson pediatrics review (mcqs) 19ed
Nelson pediatrics review (mcqs) 19ed
Nelson pediatrics review (mcqs) 19ed
Nelson pediatrics review (mcqs) 19ed
Nelson pediatrics review (mcqs) 19ed
Nelson pediatrics review (mcqs) 19ed
Nelson pediatrics review (mcqs) 19ed
Nelson pediatrics review (mcqs) 19ed
Nelson pediatrics review (mcqs) 19ed
Nelson pediatrics review (mcqs) 19ed
Nelson pediatrics review (mcqs) 19ed
Nelson pediatrics review (mcqs) 19ed
Nelson pediatrics review (mcqs) 19ed
Nelson pediatrics review (mcqs) 19ed
Nelson pediatrics review (mcqs) 19ed
Nelson pediatrics review (mcqs) 19ed
Nelson pediatrics review (mcqs) 19ed
Nelson pediatrics review (mcqs) 19ed
Nelson pediatrics review (mcqs) 19ed
Nelson pediatrics review (mcqs) 19ed
Nelson pediatrics review (mcqs) 19ed
Nelson pediatrics review (mcqs) 19ed
Nelson pediatrics review (mcqs) 19ed
Nelson pediatrics review (mcqs) 19ed
Nelson pediatrics review (mcqs) 19ed
Nelson pediatrics review (mcqs) 19ed
Nelson pediatrics review (mcqs) 19ed
Nelson pediatrics review (mcqs) 19ed
Nelson pediatrics review (mcqs) 19ed
Nelson pediatrics review (mcqs) 19ed
Nelson pediatrics review (mcqs) 19ed
Nelson pediatrics review (mcqs) 19ed
Nelson pediatrics review (mcqs) 19ed
Nelson pediatrics review (mcqs) 19ed
Nelson pediatrics review (mcqs) 19ed
Nelson pediatrics review (mcqs) 19ed
Nelson pediatrics review (mcqs) 19ed
Nelson pediatrics review (mcqs) 19ed
Nelson pediatrics review (mcqs) 19ed
Nelson pediatrics review (mcqs) 19ed
Nelson pediatrics review (mcqs) 19ed
Nelson pediatrics review (mcqs) 19ed
Nelson pediatrics review (mcqs) 19ed
Nelson pediatrics review (mcqs) 19ed
Nelson pediatrics review (mcqs) 19ed
Nelson pediatrics review (mcqs) 19ed
Nelson pediatrics review (mcqs) 19ed
Nelson pediatrics review (mcqs) 19ed
Nelson pediatrics review (mcqs) 19ed
Nelson pediatrics review (mcqs) 19ed
Nelson pediatrics review (mcqs) 19ed
Nelson pediatrics review (mcqs) 19ed
Nelson pediatrics review (mcqs) 19ed
Nelson pediatrics review (mcqs) 19ed
Nelson pediatrics review (mcqs) 19ed
Nelson pediatrics review (mcqs) 19ed
Nelson pediatrics review (mcqs) 19ed
Nelson pediatrics review (mcqs) 19ed
Nelson pediatrics review (mcqs) 19ed
Nelson pediatrics review (mcqs) 19ed
Nelson pediatrics review (mcqs) 19ed
Nelson pediatrics review (mcqs) 19ed
Nelson pediatrics review (mcqs) 19ed
Nelson pediatrics review (mcqs) 19ed
Nelson pediatrics review (mcqs) 19ed
Nelson pediatrics review (mcqs) 19ed
Nelson pediatrics review (mcqs) 19ed
Nelson pediatrics review (mcqs) 19ed
Nelson pediatrics review (mcqs) 19ed
Nelson pediatrics review (mcqs) 19ed
Nelson pediatrics review (mcqs) 19ed
Nelson pediatrics review (mcqs) 19ed
Nelson pediatrics review (mcqs) 19ed
Nelson pediatrics review (mcqs) 19ed
Nelson pediatrics review (mcqs) 19ed
Nelson pediatrics review (mcqs) 19ed
Nelson pediatrics review (mcqs) 19ed
Nelson pediatrics review (mcqs) 19ed
Nelson pediatrics review (mcqs) 19ed
Nelson pediatrics review (mcqs) 19ed
Nelson pediatrics review (mcqs) 19ed
Nelson pediatrics review (mcqs) 19ed
Nelson pediatrics review (mcqs) 19ed
Nelson pediatrics review (mcqs) 19ed
Nelson pediatrics review (mcqs) 19ed
Nelson pediatrics review (mcqs) 19ed
Nelson pediatrics review (mcqs) 19ed
Nelson pediatrics review (mcqs) 19ed
Nelson pediatrics review (mcqs) 19ed
Nelson pediatrics review (mcqs) 19ed
Nelson pediatrics review (mcqs) 19ed
Nelson pediatrics review (mcqs) 19ed
Nelson pediatrics review (mcqs) 19ed
Nelson pediatrics review (mcqs) 19ed
Nelson pediatrics review (mcqs) 19ed
Nelson pediatrics review (mcqs) 19ed
Nelson pediatrics review (mcqs) 19ed
Nelson pediatrics review (mcqs) 19ed
Nelson pediatrics review (mcqs) 19ed
Nelson pediatrics review (mcqs) 19ed
Nelson pediatrics review (mcqs) 19ed
Nelson pediatrics review (mcqs) 19ed
Nelson pediatrics review (mcqs) 19ed
Nelson pediatrics review (mcqs) 19ed
Nelson pediatrics review (mcqs) 19ed
Nelson pediatrics review (mcqs) 19ed
Nelson pediatrics review (mcqs) 19ed
Nelson pediatrics review (mcqs) 19ed
Nelson pediatrics review (mcqs) 19ed
Nelson pediatrics review (mcqs) 19ed
Nelson pediatrics review (mcqs) 19ed
Nelson pediatrics review (mcqs) 19ed
Nelson pediatrics review (mcqs) 19ed
Nelson pediatrics review (mcqs) 19ed
Nelson pediatrics review (mcqs) 19ed
Nelson pediatrics review (mcqs) 19ed
Nelson pediatrics review (mcqs) 19ed
Nelson pediatrics review (mcqs) 19ed
Nelson pediatrics review (mcqs) 19ed
Nelson pediatrics review (mcqs) 19ed
Nelson pediatrics review (mcqs) 19ed
Nelson pediatrics review (mcqs) 19ed
Nelson pediatrics review (mcqs) 19ed
Nelson pediatrics review (mcqs) 19ed
Nelson pediatrics review (mcqs) 19ed
Nelson pediatrics review (mcqs) 19ed
Nelson pediatrics review (mcqs) 19ed
Nelson pediatrics review (mcqs) 19ed
Nelson pediatrics review (mcqs) 19ed
Nelson pediatrics review (mcqs) 19ed
Nelson pediatrics review (mcqs) 19ed
Nelson pediatrics review (mcqs) 19ed
Nelson pediatrics review (mcqs) 19ed
Nelson pediatrics review (mcqs) 19ed
Nelson pediatrics review (mcqs) 19ed
Nelson pediatrics review (mcqs) 19ed
Nelson pediatrics review (mcqs) 19ed
Nelson pediatrics review (mcqs) 19ed
Nelson pediatrics review (mcqs) 19ed
Nelson pediatrics review (mcqs) 19ed
Nelson pediatrics review (mcqs) 19ed
Nelson pediatrics review (mcqs) 19ed
Nelson pediatrics review (mcqs) 19ed
Nelson pediatrics review (mcqs) 19ed
Nelson pediatrics review (mcqs) 19ed
Nelson pediatrics review (mcqs) 19ed
Nelson pediatrics review (mcqs) 19ed
Nelson pediatrics review (mcqs) 19ed
Nelson pediatrics review (mcqs) 19ed
Nelson pediatrics review (mcqs) 19ed
Nelson pediatrics review (mcqs) 19ed
Nelson pediatrics review (mcqs) 19ed
Nelson pediatrics review (mcqs) 19ed
Nelson pediatrics review (mcqs) 19ed
Nelson pediatrics review (mcqs) 19ed
Nelson pediatrics review (mcqs) 19ed
Nelson pediatrics review (mcqs) 19ed
Nelson pediatrics review (mcqs) 19ed
Nelson pediatrics review (mcqs) 19ed
Nelson pediatrics review (mcqs) 19ed
Nelson pediatrics review (mcqs) 19ed
Nelson pediatrics review (mcqs) 19ed
Nelson pediatrics review (mcqs) 19ed
Nelson pediatrics review (mcqs) 19ed
Nelson pediatrics review (mcqs) 19ed
Nelson pediatrics review (mcqs) 19ed
Nelson pediatrics review (mcqs) 19ed
Nelson pediatrics review (mcqs) 19ed
Nelson pediatrics review (mcqs) 19ed
Nelson pediatrics review (mcqs) 19ed
Nelson pediatrics review (mcqs) 19ed
Nelson pediatrics review (mcqs) 19ed
Nelson pediatrics review (mcqs) 19ed
Nelson pediatrics review (mcqs) 19ed
Nelson pediatrics review (mcqs) 19ed
Nelson pediatrics review (mcqs) 19ed
Nelson pediatrics review (mcqs) 19ed
Nelson pediatrics review (mcqs) 19ed
Nelson pediatrics review (mcqs) 19ed
Nelson pediatrics review (mcqs) 19ed
Nelson pediatrics review (mcqs) 19ed
Nelson pediatrics review (mcqs) 19ed
Nelson pediatrics review (mcqs) 19ed
Nelson pediatrics review (mcqs) 19ed
Nelson pediatrics review (mcqs) 19ed
Nelson pediatrics review (mcqs) 19ed
Nelson pediatrics review (mcqs) 19ed
Nelson pediatrics review (mcqs) 19ed
Nelson pediatrics review (mcqs) 19ed
Nelson pediatrics review (mcqs) 19ed
Nelson pediatrics review (mcqs) 19ed
Nelson pediatrics review (mcqs) 19ed
Nelson pediatrics review (mcqs) 19ed
Nelson pediatrics review (mcqs) 19ed
Nelson pediatrics review (mcqs) 19ed
Nelson pediatrics review (mcqs) 19ed
Nelson pediatrics review (mcqs) 19ed
Nelson pediatrics review (mcqs) 19ed
Nelson pediatrics review (mcqs) 19ed
Nelson pediatrics review (mcqs) 19ed
Nelson pediatrics review (mcqs) 19ed
Nelson pediatrics review (mcqs) 19ed
Nelson pediatrics review (mcqs) 19ed
Nelson pediatrics review (mcqs) 19ed
Nelson pediatrics review (mcqs) 19ed
Nelson pediatrics review (mcqs) 19ed
Nelson pediatrics review (mcqs) 19ed
Nelson pediatrics review (mcqs) 19ed
Nelson pediatrics review (mcqs) 19ed
Nelson pediatrics review (mcqs) 19ed
Nelson pediatrics review (mcqs) 19ed
Nelson pediatrics review (mcqs) 19ed
Nelson pediatrics review (mcqs) 19ed
Nelson pediatrics review (mcqs) 19ed
Nelson pediatrics review (mcqs) 19ed
Nelson pediatrics review (mcqs) 19ed
Nelson pediatrics review (mcqs) 19ed
Nelson pediatrics review (mcqs) 19ed
Nelson pediatrics review (mcqs) 19ed
Nelson pediatrics review (mcqs) 19ed
Nelson pediatrics review (mcqs) 19ed
Nelson pediatrics review (mcqs) 19ed
Nelson pediatrics review (mcqs) 19ed
Nelson pediatrics review (mcqs) 19ed
Nelson pediatrics review (mcqs) 19ed
Nelson pediatrics review (mcqs) 19ed
Nelson pediatrics review (mcqs) 19ed
Nelson pediatrics review (mcqs) 19ed
Nelson pediatrics review (mcqs) 19ed
Nelson pediatrics review (mcqs) 19ed
Nelson pediatrics review (mcqs) 19ed
Nelson pediatrics review (mcqs) 19ed
Nelson pediatrics review (mcqs) 19ed
Nelson pediatrics review (mcqs) 19ed
Nelson pediatrics review (mcqs) 19ed
Nelson pediatrics review (mcqs) 19ed
Nelson pediatrics review (mcqs) 19ed
Nelson pediatrics review (mcqs) 19ed
Nelson pediatrics review (mcqs) 19ed
Nelson pediatrics review (mcqs) 19ed
Nelson pediatrics review (mcqs) 19ed
Nelson pediatrics review (mcqs) 19ed
Nelson pediatrics review (mcqs) 19ed
Nelson pediatrics review (mcqs) 19ed
Nelson pediatrics review (mcqs) 19ed
Nelson pediatrics review (mcqs) 19ed
Nelson pediatrics review (mcqs) 19ed
Nelson pediatrics review (mcqs) 19ed
Nelson pediatrics review (mcqs) 19ed
Nelson pediatrics review (mcqs) 19ed
Nelson pediatrics review (mcqs) 19ed
Nelson pediatrics review (mcqs) 19ed
Nelson pediatrics review (mcqs) 19ed
Nelson pediatrics review (mcqs) 19ed
Nelson pediatrics review (mcqs) 19ed
Nelson pediatrics review (mcqs) 19ed
Nelson pediatrics review (mcqs) 19ed
Nelson pediatrics review (mcqs) 19ed
Nelson pediatrics review (mcqs) 19ed
Nelson pediatrics review (mcqs) 19ed
Nelson pediatrics review (mcqs) 19ed
Nelson pediatrics review (mcqs) 19ed
Nelson pediatrics review (mcqs) 19ed
Nelson pediatrics review (mcqs) 19ed
Nelson pediatrics review (mcqs) 19ed
Nelson pediatrics review (mcqs) 19ed
Nelson pediatrics review (mcqs) 19ed
Nelson pediatrics review (mcqs) 19ed
Nelson pediatrics review (mcqs) 19ed
Nelson pediatrics review (mcqs) 19ed
Nelson pediatrics review (mcqs) 19ed
Nelson pediatrics review (mcqs) 19ed
Nelson pediatrics review (mcqs) 19ed
Nelson pediatrics review (mcqs) 19ed
Nelson pediatrics review (mcqs) 19ed
Nelson pediatrics review (mcqs) 19ed
Nelson pediatrics review (mcqs) 19ed
Nelson pediatrics review (mcqs) 19ed
Nelson pediatrics review (mcqs) 19ed
Nelson pediatrics review (mcqs) 19ed
Nelson pediatrics review (mcqs) 19ed
Nelson pediatrics review (mcqs) 19ed
Nelson pediatrics review (mcqs) 19ed
Nelson pediatrics review (mcqs) 19ed
Nelson pediatrics review (mcqs) 19ed
Nelson pediatrics review (mcqs) 19ed
Nelson pediatrics review (mcqs) 19ed
Nelson pediatrics review (mcqs) 19ed
Nelson pediatrics review (mcqs) 19ed
Nelson pediatrics review (mcqs) 19ed
Nelson pediatrics review (mcqs) 19ed
Nelson pediatrics review (mcqs) 19ed
Nelson pediatrics review (mcqs) 19ed
Nelson pediatrics review (mcqs) 19ed
Nelson pediatrics review (mcqs) 19ed
Nelson pediatrics review (mcqs) 19ed
Nelson pediatrics review (mcqs) 19ed
Nelson pediatrics review (mcqs) 19ed
Nelson pediatrics review (mcqs) 19ed
Nelson pediatrics review (mcqs) 19ed
Nelson pediatrics review (mcqs) 19ed
Nelson pediatrics review (mcqs) 19ed
Nelson pediatrics review (mcqs) 19ed
Nelson pediatrics review (mcqs) 19ed
Nelson pediatrics review (mcqs) 19ed
Nelson pediatrics review (mcqs) 19ed
Nelson pediatrics review (mcqs) 19ed
Nelson pediatrics review (mcqs) 19ed
Nelson pediatrics review (mcqs) 19ed
Nelson pediatrics review (mcqs) 19ed
Nelson pediatrics review (mcqs) 19ed
Nelson pediatrics review (mcqs) 19ed
Nelson pediatrics review (mcqs) 19ed
Nelson pediatrics review (mcqs) 19ed
Nelson pediatrics review (mcqs) 19ed
Nelson pediatrics review (mcqs) 19ed
Nelson pediatrics review (mcqs) 19ed
Nelson pediatrics review (mcqs) 19ed
Nelson pediatrics review (mcqs) 19ed
Nelson pediatrics review (mcqs) 19ed
Nelson pediatrics review (mcqs) 19ed
Nelson pediatrics review (mcqs) 19ed
Nelson pediatrics review (mcqs) 19ed
Nelson pediatrics review (mcqs) 19ed
Nelson pediatrics review (mcqs) 19ed
Nelson pediatrics review (mcqs) 19ed
Nelson pediatrics review (mcqs) 19ed
Nelson pediatrics review (mcqs) 19ed
Nelson pediatrics review (mcqs) 19ed
Nelson pediatrics review (mcqs) 19ed
Nelson pediatrics review (mcqs) 19ed
Nelson pediatrics review (mcqs) 19ed
Nelson pediatrics review (mcqs) 19ed
Nelson pediatrics review (mcqs) 19ed
Nelson pediatrics review (mcqs) 19ed
Nelson pediatrics review (mcqs) 19ed
Nelson pediatrics review (mcqs) 19ed
Nelson pediatrics review (mcqs) 19ed
Nelson pediatrics review (mcqs) 19ed
Nelson pediatrics review (mcqs) 19ed
Nelson pediatrics review (mcqs) 19ed
Nelson pediatrics review (mcqs) 19ed
Nelson pediatrics review (mcqs) 19ed
Nelson pediatrics review (mcqs) 19ed
Nelson pediatrics review (mcqs) 19ed
Nelson pediatrics review (mcqs) 19ed
Nelson pediatrics review (mcqs) 19ed
Nelson pediatrics review (mcqs) 19ed
Nelson pediatrics review (mcqs) 19ed
Nelson pediatrics review (mcqs) 19ed
Nelson pediatrics review (mcqs) 19ed
Nelson pediatrics review (mcqs) 19ed
Nelson pediatrics review (mcqs) 19ed
Nelson pediatrics review (mcqs) 19ed
Nelson pediatrics review (mcqs) 19ed
Nelson pediatrics review (mcqs) 19ed
Nelson pediatrics review (mcqs) 19ed
Nelson pediatrics review (mcqs) 19ed
Nelson pediatrics review (mcqs) 19ed
Nelson pediatrics review (mcqs) 19ed
Nelson pediatrics review (mcqs) 19ed
Nelson pediatrics review (mcqs) 19ed
Nelson pediatrics review (mcqs) 19ed
Nelson pediatrics review (mcqs) 19ed
Nelson pediatrics review (mcqs) 19ed
Nelson pediatrics review (mcqs) 19ed
Nelson pediatrics review (mcqs) 19ed
Nelson pediatrics review (mcqs) 19ed
Nelson pediatrics review (mcqs) 19ed
Nelson pediatrics review (mcqs) 19ed
Nelson pediatrics review (mcqs) 19ed
Nelson pediatrics review (mcqs) 19ed
Nelson pediatrics review (mcqs) 19ed
Nelson pediatrics review (mcqs) 19ed
Nelson pediatrics review (mcqs) 19ed
Nelson pediatrics review (mcqs) 19ed
Nelson pediatrics review (mcqs) 19ed
Nelson pediatrics review (mcqs) 19ed
Nelson pediatrics review (mcqs) 19ed
Nelson pediatrics review (mcqs) 19ed
Nelson pediatrics review (mcqs) 19ed
Nelson pediatrics review (mcqs) 19ed
Nelson pediatrics review (mcqs) 19ed
Nelson pediatrics review (mcqs) 19ed
Nelson pediatrics review (mcqs) 19ed
Nelson pediatrics review (mcqs) 19ed
Nelson pediatrics review (mcqs) 19ed
Nelson pediatrics review (mcqs) 19ed
Nelson pediatrics review (mcqs) 19ed
Nelson pediatrics review (mcqs) 19ed
Nelson pediatrics review (mcqs) 19ed
Nelson pediatrics review (mcqs) 19ed
Nelson pediatrics review (mcqs) 19ed
Nelson pediatrics review (mcqs) 19ed
Nelson pediatrics review (mcqs) 19ed
Nelson pediatrics review (mcqs) 19ed
Nelson pediatrics review (mcqs) 19ed
Nelson pediatrics review (mcqs) 19ed
Nelson pediatrics review (mcqs) 19ed
Nelson pediatrics review (mcqs) 19ed
Nelson pediatrics review (mcqs) 19ed
Nelson pediatrics review (mcqs) 19ed
Nelson pediatrics review (mcqs) 19ed
Nelson pediatrics review (mcqs) 19ed
Nelson pediatrics review (mcqs) 19ed
Nelson pediatrics review (mcqs) 19ed
Nelson pediatrics review (mcqs) 19ed
Nelson pediatrics review (mcqs) 19ed
Nelson pediatrics review (mcqs) 19ed
Nelson pediatrics review (mcqs) 19ed
Nelson pediatrics review (mcqs) 19ed
Nelson pediatrics review (mcqs) 19ed
Nelson pediatrics review (mcqs) 19ed
Nelson pediatrics review (mcqs) 19ed
Nelson pediatrics review (mcqs) 19ed
Nelson pediatrics review (mcqs) 19ed
Nelson pediatrics review (mcqs) 19ed
Nelson pediatrics review (mcqs) 19ed
Nelson pediatrics review (mcqs) 19ed
Nelson pediatrics review (mcqs) 19ed
Nelson pediatrics review (mcqs) 19ed
Nelson pediatrics review (mcqs) 19ed
Nelson pediatrics review (mcqs) 19ed
Nelson pediatrics review (mcqs) 19ed
Nelson pediatrics review (mcqs) 19ed
Nelson pediatrics review (mcqs) 19ed
Nelson pediatrics review (mcqs) 19ed
Nelson pediatrics review (mcqs) 19ed
Nelson pediatrics review (mcqs) 19ed
Nelson pediatrics review (mcqs) 19ed
Nelson pediatrics review (mcqs) 19ed
Nelson pediatrics review (mcqs) 19ed
Nelson pediatrics review (mcqs) 19ed
Nelson pediatrics review (mcqs) 19ed
Nelson pediatrics review (mcqs) 19ed
Nelson pediatrics review (mcqs) 19ed
Nelson pediatrics review (mcqs) 19ed
Nelson pediatrics review (mcqs) 19ed
Nelson pediatrics review (mcqs) 19ed
Nelson pediatrics review (mcqs) 19ed
Nelson pediatrics review (mcqs) 19ed
Nelson pediatrics review (mcqs) 19ed
Nelson pediatrics review (mcqs) 19ed
Nelson pediatrics review (mcqs) 19ed
Nelson pediatrics review (mcqs) 19ed
Nelson pediatrics review (mcqs) 19ed
Nelson pediatrics review (mcqs) 19ed
Nelson pediatrics review (mcqs) 19ed
Nelson pediatrics review (mcqs) 19ed
Nelson pediatrics review (mcqs) 19ed
Nelson pediatrics review (mcqs) 19ed
Nelson pediatrics review (mcqs) 19ed
Nelson pediatrics review (mcqs) 19ed
Nelson pediatrics review (mcqs) 19ed
Nelson pediatrics review (mcqs) 19ed
Nelson pediatrics review (mcqs) 19ed
Nelson pediatrics review (mcqs) 19ed
Nelson pediatrics review (mcqs) 19ed
Nelson pediatrics review (mcqs) 19ed
Nelson pediatrics review (mcqs) 19ed
Nelson pediatrics review (mcqs) 19ed
Nelson pediatrics review (mcqs) 19ed
Nelson pediatrics review (mcqs) 19ed
Nelson pediatrics review (mcqs) 19ed
Nelson pediatrics review (mcqs) 19ed
Nelson pediatrics review (mcqs) 19ed
Nelson pediatrics review (mcqs) 19ed
Nelson pediatrics review (mcqs) 19ed
Nelson pediatrics review (mcqs) 19ed
Nelson pediatrics review (mcqs) 19ed
Nelson pediatrics review (mcqs) 19ed
Nelson pediatrics review (mcqs) 19ed
Nelson pediatrics review (mcqs) 19ed
Nelson pediatrics review (mcqs) 19ed
Nelson pediatrics review (mcqs) 19ed
Nelson pediatrics review (mcqs) 19ed
Nelson pediatrics review (mcqs) 19ed
Nelson pediatrics review (mcqs) 19ed
Nelson pediatrics review (mcqs) 19ed
Nelson pediatrics review (mcqs) 19ed
Nelson pediatrics review (mcqs) 19ed
Nelson pediatrics review (mcqs) 19ed
Nelson pediatrics review (mcqs) 19ed
Nelson pediatrics review (mcqs) 19ed
Nelson pediatrics review (mcqs) 19ed
Nelson pediatrics review (mcqs) 19ed
Nelson pediatrics review (mcqs) 19ed
Nelson pediatrics review (mcqs) 19ed
Nelson pediatrics review (mcqs) 19ed
Nelson pediatrics review (mcqs) 19ed
Nelson pediatrics review (mcqs) 19ed
Nelson pediatrics review (mcqs) 19ed
Nelson pediatrics review (mcqs) 19ed
Nelson pediatrics review (mcqs) 19ed
Nelson pediatrics review (mcqs) 19ed
Nelson pediatrics review (mcqs) 19ed
Nelson pediatrics review (mcqs) 19ed
Nelson pediatrics review (mcqs) 19ed
Nelson pediatrics review (mcqs) 19ed
Nelson pediatrics review (mcqs) 19ed
Nelson pediatrics review (mcqs) 19ed
Nelson pediatrics review (mcqs) 19ed
Nelson pediatrics review (mcqs) 19ed
Nelson pediatrics review (mcqs) 19ed
Nelson pediatrics review (mcqs) 19ed
Nelson pediatrics review (mcqs) 19ed
Nelson pediatrics review (mcqs) 19ed
Nelson pediatrics review (mcqs) 19ed
Nelson pediatrics review (mcqs) 19ed
Nelson pediatrics review (mcqs) 19ed
Nelson pediatrics review (mcqs) 19ed
Nelson pediatrics review (mcqs) 19ed
Nelson pediatrics review (mcqs) 19ed
Nelson pediatrics review (mcqs) 19ed
Nelson pediatrics review (mcqs) 19ed
Nelson pediatrics review (mcqs) 19ed
Nelson pediatrics review (mcqs) 19ed
Nelson pediatrics review (mcqs) 19ed
Nelson pediatrics review (mcqs) 19ed
Nelson pediatrics review (mcqs) 19ed
Nelson pediatrics review (mcqs) 19ed
Nelson pediatrics review (mcqs) 19ed
Nelson pediatrics review (mcqs) 19ed
Nelson pediatrics review (mcqs) 19ed
Nelson pediatrics review (mcqs) 19ed
Nelson pediatrics review (mcqs) 19ed
Nelson pediatrics review (mcqs) 19ed
Nelson pediatrics review (mcqs) 19ed
Nelson pediatrics review (mcqs) 19ed
Nelson pediatrics review (mcqs) 19ed
Nelson pediatrics review (mcqs) 19ed
Nelson pediatrics review (mcqs) 19ed
Nelson pediatrics review (mcqs) 19ed
Nelson pediatrics review (mcqs) 19ed
Nelson pediatrics review (mcqs) 19ed
Nelson pediatrics review (mcqs) 19ed
Nelson pediatrics review (mcqs) 19ed
Nelson pediatrics review (mcqs) 19ed
Nelson pediatrics review (mcqs) 19ed
Nelson pediatrics review (mcqs) 19ed
Nelson pediatrics review (mcqs) 19ed
Nelson pediatrics review (mcqs) 19ed
Nelson pediatrics review (mcqs) 19ed
Nelson pediatrics review (mcqs) 19ed
Nelson pediatrics review (mcqs) 19ed
Nelson pediatrics review (mcqs) 19ed
Nelson pediatrics review (mcqs) 19ed
Nelson pediatrics review (mcqs) 19ed
Nelson pediatrics review (mcqs) 19ed
Nelson pediatrics review (mcqs) 19ed
Nelson pediatrics review (mcqs) 19ed
Nelson pediatrics review (mcqs) 19ed
Nelson pediatrics review (mcqs) 19ed
Nelson pediatrics review (mcqs) 19ed
Nelson pediatrics review (mcqs) 19ed
Nelson pediatrics review (mcqs) 19ed
Nelson pediatrics review (mcqs) 19ed
Nelson pediatrics review (mcqs) 19ed
Nelson pediatrics review (mcqs) 19ed
Nelson pediatrics review (mcqs) 19ed
Nelson pediatrics review (mcqs) 19ed
Nelson pediatrics review (mcqs) 19ed
Nelson pediatrics review (mcqs) 19ed
Nelson pediatrics review (mcqs) 19ed
Nelson pediatrics review (mcqs) 19ed
Nelson pediatrics review (mcqs) 19ed
Nelson pediatrics review (mcqs) 19ed
Nelson pediatrics review (mcqs) 19ed
Nelson pediatrics review (mcqs) 19ed
Nelson pediatrics review (mcqs) 19ed
Nelson pediatrics review (mcqs) 19ed
Nelson pediatrics review (mcqs) 19ed
Nelson pediatrics review (mcqs) 19ed
Nelson pediatrics review (mcqs) 19ed
Nelson pediatrics review (mcqs) 19ed
Nelson pediatrics review (mcqs) 19ed
Nelson pediatrics review (mcqs) 19ed
Nelson pediatrics review (mcqs) 19ed
Nelson pediatrics review (mcqs) 19ed
Nelson pediatrics review (mcqs) 19ed
Nelson pediatrics review (mcqs) 19ed
Nelson pediatrics review (mcqs) 19ed
Nelson pediatrics review (mcqs) 19ed
Nelson pediatrics review (mcqs) 19ed
Nelson pediatrics review (mcqs) 19ed
Nelson pediatrics review (mcqs) 19ed
Nelson pediatrics review (mcqs) 19ed
Nelson pediatrics review (mcqs) 19ed
Nelson pediatrics review (mcqs) 19ed
Nelson pediatrics review (mcqs) 19ed
Nelson pediatrics review (mcqs) 19ed
Nelson pediatrics review (mcqs) 19ed
Nelson pediatrics review (mcqs) 19ed
Nelson pediatrics review (mcqs) 19ed
Nelson pediatrics review (mcqs) 19ed
Nelson pediatrics review (mcqs) 19ed
Nelson pediatrics review (mcqs) 19ed
Nelson pediatrics review (mcqs) 19ed
Nelson pediatrics review (mcqs) 19ed
Nelson pediatrics review (mcqs) 19ed
Nelson pediatrics review (mcqs) 19ed
Nelson pediatrics review (mcqs) 19ed
Nelson pediatrics review (mcqs) 19ed
Nelson pediatrics review (mcqs) 19ed
Nelson pediatrics review (mcqs) 19ed
Nelson pediatrics review (mcqs) 19ed
Nelson pediatrics review (mcqs) 19ed
Nelson pediatrics review (mcqs) 19ed
Nelson pediatrics review (mcqs) 19ed
Nelson pediatrics review (mcqs) 19ed
Nelson pediatrics review (mcqs) 19ed
Nelson pediatrics review (mcqs) 19ed
Nelson pediatrics review (mcqs) 19ed
Nelson pediatrics review (mcqs) 19ed
Nelson pediatrics review (mcqs) 19ed
Nelson pediatrics review (mcqs) 19ed
Nelson pediatrics review (mcqs) 19ed
Nelson pediatrics review (mcqs) 19ed
Nelson pediatrics review (mcqs) 19ed
Nelson pediatrics review (mcqs) 19ed
Nelson pediatrics review (mcqs) 19ed
Nelson pediatrics review (mcqs) 19ed
Nelson pediatrics review (mcqs) 19ed
Nelson pediatrics review (mcqs) 19ed
Nelson pediatrics review (mcqs) 19ed
Nelson pediatrics review (mcqs) 19ed
Nelson pediatrics review (mcqs) 19ed
Nelson pediatrics review (mcqs) 19ed
Nelson pediatrics review (mcqs) 19ed
Nelson pediatrics review (mcqs) 19ed
Nelson pediatrics review (mcqs) 19ed
Nelson pediatrics review (mcqs) 19ed
Nelson pediatrics review (mcqs) 19ed
Nelson pediatrics review (mcqs) 19ed
Nelson pediatrics review (mcqs) 19ed
Nelson pediatrics review (mcqs) 19ed
Nelson pediatrics review (mcqs) 19ed
Nelson pediatrics review (mcqs) 19ed
Nelson pediatrics review (mcqs) 19ed
Nelson pediatrics review (mcqs) 19ed
Nelson pediatrics review (mcqs) 19ed
Nelson pediatrics review (mcqs) 19ed
Nelson pediatrics review (mcqs) 19ed
Nelson pediatrics review (mcqs) 19ed
Nelson pediatrics review (mcqs) 19ed
Nelson pediatrics review (mcqs) 19ed
Nelson pediatrics review (mcqs) 19ed
Nelson pediatrics review (mcqs) 19ed
Nelson pediatrics review (mcqs) 19ed
Nelson pediatrics review (mcqs) 19ed
Nelson pediatrics review (mcqs) 19ed
Nelson pediatrics review (mcqs) 19ed
Nelson pediatrics review (mcqs) 19ed
Nelson pediatrics review (mcqs) 19ed
Nelson pediatrics review (mcqs) 19ed
Nelson pediatrics review (mcqs) 19ed
Nelson pediatrics review (mcqs) 19ed
Nelson pediatrics review (mcqs) 19ed
Nelson pediatrics review (mcqs) 19ed
Nelson pediatrics review (mcqs) 19ed
Nelson pediatrics review (mcqs) 19ed
Nelson pediatrics review (mcqs) 19ed
Nelson pediatrics review (mcqs) 19ed
Nelson pediatrics review (mcqs) 19ed
Nelson pediatrics review (mcqs) 19ed

More Related Content

What's hot

Pediatric neurology mcq
Pediatric neurology mcqPediatric neurology mcq
Pediatric neurology mcqHari Meshram
 
Pediatrics OSCE pictures
Pediatrics OSCE pictures Pediatrics OSCE pictures
Pediatrics OSCE pictures Shahd Al Ali
 
OSCE MAY 2022-PART-5 -PAED.pptx
OSCE MAY 2022-PART-5 -PAED.pptxOSCE MAY 2022-PART-5 -PAED.pptx
OSCE MAY 2022-PART-5 -PAED.pptxGururajaRamaiah1
 
Heart failure in childhood
Heart failure in childhoodHeart failure in childhood
Heart failure in childhoodReyad Al_Faky
 
OSCE Pediatrics CME (Dr.D.Y.Patil Medical College)
OSCE Pediatrics CME (Dr.D.Y.Patil Medical College)OSCE Pediatrics CME (Dr.D.Y.Patil Medical College)
OSCE Pediatrics CME (Dr.D.Y.Patil Medical College)Dr Padmesh Vadakepat
 
Approach to bleeding neonate
Approach to bleeding neonateApproach to bleeding neonate
Approach to bleeding neonateDr Praman Kushwah
 
Persistent pulmonary hypertension of newborn PPHN
Persistent pulmonary hypertension of newborn PPHNPersistent pulmonary hypertension of newborn PPHN
Persistent pulmonary hypertension of newborn PPHNChandan Gowda
 
Neonatologymcqs 100403011632-phpapp02
Neonatologymcqs 100403011632-phpapp02Neonatologymcqs 100403011632-phpapp02
Neonatologymcqs 100403011632-phpapp02AmarSETIA
 
Urinary Tract Infections in children
 Urinary Tract Infections in children Urinary Tract Infections in children
Urinary Tract Infections in childrenAzad Haleem
 
Pediatrics OSCE Observed Stations Oct 2013
Pediatrics OSCE Observed Stations Oct 2013Pediatrics OSCE Observed Stations Oct 2013
Pediatrics OSCE Observed Stations Oct 2013Dr Padmesh Vadakepat
 
Pediatric hypertension
Pediatric hypertensionPediatric hypertension
Pediatric hypertensionTauhid Iqbali
 
OSCE in Pediatrics (Wadia, Sept 2011)
OSCE in Pediatrics (Wadia, Sept 2011)OSCE in Pediatrics (Wadia, Sept 2011)
OSCE in Pediatrics (Wadia, Sept 2011)Dr Padmesh Vadakepat
 
Dnb Pediatrics Theory Question bank
Dnb Pediatrics Theory Question bankDnb Pediatrics Theory Question bank
Dnb Pediatrics Theory Question bankDr Padmesh Vadakepat
 
Pediatric endocrinology review MCQs- part 6
Pediatric endocrinology review MCQs- part 6Pediatric endocrinology review MCQs- part 6
Pediatric endocrinology review MCQs- part 6Abdulmoein AlAgha
 
Anemia in children
Anemia in children Anemia in children
Anemia in children Sayed Ahmed
 
Neonatal fluid requirements and specials conditions
Neonatal fluid requirements and specials conditionsNeonatal fluid requirements and specials conditions
Neonatal fluid requirements and specials conditionsRakesh Verma
 
Bronchiolitis in children
Bronchiolitis in childrenBronchiolitis in children
Bronchiolitis in childrenAzad Haleem
 

What's hot (20)

Pediatric neurology mcq
Pediatric neurology mcqPediatric neurology mcq
Pediatric neurology mcq
 
Bleeding neonate
Bleeding neonateBleeding neonate
Bleeding neonate
 
Pediatrics OSCE pictures
Pediatrics OSCE pictures Pediatrics OSCE pictures
Pediatrics OSCE pictures
 
OSCE MAY 2022-PART-5 -PAED.pptx
OSCE MAY 2022-PART-5 -PAED.pptxOSCE MAY 2022-PART-5 -PAED.pptx
OSCE MAY 2022-PART-5 -PAED.pptx
 
OSCE Pediatrics (Pune)
OSCE Pediatrics (Pune)OSCE Pediatrics (Pune)
OSCE Pediatrics (Pune)
 
Heart failure in childhood
Heart failure in childhoodHeart failure in childhood
Heart failure in childhood
 
OSCE Pediatrics CME (Dr.D.Y.Patil Medical College)
OSCE Pediatrics CME (Dr.D.Y.Patil Medical College)OSCE Pediatrics CME (Dr.D.Y.Patil Medical College)
OSCE Pediatrics CME (Dr.D.Y.Patil Medical College)
 
Approach to bleeding neonate
Approach to bleeding neonateApproach to bleeding neonate
Approach to bleeding neonate
 
Persistent pulmonary hypertension of newborn PPHN
Persistent pulmonary hypertension of newborn PPHNPersistent pulmonary hypertension of newborn PPHN
Persistent pulmonary hypertension of newborn PPHN
 
Neonatologymcqs 100403011632-phpapp02
Neonatologymcqs 100403011632-phpapp02Neonatologymcqs 100403011632-phpapp02
Neonatologymcqs 100403011632-phpapp02
 
Urinary Tract Infections in children
 Urinary Tract Infections in children Urinary Tract Infections in children
Urinary Tract Infections in children
 
Pediatrics OSCE Observed Stations Oct 2013
Pediatrics OSCE Observed Stations Oct 2013Pediatrics OSCE Observed Stations Oct 2013
Pediatrics OSCE Observed Stations Oct 2013
 
Pediatric hypertension
Pediatric hypertensionPediatric hypertension
Pediatric hypertension
 
Pediatrics usmle rx mcq
Pediatrics usmle rx mcqPediatrics usmle rx mcq
Pediatrics usmle rx mcq
 
OSCE in Pediatrics (Wadia, Sept 2011)
OSCE in Pediatrics (Wadia, Sept 2011)OSCE in Pediatrics (Wadia, Sept 2011)
OSCE in Pediatrics (Wadia, Sept 2011)
 
Dnb Pediatrics Theory Question bank
Dnb Pediatrics Theory Question bankDnb Pediatrics Theory Question bank
Dnb Pediatrics Theory Question bank
 
Pediatric endocrinology review MCQs- part 6
Pediatric endocrinology review MCQs- part 6Pediatric endocrinology review MCQs- part 6
Pediatric endocrinology review MCQs- part 6
 
Anemia in children
Anemia in children Anemia in children
Anemia in children
 
Neonatal fluid requirements and specials conditions
Neonatal fluid requirements and specials conditionsNeonatal fluid requirements and specials conditions
Neonatal fluid requirements and specials conditions
 
Bronchiolitis in children
Bronchiolitis in childrenBronchiolitis in children
Bronchiolitis in children
 

Viewers also liked

DNB PEDIATRICS THEORY QUESTION BANK (updated Oct 2014)
DNB PEDIATRICS THEORY QUESTION BANK (updated Oct 2014)DNB PEDIATRICS THEORY QUESTION BANK (updated Oct 2014)
DNB PEDIATRICS THEORY QUESTION BANK (updated Oct 2014)Dr Padmesh Vadakepat
 
MCQs - Urinary Tract Infection in Children
MCQs - Urinary Tract Infection in ChildrenMCQs - Urinary Tract Infection in Children
MCQs - Urinary Tract Infection in ChildrenDr Padmesh Vadakepat
 
E-Learning in Newborn Health A paradigm shift in continuing professional deve...
E-Learning in Newborn HealthA paradigm shift in continuing professional deve...E-Learning in Newborn HealthA paradigm shift in continuing professional deve...
E-Learning in Newborn Health A paradigm shift in continuing professional deve...nisaiims
 
approach to hyponatremia in children
approach to hyponatremia in childrenapproach to hyponatremia in children
approach to hyponatremia in childrendrranvijayrana
 

Viewers also liked (6)

Newborn mcq
Newborn mcqNewborn mcq
Newborn mcq
 
DNB PEDIATRICS THEORY QUESTION BANK (updated Oct 2014)
DNB PEDIATRICS THEORY QUESTION BANK (updated Oct 2014)DNB PEDIATRICS THEORY QUESTION BANK (updated Oct 2014)
DNB PEDIATRICS THEORY QUESTION BANK (updated Oct 2014)
 
Indiska zapoest
Indiska zapoestIndiska zapoest
Indiska zapoest
 
MCQs - Urinary Tract Infection in Children
MCQs - Urinary Tract Infection in ChildrenMCQs - Urinary Tract Infection in Children
MCQs - Urinary Tract Infection in Children
 
E-Learning in Newborn Health A paradigm shift in continuing professional deve...
E-Learning in Newborn HealthA paradigm shift in continuing professional deve...E-Learning in Newborn HealthA paradigm shift in continuing professional deve...
E-Learning in Newborn Health A paradigm shift in continuing professional deve...
 
approach to hyponatremia in children
approach to hyponatremia in childrenapproach to hyponatremia in children
approach to hyponatremia in children
 

Similar to Nelson pediatrics review (mcqs) 19ed

Pediatric Nutritional Deficiencies - Spot Diagnosis
Pediatric Nutritional Deficiencies - Spot DiagnosisPediatric Nutritional Deficiencies - Spot Diagnosis
Pediatric Nutritional Deficiencies - Spot DiagnosisFatima Farid
 
Intrauterine growth restriction
Intrauterine growth restrictionIntrauterine growth restriction
Intrauterine growth restrictiondrmcbansal
 
Epilepsy in women
Epilepsy in womenEpilepsy in women
Epilepsy in womenPS Deb
 
CARE OF LOW BIRTH WEIGHT CHILDREN
CARE OF LOW BIRTH WEIGHT CHILDREN CARE OF LOW BIRTH WEIGHT CHILDREN
CARE OF LOW BIRTH WEIGHT CHILDREN DR DHAN RAJ BAGRI
 
WHO guidelines on Nutrition
WHO guidelines on NutritionWHO guidelines on Nutrition
WHO guidelines on NutritionSanthiNori1
 
nutrientsReviewNutritional Needs and Support for Child.docx
nutrientsReviewNutritional Needs and Support for Child.docxnutrientsReviewNutritional Needs and Support for Child.docx
nutrientsReviewNutritional Needs and Support for Child.docxdunhamadell
 
hyperemesis gravidarum.pptx
hyperemesis gravidarum.pptxhyperemesis gravidarum.pptx
hyperemesis gravidarum.pptxSharwajitJha1
 
Hyperemesis Gravidarum
Hyperemesis GravidarumHyperemesis Gravidarum
Hyperemesis GravidarumJoisy S. Joy
 
Chronic Liver Disease in pediatric: a case presentation and discussion
Chronic Liver Disease in pediatric: a case presentation and discussionChronic Liver Disease in pediatric: a case presentation and discussion
Chronic Liver Disease in pediatric: a case presentation and discussionDr Abdalla M. Gamal
 

Similar to Nelson pediatrics review (mcqs) 19ed (20)

LBW
LBWLBW
LBW
 
Metabolic screening in newborn
Metabolic screening in newborn   Metabolic screening in newborn
Metabolic screening in newborn
 
Failure to thrive
Failure to thriveFailure to thrive
Failure to thrive
 
Short stature
Short stature Short stature
Short stature
 
ytp newsletter shehla jamal
ytp newsletter shehla jamalytp newsletter shehla jamal
ytp newsletter shehla jamal
 
Pediatric Nutritional Deficiencies - Spot Diagnosis
Pediatric Nutritional Deficiencies - Spot DiagnosisPediatric Nutritional Deficiencies - Spot Diagnosis
Pediatric Nutritional Deficiencies - Spot Diagnosis
 
Intrauterine growth restriction
Intrauterine growth restrictionIntrauterine growth restriction
Intrauterine growth restriction
 
Epilepsy in women
Epilepsy in womenEpilepsy in women
Epilepsy in women
 
Severe acute malnutrition
Severe acute malnutritionSevere acute malnutrition
Severe acute malnutrition
 
CARE OF LOW BIRTH WEIGHT CHILDREN
CARE OF LOW BIRTH WEIGHT CHILDREN CARE OF LOW BIRTH WEIGHT CHILDREN
CARE OF LOW BIRTH WEIGHT CHILDREN
 
Chapter 9 protein metabolism
Chapter 9 protein metabolismChapter 9 protein metabolism
Chapter 9 protein metabolism
 
Iugr chandni
Iugr chandniIugr chandni
Iugr chandni
 
WHO guidelines on Nutrition
WHO guidelines on NutritionWHO guidelines on Nutrition
WHO guidelines on Nutrition
 
nutrientsReviewNutritional Needs and Support for Child.docx
nutrientsReviewNutritional Needs and Support for Child.docxnutrientsReviewNutritional Needs and Support for Child.docx
nutrientsReviewNutritional Needs and Support for Child.docx
 
hyperemesis gravidarum.pptx
hyperemesis gravidarum.pptxhyperemesis gravidarum.pptx
hyperemesis gravidarum.pptx
 
emesis .ppt
emesis .pptemesis .ppt
emesis .ppt
 
Hyperemesis Gravidarum
Hyperemesis GravidarumHyperemesis Gravidarum
Hyperemesis Gravidarum
 
Protein energy malnurition
Protein energy malnuritionProtein energy malnurition
Protein energy malnurition
 
Chronic Liver Disease in pediatric: a case presentation and discussion
Chronic Liver Disease in pediatric: a case presentation and discussionChronic Liver Disease in pediatric: a case presentation and discussion
Chronic Liver Disease in pediatric: a case presentation and discussion
 
Maternal Nutrition.pptx
Maternal Nutrition.pptxMaternal Nutrition.pptx
Maternal Nutrition.pptx
 

More from palpeds

SLE 5000 HFOV Dr.ALLAM ABUHAMDA CONSULTANT NEONATOLOGIST
SLE 5000 HFOV Dr.ALLAM ABUHAMDA CONSULTANT NEONATOLOGIST SLE 5000 HFOV Dr.ALLAM ABUHAMDA CONSULTANT NEONATOLOGIST
SLE 5000 HFOV Dr.ALLAM ABUHAMDA CONSULTANT NEONATOLOGIST palpeds
 
Principles of Neurodevelopment in Newborn Dr.Hanan Elwadia
Principles of Neurodevelopment in Newborn Dr.Hanan ElwadiaPrinciples of Neurodevelopment in Newborn Dr.Hanan Elwadia
Principles of Neurodevelopment in Newborn Dr.Hanan Elwadiapalpeds
 
Trisomy 07-04-2015
Trisomy 07-04-2015Trisomy 07-04-2015
Trisomy 07-04-2015palpeds
 
Brachial plexus
Brachial plexusBrachial plexus
Brachial plexuspalpeds
 
Newborn Examination
Newborn ExaminationNewborn Examination
Newborn Examinationpalpeds
 
Neonatal guidelines NHS 2011 2013
Neonatal guidelines NHS 2011 2013Neonatal guidelines NHS 2011 2013
Neonatal guidelines NHS 2011 2013palpeds
 
Mechanical ventilation in neonates
Mechanical ventilation in neonatesMechanical ventilation in neonates
Mechanical ventilation in neonatespalpeds
 
Neonatal resuccitation sami sayegh
Neonatal resuccitation sami sayeghNeonatal resuccitation sami sayegh
Neonatal resuccitation sami sayeghpalpeds
 
Neonatal Parenteral Feeding Dr.allam abuhamda Consultant of Pediatrics Neona...
 Neonatal Parenteral Feeding Dr.allam abuhamda Consultant of Pediatrics Neona... Neonatal Parenteral Feeding Dr.allam abuhamda Consultant of Pediatrics Neona...
Neonatal Parenteral Feeding Dr.allam abuhamda Consultant of Pediatrics Neona...palpeds
 
Mechanical ventilation in neonates
Mechanical ventilation in neonatesMechanical ventilation in neonates
Mechanical ventilation in neonatespalpeds
 

More from palpeds (10)

SLE 5000 HFOV Dr.ALLAM ABUHAMDA CONSULTANT NEONATOLOGIST
SLE 5000 HFOV Dr.ALLAM ABUHAMDA CONSULTANT NEONATOLOGIST SLE 5000 HFOV Dr.ALLAM ABUHAMDA CONSULTANT NEONATOLOGIST
SLE 5000 HFOV Dr.ALLAM ABUHAMDA CONSULTANT NEONATOLOGIST
 
Principles of Neurodevelopment in Newborn Dr.Hanan Elwadia
Principles of Neurodevelopment in Newborn Dr.Hanan ElwadiaPrinciples of Neurodevelopment in Newborn Dr.Hanan Elwadia
Principles of Neurodevelopment in Newborn Dr.Hanan Elwadia
 
Trisomy 07-04-2015
Trisomy 07-04-2015Trisomy 07-04-2015
Trisomy 07-04-2015
 
Brachial plexus
Brachial plexusBrachial plexus
Brachial plexus
 
Newborn Examination
Newborn ExaminationNewborn Examination
Newborn Examination
 
Neonatal guidelines NHS 2011 2013
Neonatal guidelines NHS 2011 2013Neonatal guidelines NHS 2011 2013
Neonatal guidelines NHS 2011 2013
 
Mechanical ventilation in neonates
Mechanical ventilation in neonatesMechanical ventilation in neonates
Mechanical ventilation in neonates
 
Neonatal resuccitation sami sayegh
Neonatal resuccitation sami sayeghNeonatal resuccitation sami sayegh
Neonatal resuccitation sami sayegh
 
Neonatal Parenteral Feeding Dr.allam abuhamda Consultant of Pediatrics Neona...
 Neonatal Parenteral Feeding Dr.allam abuhamda Consultant of Pediatrics Neona... Neonatal Parenteral Feeding Dr.allam abuhamda Consultant of Pediatrics Neona...
Neonatal Parenteral Feeding Dr.allam abuhamda Consultant of Pediatrics Neona...
 
Mechanical ventilation in neonates
Mechanical ventilation in neonatesMechanical ventilation in neonates
Mechanical ventilation in neonates
 

Recently uploaded

ayurvedic formulations herbal drug technologyppt
ayurvedic formulations herbal drug technologypptayurvedic formulations herbal drug technologyppt
ayurvedic formulations herbal drug technologypptPradnya Wadekar
 
Red Blood Cells_anemia & polycythemia.pdf
Red Blood Cells_anemia & polycythemia.pdfRed Blood Cells_anemia & polycythemia.pdf
Red Blood Cells_anemia & polycythemia.pdfMedicoseAcademics
 
AUTONOMIC NERVOUS SYSTEM organization and functions
AUTONOMIC NERVOUS SYSTEM organization and functionsAUTONOMIC NERVOUS SYSTEM organization and functions
AUTONOMIC NERVOUS SYSTEM organization and functionsMedicoseAcademics
 
SGK ĐIỆN GIẬT ĐHYHN RẤT LÀ HAY TUYỆT VỜI.pdf
SGK ĐIỆN GIẬT ĐHYHN        RẤT LÀ HAY TUYỆT VỜI.pdfSGK ĐIỆN GIẬT ĐHYHN        RẤT LÀ HAY TUYỆT VỜI.pdf
SGK ĐIỆN GIẬT ĐHYHN RẤT LÀ HAY TUYỆT VỜI.pdfHongBiThi1
 
Different drug regularity bodies in different countries.
Different drug regularity bodies in different countries.Different drug regularity bodies in different countries.
Different drug regularity bodies in different countries.kishan singh tomar
 
Breast cancer -ONCO IN MEDICAL AND SURGICAL NURSING.pptx
Breast cancer -ONCO IN MEDICAL AND SURGICAL NURSING.pptxBreast cancer -ONCO IN MEDICAL AND SURGICAL NURSING.pptx
Breast cancer -ONCO IN MEDICAL AND SURGICAL NURSING.pptxNaveenkumar267201
 
SGK NGẠT NƯỚC ĐHYHN RẤT LÀ HAY NHA .pdf
SGK NGẠT NƯỚC ĐHYHN RẤT LÀ HAY NHA    .pdfSGK NGẠT NƯỚC ĐHYHN RẤT LÀ HAY NHA    .pdf
SGK NGẠT NƯỚC ĐHYHN RẤT LÀ HAY NHA .pdfHongBiThi1
 
How to cure cirrhosis and chronic hepatitis naturally
How to cure cirrhosis and chronic hepatitis naturallyHow to cure cirrhosis and chronic hepatitis naturally
How to cure cirrhosis and chronic hepatitis naturallyZurück zum Ursprung
 
Female Reproductive Physiology Before Pregnancy
Female Reproductive Physiology Before PregnancyFemale Reproductive Physiology Before Pregnancy
Female Reproductive Physiology Before PregnancyMedicoseAcademics
 
CPR.nursingoutlook.pdf , Bsc nursing student
CPR.nursingoutlook.pdf , Bsc nursing studentCPR.nursingoutlook.pdf , Bsc nursing student
CPR.nursingoutlook.pdf , Bsc nursing studentsaileshpanda05
 
Clinical Research Informatics Year-in-Review 2024
Clinical Research Informatics Year-in-Review 2024Clinical Research Informatics Year-in-Review 2024
Clinical Research Informatics Year-in-Review 2024Peter Embi
 
MedMatch: Your Health, Our Mission. Pitch deck.
MedMatch: Your Health, Our Mission. Pitch deck.MedMatch: Your Health, Our Mission. Pitch deck.
MedMatch: Your Health, Our Mission. Pitch deck.whalesdesign
 
Generative AI in Health Care a scoping review and a persoanl experience.
Generative AI in Health Care a scoping review and a persoanl experience.Generative AI in Health Care a scoping review and a persoanl experience.
Generative AI in Health Care a scoping review and a persoanl experience.Vaikunthan Rajaratnam
 
Neurological history taking (2024) .
Neurological  history  taking  (2024)  .Neurological  history  taking  (2024)  .
Neurological history taking (2024) .Mohamed Rizk Khodair
 
BENIGN BREAST DISEASE
BENIGN BREAST DISEASE BENIGN BREAST DISEASE
BENIGN BREAST DISEASE Mamatha Lakka
 
blood bank management system project report
blood bank management system project reportblood bank management system project report
blood bank management system project reportNARMADAPETROLEUMGAS
 
PAIN/CLASSIFICATION AND MANAGEMENT OF PAIN.pdf
PAIN/CLASSIFICATION AND MANAGEMENT OF PAIN.pdfPAIN/CLASSIFICATION AND MANAGEMENT OF PAIN.pdf
PAIN/CLASSIFICATION AND MANAGEMENT OF PAIN.pdfDolisha Warbi
 

Recently uploaded (20)

ayurvedic formulations herbal drug technologyppt
ayurvedic formulations herbal drug technologypptayurvedic formulations herbal drug technologyppt
ayurvedic formulations herbal drug technologyppt
 
Immune labs basics part 1 acute phase reactants ESR, CRP Ahmed Yehia Ismaeel,...
Immune labs basics part 1 acute phase reactants ESR, CRP Ahmed Yehia Ismaeel,...Immune labs basics part 1 acute phase reactants ESR, CRP Ahmed Yehia Ismaeel,...
Immune labs basics part 1 acute phase reactants ESR, CRP Ahmed Yehia Ismaeel,...
 
Red Blood Cells_anemia & polycythemia.pdf
Red Blood Cells_anemia & polycythemia.pdfRed Blood Cells_anemia & polycythemia.pdf
Red Blood Cells_anemia & polycythemia.pdf
 
Biologic therapy ice breaking in rheumatology, Case based approach with appli...
Biologic therapy ice breaking in rheumatology, Case based approach with appli...Biologic therapy ice breaking in rheumatology, Case based approach with appli...
Biologic therapy ice breaking in rheumatology, Case based approach with appli...
 
AUTONOMIC NERVOUS SYSTEM organization and functions
AUTONOMIC NERVOUS SYSTEM organization and functionsAUTONOMIC NERVOUS SYSTEM organization and functions
AUTONOMIC NERVOUS SYSTEM organization and functions
 
SGK ĐIỆN GIẬT ĐHYHN RẤT LÀ HAY TUYỆT VỜI.pdf
SGK ĐIỆN GIẬT ĐHYHN        RẤT LÀ HAY TUYỆT VỜI.pdfSGK ĐIỆN GIẬT ĐHYHN        RẤT LÀ HAY TUYỆT VỜI.pdf
SGK ĐIỆN GIẬT ĐHYHN RẤT LÀ HAY TUYỆT VỜI.pdf
 
Different drug regularity bodies in different countries.
Different drug regularity bodies in different countries.Different drug regularity bodies in different countries.
Different drug regularity bodies in different countries.
 
Breast cancer -ONCO IN MEDICAL AND SURGICAL NURSING.pptx
Breast cancer -ONCO IN MEDICAL AND SURGICAL NURSING.pptxBreast cancer -ONCO IN MEDICAL AND SURGICAL NURSING.pptx
Breast cancer -ONCO IN MEDICAL AND SURGICAL NURSING.pptx
 
SGK NGẠT NƯỚC ĐHYHN RẤT LÀ HAY NHA .pdf
SGK NGẠT NƯỚC ĐHYHN RẤT LÀ HAY NHA    .pdfSGK NGẠT NƯỚC ĐHYHN RẤT LÀ HAY NHA    .pdf
SGK NGẠT NƯỚC ĐHYHN RẤT LÀ HAY NHA .pdf
 
How to cure cirrhosis and chronic hepatitis naturally
How to cure cirrhosis and chronic hepatitis naturallyHow to cure cirrhosis and chronic hepatitis naturally
How to cure cirrhosis and chronic hepatitis naturally
 
Female Reproductive Physiology Before Pregnancy
Female Reproductive Physiology Before PregnancyFemale Reproductive Physiology Before Pregnancy
Female Reproductive Physiology Before Pregnancy
 
CPR.nursingoutlook.pdf , Bsc nursing student
CPR.nursingoutlook.pdf , Bsc nursing studentCPR.nursingoutlook.pdf , Bsc nursing student
CPR.nursingoutlook.pdf , Bsc nursing student
 
Clinical Research Informatics Year-in-Review 2024
Clinical Research Informatics Year-in-Review 2024Clinical Research Informatics Year-in-Review 2024
Clinical Research Informatics Year-in-Review 2024
 
MedMatch: Your Health, Our Mission. Pitch deck.
MedMatch: Your Health, Our Mission. Pitch deck.MedMatch: Your Health, Our Mission. Pitch deck.
MedMatch: Your Health, Our Mission. Pitch deck.
 
Generative AI in Health Care a scoping review and a persoanl experience.
Generative AI in Health Care a scoping review and a persoanl experience.Generative AI in Health Care a scoping review and a persoanl experience.
Generative AI in Health Care a scoping review and a persoanl experience.
 
Neurological history taking (2024) .
Neurological  history  taking  (2024)  .Neurological  history  taking  (2024)  .
Neurological history taking (2024) .
 
BENIGN BREAST DISEASE
BENIGN BREAST DISEASE BENIGN BREAST DISEASE
BENIGN BREAST DISEASE
 
blood bank management system project report
blood bank management system project reportblood bank management system project report
blood bank management system project report
 
PAIN/CLASSIFICATION AND MANAGEMENT OF PAIN.pdf
PAIN/CLASSIFICATION AND MANAGEMENT OF PAIN.pdfPAIN/CLASSIFICATION AND MANAGEMENT OF PAIN.pdf
PAIN/CLASSIFICATION AND MANAGEMENT OF PAIN.pdf
 
GOUT UPDATE AHMED YEHIA 2024, case based approach with application of the lat...
GOUT UPDATE AHMED YEHIA 2024, case based approach with application of the lat...GOUT UPDATE AHMED YEHIA 2024, case based approach with application of the lat...
GOUT UPDATE AHMED YEHIA 2024, case based approach with application of the lat...
 

Nelson pediatrics review (mcqs) 19ed

  • 1. Nelson Pediatrics Review(MCQs)19ed ‫ﺩ‬ ‫ﺘﻤﻨﻴﺎﺕ‬ ‫ﻤﻊ‬.‫ﺍﻟﺩﻋﺎﺀ‬ ‫ﺼﺎﻟﺢ‬ ‫ﻤﻥ‬ ‫ﺘﻨﺴﻭﻨﺎ‬ ‫ﻭﻻ‬ ، ‫ﺒﺎﻟﺘﻭﻓﻴﻕ‬ ‫ﺍﻟﺩﺠﺎﻨﻲ‬ ‫ﺯﻫﻴﺭ‬ -------------------------------------------------------------- 1. Which of the following statements regarding foster care is true? □A permanency plan must be made for a child in foster care no later than 12 mo from the child's entry into care □A minority of children in foster care have a history of abuse or neglect □The mission of foster care is to safely care for children while providing services to families to promote reunification □Most (>70%) of children in foster care are reunited with their families ■A and C description The mission of foster care is to provide for the health, safety, and well-being of children while assisting their families with services to promote reunification. Children entering foster care have frequently experienced early childhood trauma. More than 70% have a history of abuse, neglect, or both. Only about 50% of children achieve reunification. In the USA, the Adoption and Safe Families Act (P.L. 105-89) passed in 1997 requires that a permanency plan be made for each child no later than 12 mo after entry to foster care and that a petition to terminate parental rights typically must be filed when a child has been in foster care for at least 15 of the previous 22 mo. (See Chapter 35, page 134, and e35-1.) 2. A 4 yr old girl is admitted to the hospital for her third evaluation for vaginal bleeding. The mother noted bright red blood on the child's underwear. Previous examinations revealed a normal 4 yr old girl, Tanner stage 1, with normal external genitalia. Pelvic ultrasound results were normal, as was the serum estradiol level. The hemoglobin and platelet counts were normal, as were the bleeding time and coagulation studies. Findings on pelvic examination conducted under anesthesia also were normal. The next step in the examination is to: ■Determine the blood type of the blood on the underwear □Interrogate the father □Isolate the parents and child □Determine von Willebrand factor levels
  • 2. □Measure fibronectin in the vagina description Consideration of factitious disorder by proxy should be triggered when the reported symptoms are repeatedly noted by only one parent, appropriate testing fails to confirm a diagnosis, and seemingly appropriate treatment is ineffective. At times, the child's symptoms, their course, or the response to treatment may be incompatible with any recognized disease. Preverbal children are usually involved. Bleeding is a particularly common presentation. This may be caused by adding dyes to samples, adding blood (e.g., from the mother) to the child's sample, or giving the child an anticoagulant (e.g., warfarin). (See Chapter 37, page 146.) 3. Munchausen syndrome by proxy is characterized by all of the following EXCEPT: □Mother who appears devoted and wins over members of care team □Multiple hospitalizations and investigations without diagnosis □Symptoms on history but not witnessed by medical team ■Symptoms occurring in presence of different caregivers (e.g., while mother is out of town) □Use of medications or toxins description Symptoms in young children are mostly associated with proximity of the offending caregiver to the child. The mother may present as a devoted or even model parent who forms close relationships with members of the health care team. While appearing very interested in her child's condition, she may be relatively distant emotionally. (See Chapter 37, page 146.) 4. Which statement is false? ■Malnutrition is the second leading cause of acquired immune deficiency worldwide behind HIV infection □Zinc is important in immune function and linear growth □Kwashiorkor and marasmus are rare in developed countries □The Western diet is associated with increased noncommunicable disease description The significant global burden of malnutrition and undernutrition is the leading worldwide cause of acquired immunodeficiency and the major underlying factor for morbidity and mortality globally for children <5 yr of age. Zinc is a micronutrient that supports multiple metabolic functions in the body, is essential for normal immune functioning, and is required to support linear growth; zinc deficiency is associated with impaired immune functioning and poor linear growth. In parallel to the risk for nutrient and energy deficiencies, issues relating to excesses pose important challenges because of their negative health effects, such as obesity or cardiovascular disease risk factors. The nutrition transition under way in the
  • 3. developing world from traditional diets to the Western diet has been associated with increases in noncommunicable diseases, often coexisting with undernutrition and malnutrition, observed sometimes in the same communities or even the same families. (See e41-1.) 5. Components of energy expenditure in children include: □Thermal effect of food □Basal metabolic rate □Energy for physical activity □Energy to support growth ■All of the above description The 3 components of energy expenditure in adults are the basal metabolic rate, the thermal effect of food (energy required for digestion and absorption), and energy for physical activity. Additional energy intake and expenditure are required to support growth and development for children. (See e41-4.) 6. Which of the following clinical scenarios increases the risk of vitamin A deficiency? □Vegetarian diet □Chronic intestinal disorders □Zinc deficiency ■B and C □All of the above description Vitamin A is an essential micronutrient because it cannot be biogenerated de novo by animals. It must be obtained from plants in the form of provitamin-A carotenoids. In the USA, grains and vegetables supply approximately 55% and dairy and meat products supply approximately 30% of vitamin A intake from food. Vitamin A and the provitamins-A are fat soluble, and their absorption depends on the presence of adequate lipid and protein within the meal. Chronic intestinal disorders or lipid malabsorption syndromes can result in vitamin A deficiency. In developing countries, subclinical or clinical zinc deficiency can increase the risk of vitamin A deficiency. There is also some evidence of marginal zinc intakes in children in the USA. (See Chapter 45, page 188.) 7. Which statement about vitamin A toxicity is NOT true?
  • 4. □Excess vitamin A in utero can cause congenital malformations □It may present as pseudotumor cerebri ■An infant with a preference for carrots and butternut squash may develop toxicity □It may cause fissures at the corners of the mouth, pruritus, and alopecia □Symptoms subside rapidly after withdrawing the source of the vitamin description Excessive intake of carotenoids is not associated with toxicity but can cause yellow coloration of the skin that disappears when intake is reduced; this disorder (carotenemia) is especially likely to occur in children with liver disease, diabetes mellitus, or hypothyroidism and in those who do not have enzymes that metabolize carotenoids. (See Chapter 45, page 191.) 8. Which statement about vitamin E is false? □The most common form of vitamin E is tocopherol ■Premature infants given formula with a high content of polyunsaturated fatty acids and iron supplementation are protected from deficiency □Cholestatic liver disease increases the risk of deficiency □Premature infants with vitamin E deficiency develop hemolysis, thrombocytosis, and edema □Prolonged vitamin E deficiency causes a severe, progressive neurologic disorder description Premature infants are particularly susceptible to vitamin E deficiency because there is significant transfer of vitamin E during the last trimester of pregnancy. Vitamin E deficiency in premature infants causes thrombocytosis, edema, and hemolysis, potentially causing anemia. The risk of symptomatic vitamin E deficiency was increased by the use of formulas for premature infants that had a high content of polyunsaturated fatty acids (PUFAs). These formulas led to a high content of PUFAs in red blood cell membranes, making them more susceptible to oxidative stress, which could be ameliorated by vitamin E. Oxidative stress was augmented by aggressive use of iron supplementation; iron increases the production of oxygen radicals. The incidence of hemolysis due to vitamin E deficiency in premature infants decreased secondary to the use of formulas with a lower content of polyunsaturated fatty acids, less-aggressive use of iron, and provision of adequate vitamin E. (See e49-1.) 9. Manifestations of hyperkalemia include all of the following EXCEPT:
  • 5. □Paresthesias □Weakness □Paralysis □Wide QRS complex ■Tetany description The most important effects of hyperkalemia are due to the role of potassium in membrane polarization. The cardiac conduction system is usually the dominant concern. Changes in the electrocardiogram (ECG) begin with peaking of the T waves. This is followed, as the potassium level increases, by ST segment depression, an increased PR interval, flattening of the P wave, and widening of the QRS complex. This process can eventually progress to ventricular fibrillation. Asystole may also occur. Some patients have paresthesias, fasciculations, weakness, and even an ascending paralysis, but cardiac toxicity usually precedes these clinical symptoms, emphasizing the danger of assuming that an absence of symptoms implies an absence of danger. (See Chapter 52, page 221.) 10. Hyperkalemia may be associated with all of the following EXCEPT: □Succinylcholine use □Burns □Trauma □Chemotherapy ■Metabolic alkalosis □Digitalis toxicity □Uremia description Many causes of hyperkalemia result in metabolic acidosis; a metabolic acidosis worsens hyperkalemia through the transcellular shift of potassium out of cells. Renal insufficiency is a common cause of the combination of metabolic acidosis and hyperkalemia. This association is also seen in diseases associated with aldosterone insufficiency or aldosterone resistance. (See Chapter 52, page 221.) 11. The best method to reduce the potassium level during hyperkalemia, by reducing the body burden of potassium, is:
  • 6. □Sodium bicarbonate infusion □Glucose and insulin infusion □Calcium infusion □Albuterol aerosol ■Kayexalate enema description Treatment of hyperkalemia has 2 basic goals: (1) to stabilize the heart to prevent life-threatening arrhythmias and (2) to remove potassium from the body. The treatments that acutely prevent arrhythmias all have the advantage of working quickly (within minutes) but do not remove potassium from the body. Calcium stabilizes the cell membrane of heart cells, preventing arrhythmias. It is given intravenously over a few minutes, and its action is almost immediate. Several medications cause potassium to move intracellularly and thus rapidly reduce the plasma level to prevent arrhythmias. These include bicarbonate, insulin and glucose, and nebulized albuterol. However, these medicines do not remove potassium from the body. To reduce the total body potassium, 3 options are available. In patients who are not anuric, a loop diuretic increases renal excretion of potassium. A high dose may be required in a patient with significant renal insufficiency. Sodium polystyrene sulfonate (Kayexalate) is an exchange resin that is given either rectally or orally. Sodium in the resin is exchanged for body potassium, and the potassium-containing resin is then excreted from the body. Some patients require dialysis for acute removal of potassium. Dialysis is often necessary if the patient has either severe renal failure or an especially high rate of endogenous potassium release, as is sometimes present with tumor lysis syndrome or rhabdomyolysis. (See Chapter 52, page 222.) 12. Clinical manifestations of hypokalemia include all of the following EXCEPT: □ECG changes □Paralysis □Urinary retention □Constipation □Muscle cramps ■Blurry vision description The heart and skeletal muscle are especially vulnerable to hypokalemia. ECG changes include a flattened T wave, a depressed ST segment, and the appearance of a U wave, which is located between the T wave (if still visible) and the P wave. Ventricular fibrillation and torsades de pointes may occur, although usually only in the context of underlying heart disease. The clinical consequences of hypokalemia in skeletal muscle include muscle weakness and cramps. Paralysis is a possible complication, generally only at potassium levels <2.5 mEq/L. It usually starts in the legs and moves to the arms. Respiratory paralysis may require mechanical ventilation. Some patients have rhabdomyolysis; the risk increases with
  • 7. exercise. Hypokalemia slows gastrointestinal motility. This effect manifests as constipation; with potassium levels <2.5 mEq/L, an ileus may occur. Hypokalemia impairs bladder function, potentially leading to urinary retention. (See Chapter 52, page 224.) 13. From the following list, choose the route(s) by which insensible water loss may occur: 1. Sweat, 2. Fecal loss, 3. Evaporative loss from skin, 4. Respiratory water loss, 5. Obligate water for urinary solute excretion □1 and 3 □1, 2, and 3 □3 only ■3 and 4 □2 and 5 description Water is a crucial component of maintenance fluid therapy because of the obligatory daily water losses. These losses are both measurable (urine, stool) and not measurable (insensible losses from the skin and lungs). Failure to replace these losses leads to a child who is thirsty, uncomfortable, and, ultimately, dehydrated. (See Chapter 53, page 242.) 14. Which of the following is a goal of maintenance fluids? □Diminish protein degradation □Prevent dehydration ■Prevent hunger □Prevent electrolyte derangements □Prevent ketoacidosis description The glucose in maintenance fluids provides approximately 20% of the normal caloric needs of the patient, prevents the development of starvation ketoacidosis, and diminishes the protein degradation that would occur if the patient received no calories. Glucose also provides added osmoles, thus avoiding the administration of hypotonic fluids that may cause hemolysis. Maintenance fluids do not provide adequate calories, protein, fat, minerals, or vitamins. This fact is typically not problematic for a patient receiving intravenous fluids for a few days. A patient receiving maintenance intravenous fluids is receiving inadequate calories and will lose 0.5-1% of weight each day. Table 53-1 summarizes the goals of maintenance fluids. (See Chapter 53, page 242.)
  • 8. 15. Which patient has an elevated risk of hyponatremia with standard maintenance fluid therapy (D5 ½ NS if >10 kg, D5 ¼ NS if <10 kg)? □6 mo old NPO for elective hernia repair □4 month old with bronchiolitis and poor oral intake □13 yr old status post motor vehicle accident with multiple fractures, requiring treatment with narcotics and antiemetics □8 yr old with nephrotic syndrome □A and D ■B, C, and D description Patients who are producing antidiuretic hormone (ADH) may retain water, creating a risk of hyponatremia due to water intoxication. Patients who may be producing ADH owing to subtle volume depletion or other mechanisms (respiratory disease, stress, pain, nausea, medications such as narcotics) may be more safely treated with fluids that have a higher sodium concentration, with a decrease in fluid rate, or with a combination of these strategies. Patients with persistent ADH production due to an underlying disease process (syndrome of inappropriate ADH secretion [SIADH], congestive heart failure, nephrotic syndrome, liver disease) should receive less than maintenance fluids. Treatment is individualized, and careful monitoring is critical. Special caution is needed in patients who are known to have low-normal serum sodium concentrations or hyponatremia. (See Chapter 52, page 243.) 16. In which patient is oral rehydration NOT indicated? □2 yr old with moderate hypernatremic dehydration □6 mo old with mild hyponatremic dehydration ■4 mo old with severe dehydration and normal serum sodium □3 yr old with moderate dehydration and normal serum sodium □A and C description Dehydration, most often due to gastroenteritis, is a common problem in children. Most cases can be managed with oral rehydration. Even children with mild to moderate hyponatremic or hypernatremic dehydration can be managed with oral rehydration. The infant with severe dehydration is gravely ill. The decrease in blood pressure indicates that vital organs may be receiving inadequate perfusion. Immediate and aggressive intervention is necessary. If possible, the child with severe dehydration initially should receive intravenous therapy. (See Chapter 54, page 245.)
  • 9. 17. Which statement about pediatric poisoning is NOT true? □Most poisonings among young children involve a single substance and are unintentional □Poison prevention should be discussed at all well child visits beginning at 6 months □Pediatric poisonings occur most frequently in the toddler and adolescent age ranges ■The toddler age group experiences the majority of poisoning deaths □Poison control centers are available via phone, 24-7, toll free description Although the majority of exposures are in children <6 yr, only 2% of the reported deaths occur in this age group. In addition to the exploratory nature of ingestions in young children, product safety measures, poison prevention education, early recognition of exposures, and around-the-clock access to regionally based poison control centers all contribute to the favorable outcomes in this age group. Exposures in the adolescent age group are primarily intentional (suicide or abuse or misuse of substances) and thus often result in more severe toxicity. Adolescents (ages 13-19 yr) accounted for 74 of the 108 poison-related pediatric deaths in 2008 reported to the National Poison Data System. Pediatricians should be aware of the signs of drug abuse or suicidal ideation in this population and should aggressively intervene. (See Chapter 58, page 250.) 18. Quantitative levels of certain medications are helpful in the management of acute poisonings. For which of the following medications is this NOT true? □Salicylates □Acetaminophen □Iron □Carbon monoxide ■Marijuana description For select intoxications (salicylates, some anticonvulsants, acetaminophen, iron, digoxin, methanol, lithium, theophylline, ethylene glycol, carbon monoxide), quantitative blood concentrations are integral to confirming the diagnosis and formulating a treatment plan. For most exposures, quantitative measurement is not readily available and is not likely to alter management. (See Chapter 58, page 253.) 19. The 4 principles of management of poisonings are what? ■Decontamination, enhanced elimination, antidote, supportive care
  • 10. □Degradation, hydration, oxygenation, antidote □Ipecac, clinical monitoring, dialysis, reversal □Alkalinization, oxygenation, elimination, hydration description The 4 principles of management of the poisoned patient are decontamination, enhanced elimination, antidote, and supportive care. Few patients meet criteria for all of these interventions, although clinicians should consider each option in every poisoned patient so as not to miss a potentially lifesaving therapy. Antidotes are available for relatively few poisons, thus emphasizing the importance of meticulous supportive care and close clinical monitoring. (See Chapter 58, page 254.) 20. A 15 yr old boy is admitted to your care after the intentional ingestion of 2 g of his own amitriptyline in a suicide gesture. He received activated charcoal for gastrointestinal decontamination. The patient is placed in the ICU for monitoring and remains stable. He is receiving only 0.45% normal saline. You observe a change in his cardiac monitor display with a widening of his QRS complex to 0.12 second and occasional ectopic beats. The most appropriate next step in management is to: □Ignore these changes because they are still within normal limits ■Add sodium bicarbonate to his IV fluids to try to raise his serum pH above 7.4 □Repeat a dose of activated charcoal □Begin a lidocaine infusion at an appropriate dose □Order a chest radiograph description Sodium bicarbonate is the antidote of choice for TCA toxicity and works via overcoming the sodium channel blockade by providing a sodium load and via inducing an alkalosis to decrease drug binding to sodium channels. Indications for sodium bicarbonate include a QRS duration >100 msec, ventricular dysrhythmias, and hypotension. An initial bolus of 1-2 mEq/kg of sodium bicarbonate is given followed by initiation of a continuous infusion. Additional boluses may be given if the QRS duration continues to widen, with the goals of therapy being a serum pH of 7.45-7.55, improved hemodynamic stability, and narrowing of the QRS complex. (See Chapter 58, page 264.) 21. A 2 yr old boy arrives in the emergency department after a seizure. On presentation his vital signs are: temperature 40.2°C, heart rate 200 beats/min, respiratory rate 52 breaths/min. His laboratory values are: arterial pH 7.2, serum bicarbonate 6 mmol/L, arterial PCO2 18 mm Hg, sodium 148 mmol/L, potassium 3.1 mmol/L, WBC count 10,200/mm 3 . On the basis of the history obtained and the presentation, you suspect that an accidental ingestion has occurred. The most likely toxin is: □A tricyclic antidepressant
  • 11. □Acetaminophen □Cocaine □An organophosphate insecticide ■A salicylate description Salicylate ingestions are classified as acute or chronic, and acute toxicity is far more common in pediatric patients. Early signs of acute salicylism include nausea, vomiting, diaphoresis, and tinnitus. Moderate salicylate toxicity can manifest as tachypnea and hyperpnea, tachycardia, and altered mental status. The tachycardia results in large part from marked insensible losses from vomiting, tachypnea, diaphoresis, and uncoupling of oxidative phosphorylation. Thus, careful attention should be paid to volume status and early volume resuscitation in the significantly poisoned patient. Signs of severe salicylate toxicity include hyperthermia, coma, and seizures. Chronic salicylism can have a more insidious presentation, and patients can show marked toxicity at significantly lower salicylate levels than in acute toxicity. The classic blood gas of salicylate toxicity reveals a primary respiratory alkalosis and a primary, anion gap, metabolic acidosis. Hyperglycemia (early) and hypoglycemia (late) have been described. Abnormal coagulation studies, clinically manifested as bleeding and easy bruising, also may be seen. (See Chapter 58, page 260.) 22. A 2 yr old boy is noted to be drinking from a container filled with kerosene. He immediately coughs, becomes tachypneic, and is brought to the hospital. The best approach to his treatment is to: □Induce emesis □Perform nasogastric tube lavage □Instill mineral oil □Administer steroids ■None of the above description The most important manifestation of hydrocarbon toxicity is aspiration pneumonitis. Aspiration usually occurs during coughing and gagging at the time of ingestion or vomiting after the ingestion. The propensity of a hydrocarbon to cause aspiration pneumonitis is inversely proportional to its viscosity. Compounds with low viscosity, such as mineral spirits, naphtha, kerosene, gasoline, and lamp oil, spread rapidly across surfaces and cover large areas of the lungs when aspirated. Only small quantities (<1 mL) of low-viscosity hydrocarbons need be aspirated to produce significant injury. Respiratory symptoms can remain mild or progress rapidly to acute respiratory distress syndrome (ARDS) and respiratory failure. Emesis and lavage are contraindicated given the risk of aspiration. Activated charcoal is not useful because it does not bind the common hydrocarbons and can also induce vomiting. If hydrocarbon-induced pneumonitis develops, respiratory treatment is supportive. Neither corticosteroids nor prophylactic antibiotics have shown any clear benefit. (See Chapter 58, page 267.)
  • 12. 23. A 2 yr old child is found playing with a can of crystalline drain cleaner. The child's mother telephones you for help. There are several crystals in the mouth, which you have the mother wash out. The next step in treatment should be to: □Have the mother administer lemon juice or orange juice to neutralize the alkaline crystals and come to your office □Have the mother administer water or milk and call you back in 2 hr ■Have the mother administer water or milk and bring the child in for esophagoscopy □Simply observe the child because the crystals are so bitter that the child was trying to spit them out when the mother called, and therefore no problems should occur □Administer ipecac at home and bring the child in to see you description Alkalis produce a liquefaction necrosis, allowing further tissue penetration of the toxin and setting the stage for possible perforation. Ingestion of caustic materials can produce injury to the oral mucosa, esophagus, and stomach. Patients can have significant esophageal injury even in the absence of visible oral burns. Symptoms include pain, drooling, vomiting, abdominal pain, and difficulty swallowing or refusal to swallow. Laryngeal injury can manifest as stridor and respiratory distress, necessitating intubation. In the most severe cases, patients can present in shock after perforation of a hollow viscus. Initial treatment of caustic exposures includes thorough removal of the product from the skin or eye by flushing with water. Emesis and lavage are contraindicated. Activated charcoal should not be used because it does not bind these agents and can predispose the patient to vomiting and subsequent aspiration. Endoscopy should be performed within 12-24 hr of ingestion in symptomatic patients or those in whom injury is suspected on the basis of history and known characteristics of the ingested product. (See Chapter 58, page 266.) 24. A 16 yr old, 165-lb patient reports consuming 20-40 325-mg capsules containing acetaminophen 1 hr ago. The most appropriate approach to treatment is to: □Measure the plasma level and determine potential toxicity from the level on the nomogram □Wait until 4 hr after ingestion to measure the plasma level and do nothing else ■Administer activated charcoal immediately and measure the plasma level of acetaminophen 4 hr after ingestion □Send the patient home because an ingestion of this magnitude is not toxic □Administer N-acetylcysteine at a dose of 140 mg/kg description Initial treatment should focus on the ABCs and consideration of decontamination with activated charcoal in patients who present within 1-2 hr of ingestion. The antidote for acetaminophen poisoning is NAC, which works primarily via replenishing hepatic glutathione stores. NAC therapy is most effective when initiated within 8 hr of ingestion. However, there is no demonstrated benefit to giving NAC before the 4 hr postingestion mark. Thus, patients who present early after ingestion should have a 4-hr level drawn and decision to initiate NAC should be based on this level. (See Chapter 58, page
  • 13. 259.) 25. A 2 yr old child presents in the emergency department after the reported ingestion of a mouthful of lamp oil. The child reportedly vomited once at home. The child has a heart rate of 160 beats/min, a respiratory rate of 48 breaths/min, and a temperature of 37.2°C. A chest film is read as normal. The most appropriate therapy for this child is to: □Administer syrup of ipecac □Administer activated charcoal □Remove any ingested lamp oil by gastric lavage ■Admit the child for observation and supportive care □Discharge the child home with a follow-up office visit in the morning description The most important manifestation of hydrocarbon toxicity is aspiration pneumonitis. Aspiration usually occurs during coughing and gagging at the time of ingestion or vomiting after the ingestion. The propensity of a hydrocarbon to cause aspiration pneumonitis is inversely proportional to its viscosity. Compounds with low viscosity, such as mineral spirits, naphtha, kerosene, gasoline, and lamp oil, spread rapidly across surfaces and cover large areas of the lungs when aspirated. Only small quantities (<1 mL) of low-viscosity hydrocarbons need be aspirated to produce significant injury. Chest radiographs may initially be normal, but they often show abnormalities within 6 hr of exposure in patients who have aspirated. Respiratory symptoms can remain mild or progress rapidly to acute respiratory distress syndrome (ARDS) and respiratory failure. Emesis and lavage are contraindicated given the risk of aspiration. Activated charcoal is not useful because it does not bind the common hydrocarbons and can also induce vomiting. If hydrocarbon-induced pneumonitis develops, respiratory treatment is supportive. (See Chapter 58, page 267.) 26. A teenage girl presents in the emergency department with the story that she got upset with her boyfriend and swallowed a "handful of aspirin" about 4 hr previously. One hour afterward, after she began vomiting, she confessed to her mother what she had done. On examination the patient has normal vital signs and is asymptomatic except for the complaint of nausea. A serum salicylate level is ordered, but the laboratory reports no salicylates in her blood. The most appropriate next step in management is to: □Discharge the patient home ■Order an acetaminophen level □Request a psychiatric consultation □Send a second sample for salicylate determination
  • 14. □Order an abdominal radiograph to look for pills in the stomach description Acetaminophen is a widely available medication and a commonly detected co-ingestant with the potential for severe toxicity. Given that patients might initially be asymptomatic and might not report acetaminophen as a co-ingestant, an acetaminophen level should be checked in all patients who present after an intentional exposure or ingestion. (See Chapter 58, page 253.) 27. Brain perfusion pressure generally equals: ■Mean arterial pressure minus intracranial pressure □Diastolic blood pressure minus intracranial pressure □Intracranial blood pressure minus systolic blood pressure □Systolic blood pressure minus diastolic blood pressure description The perfusion pressure of the brain (cerebral perfusion pressure [CPP]) is equal to the pressure of blood entering the cranium (mean arterial pressure [MAP]) minus the ICP, in most cases. (See Chapter 63, page 296.) 28. Severe traumatic brain injury is characterized by a Glasgow Coma Score (GCS) of: □0 □1-3 ■3-8 □9-12 description The hallmark of severe TBI is coma (GCS score 3-8). Often, coma is seen immediately after the injury and is sustained. In some cases, such as with an epidural hematoma, a child may be alert at presentation but the condition may deteriorate after a period of hours. A similar picture can be seen in children with diffuse swelling, in whom a talk-and-die scenario has been described. Clinicians also should not be lulled into underappreciating the potential for deterioration of a child with moderate TBI (GCS score 9-12) with a significant contusion, because progressive swelling can potentially lead to devastating complications. In the comatose child with severe TBI, the second key clinical manifestation is the development of intracranial hypertension. Increased ICP can be appropriately managed only with continuous ICP monitoring. The development of brain swelling is progressive. Significantly raised ICP (>20 mm Hg) can occur early after severe TBI, but peak ICP generally is seen at 48-72 hr. Need for ICP-directed therapy may persist for longer than a week. A few children have coma without increased ICP, resulting from axonal injury or brainstem injury. (See Chapter 63, page 298.)
  • 15. 29. Children with moderate to severe traumatic brain injury require intracranial pressure monitoring and treatment in the critical care unit to prevent progressive swelling. First-line therapies include all of the following EXCEPT: ■Hypothermia □Head in midline position □Sedation and analgesia □Controlled mechanical ventilation □Mannitol therapy for ICP >20 mm Hg description First-tier therapy includes elevation of the head of the bed, ensuring midline positioning of the head, controlled mechanical ventilation, and sedation and analgesia (i.e., benzodiazepines and narcotics). If neuromuscular blockade is needed, it may be desirable to monitor the EEG continuously because status epilepticus can occur; this complication will not be recognized in a paralyzed patient and is associated with raised ICP and unfavorable outcome. If a ventricular rather than parenchymal catheter is used to monitor ICP, therapeutic CSF drainage is available. Other first-tier therapies include the osmolar agents mannitol and hypertonic saline given in response to ICP spikes. If ICP remains refractory to treatment, careful reassessment of the patient is needed to rule out unrecognized hypercarbia, hypoxemia, fever, hypotension, hypoglycemia, pain, and seizures. Repeat imaging should be considered to rule out a surgical lesion. Guidelines-based second-tier therapies for refractory raised ICP are available, but evidence favoring a given second-tier therapy is limited. In some centers, decompressive craniectomy is used. Others use a pentobarbital infusion. Mild hypothermia (32-34°C) to control refractory ICP can be induced and maintained by means of surface cooling. Refractory raised ICP can also be treated with hyperventilation (PaCO2 = 25-30 mm Hg). Other second-tier therapies such as lumbar CSF drainage are options. (See Figure 63-12; also Chapter 63, page 300.) 30. Which of the following statements regarding cooling treatment for perinatal hypoxic ischemic encephalopathy is not true? □Candidates for cooling therapy have perinatal asphyxia, Apgar score of 0-3, acidosis, and neurologic dysfunction □Coagulopathy is a contraindication for cooling therapy □Goal core temperature is 32-34°C during cooling therapy. □During cooling therapy, shivering should be prevented with sedation and neuromuscular blockade ■After 72 hours, the patient should be rapidly rewarmed to 36°C
  • 16. description Similarly, in perinatal asphyxia, fetal acidosis, an Apgar score of 0-3 after 5 min, neurologic dysfunction, and/or abnormal EEG findings are criteria for use of cooling therapy in term newborn infants. Exclusion criteria have included coagulopathy, bleeding, and hemodynamic instability. According to the American Heart Association (AHA) guidelines (predominantly for adults after a cardiac arrest when the initial event was associated with ventricular fibrillation), cooling should be initiated as soon as possible after return of spontaneous circulation but may be beneficial even if delayed (4-6 hr); it should be induced by means of surface cooling with cooling blankets; application of ice packs to the groin, axillae, and neck; use of wet towels; and fanning. Infusion of 20 mL/kg IV of ice-cold saline over 30 min can be considered in children and may reduce core temperature by approximately 2°C. If hypothermia is used in children, a temperature of 32-34°C should be used. The rewarming rate should be no greater than 1°C every 4-6 hr. In perinatal asphyxia, cooling should be maintained for 72 hr. Shivering should be prevented with sedation and neuromuscular blockade. Temperature should be continuously monitored. Hypothermia in children has been associated with an increased risk for neutropenia and sepsis. (See Chapter 63, page 302.) 31. First-line medications for status epilepticus include all of the following, EXCEPT: □Rectal diazepam □Intravenous lorazepam □Intravenous fosphenytoin □Intravenous phenobarbital ■Intravenous valproic acid description Several antiepileptic drugs have been advocated as first-line therapies for status epilepticus, including benzodiazepines (rectal diazepam, IV midazolam or lorazepam), phenytoin (or fosphenytoin), and barbiturates (phenobarbital). Therapy is adjusted for both symptoms and EEG evidence of seizures. In the PICU, for refractory cases, a continuous infusion of barbiturates and/or benzodiazepines may be necessary. For refractory cases that progress to this level of therapeutic intensity, respiratory support and hemodynamic monitoring and/or support should already be in place. As therapy escalates, continuous EEG monitoring should be considered to help titrate therapy. For refractory status epilepticus, newer therapies include mapping of the seizure focus followed by neurosurgical resection, IV lidocaine, or levetiracetam. (See Chapter 63, page 303.) 32. The following statements about stroke in children are true EXCEPT: □The most common causes of ischemic stroke in children are sickle cell disease and heart disease ■Atherosclerotic plaque migration is responsible for the majority of ischemic strokes □Stroke presents clinically with focal neurologic deficits or coma in children □A diagnosis of stroke requires CT or MRI findings
  • 17. □Differential diagnosis of stroke in children includes complex migraine, seizures, and encephalitis description The predominant causes of ischemic stroke in children are sickle cell disease and heart disease (either acquired or congenital), which are responsible for approximately 50% of strokes after the neonatal period. Ischemic strokes in children are generally not the result of atherosclerotic plaque migration, as they are in adults. Instead, damage to the intima of cerebral arteries can form a thrombotic nidus. In sickle cell disease, chronic turbulent blood flow likely leads to vascular damage. In intracerebral hemorrhage, blood vessel wall integrity is compromised, leading to extravasation of blood into the parenchyma or dural spaces. The usual pathology in children with heart disease is embolism from diseased valves (or intracardiac devices) and right-to-left shunts that leads to cerebrovascular occlusion. (See Chapter 63, page 303.) 33. Which of the following statements about ventilator-associated pneumonia (VAP) is NOT true? □VAP is multifactorial and causes include endotracheal tube colonization, aspiration of gastric secretions, and suppression of cough reflexes ■VAP is largely unpreventable □Fever, leukocytosis, and infiltrate on chest radiography support a diagnosis of VAP □Empirical treatment of VAP should include nosocomial pathogens description Elevation of the head of the bed to 30 degrees after initiation of mechanical ventilation and use of a protocol for oral decontamination during mechanical ventilation are two means of reducing the risk for VAP. The most effective strategy to minimize any ventilator complication is regular assessment of extubation readiness and liberation from mechanical ventilation as soon as clinically possible. (See Chapter 65, page 328.) 34. The initial ventilator settings are determined by: ■The patient's underlying disease □The patient's preferences □Standard order sets □Attempts to normalize the blood gases description The choice of mode of ventilation depends on how much ventilator-patient interaction is desired and the disease entity that is being treated. (See Chapter 65, page 327.) 35. Patients with severe forms of reactive airways disease (e.g., asthma) who require mechanical ventilation may benefit from which of the following initial ventilator parameters?
  • 18. □Rapid rates, short inspiratory times, and tidal volumes <6 mL/kg □Low rates, prolonged inspiratory/expiratory times, and low tidal volumes (<6 mL/kg) ■Low rates, prolonged inspiratory/expiratory times, and moderate tidal volumes (10-12 mL/kg) □Low rates, prolonged inspiratory/expiratory times, and high tidal volumes (>15 mL/kg) description In a patient with relatively normal lungs, an age-appropriate ventilator rate and a tidal volume of 7-10 mL/kg would be appropriate initial settings. Diseases associated with decreased time constants (decreased static compliance, e.g., ARDS, pneumonia, pulmonary edema) are best treated with small (6 mL/kg) tidal volume and relatively rapid rates (25-40 breaths/min). Diseases associated with prolonged time constants (increased airway resistance, e.g., asthma, bronchiolitis) are best treated with relatively slow rates and higher (10-12 mL/kg) tidal volume. (See Chapter 65, page 327.) 36. Per the IOM, high-quality health care by definition must be: □Timely and effective □Equitable and efficient □Safe and patient centered □A and C ■All of the above description To measure health care quality, the IOM has identified Six Dimensions of Quality, all of which relate to quality of care. The Six Dimensions of Quality are effectiveness, efficiency, equity, timeliness, patient safety, and patient-centered care. The IOM emphasizes the concept that all Six Dimensions of Quality need to be met for the provision of high-quality health care. Health care that maximizes outcomes but is not efficient (i.e., not cost-effective) is not quality care. Health care that is highly efficient but limits access also is not high quality. These concepts can be viewed as the overall value proposition—that is, the value created for a patient. (See e2-1.) 37. Which of the following is a step in the rapid cycle of improvement (PDSA)? □Perform □Define ■Study
  • 19. □Attend description The Model for Improvement can be implemented using a framework of rapid cycle of improvement also known as the Plan-Do-Study-Act (PDSA) cycle (Fig. 2-1). The PDSA cycle is typically aimed at testing small changes and then studying the results to plan and implement the next cycle of change (i.e., multiple PDSA cycles build on previous learning from PDSAs). Valuable information can be obtained from PDSA cycles that are successful and those that are not, to help plan the next iteration of the PDSA cycle. The PDSA cycle specifically requires that improvements be data driven. This is important because many clinicians attempt to make changes for improvement in their practice but do not emphasize the importance of data collection. (See e2-3.) 38. Developing a culture of safety relies on all of the following concepts EXCEPT: □De-emphasizing hierarchy and encouraging contributions of all members of the health care team □Excellent communication including patient hand-outs □Application of human factors engineering to systems and processes ■Discouraging anonymity in the reporting of adverse events description The biggest challenge in making the health system safer is changing the culture from one of treating errors as personal failures to one of treating errors as opportunities to improve the system. Organizations need to foster a culture of learning in which each individual will feel accountable for ensuring a safe and quality program, communication is open, and teamwork is valued. Reporting of errors should be valued, reports of adverse events should be handled confidentially, and those who report errors should be protected from discovery. Developing a culture of learning involves the compassionate and appropriate disclosure of system failures and medical errors to patients and families. (See e2-6). 39. A 4 yr old girl sustained a 40% 2nd- and 3rd-degree total body surface area (BSA) burn from scalding hot water. Of the following, the most therapeutic approach is: □Aggressive use of topical antibacterial agents with frequent dressing changes □Use of enzymatic debridement ointment ■Excision of the burn wounds and grafting □Use of topical analgesics description Deep 2nd-degree burns of >10% of BSA benefit from early excision and grafting. To improve outcome, sequential excision and grafting of 3rd-degree and deep 2nd-degree burns is required in children with large burns. Prompt excision with immediate wound closure is achieved with autografts, which are often meshed to increase the efficiency of coverings. (See Chapter 68, page 353.)
  • 20. 40. A 10 yr old boy sustained 30% BSA burns and had been requiring dressing changes for physical therapy. Which of the following regimens will provide the best pain management? □IV morphine bolus □Oral ibuprofen □Oral acetaminophen ■Oral morphine and oral lorazepam description From the onset of treatment, preemptive pain control during dressing changes is of paramount importance. Opiate analgesia, prescribed in an adequate dose and timed to cover dressing changes, is essential to comfort management. Anxiolytic medication added to the analgesic is usually helpful and has more than a synergistic effect. (See Chapter 68, page 355.) 41. Indications for admission to the hospital after a burn injury may include all of the following EXCEPT: □Suspected child abuse □Electric burns through an extremity □Perineal burns □Poor follow-up ■No tetanus immunization □Inhalation injury description Burns covering >10-15% of total BSA, burns associated with smoke inhalation, burns resulting from high- tension (voltage) electrical injuries, and burns associated with suspected child abuse or neglect should be treated as emergencies, and the child hospitalized. Small 1st- and 2nd-degree burns of the hands, feet, face, perineum, and joint surfaces also require admission if close follow-up care is difficult to provide. Children who have been in enclosed-space fires and those who have face and neck burns should be hospitalized for at least 24 hr for observation for signs of central nervous system (CNS) effects of anoxia from carbon monoxide poisoning and pulmonary effects from smoke inhalation. (See Table 68-2, Chapter 68, page 349.) 42. A burn wound characterized by the absence of painful sensation, bleeding, or capillary refilling is best classified as:
  • 21. □First degree □Moderate to severe □Second degree □Midlevel ■Full thickness description Full-thickness, or 3rd-degree, burns involve destruction of the entire epidermis and dermis, leaving no residual epidermal cells to repopulate the damaged area. The absence of painful sensation and capillary filling demonstrates the loss of nerve and capillary elements. The wound cannot epithelialize and can heal only by wound contraction or skin grafting. (See Table 68-5, Chapter 68, page 351.) 43. Which of the following statements regarding predictive genetic testing is true? □Predictive testing is genetic testing done in a person who is symptomatic for a genetic disorder □Predictive testing in children is ethically acceptable if the parents desire it □Federal law prevents companies from denying disability insurance based on a positive genetic test ■If a person has a positive predictive genetic test for a disease, he/she may not ever develop that disease □None of the above description Predictive genetic testing involves performing a test in a person who is at risk for developing a genetic disorder (presymptomatic), usually on the basis of family history, yet who does not manifest signs or symptoms. A major caution with predictive testing is that the presence of a gene mutation does not necessarily mean that the disease will develop. Many of the disorders with age-dependent penetrance display incomplete penetrance. A person who inherits a mutation might never develop signs of the disorder. In the USA, the Genetic Information Nondiscrimination Act of 2008 protects individuals from genetic discrimination at the hands of health insurers and employers but does not extend protection against discrimination from providers of life, disability, or long-term care insurance. (See Chapter 72, page 377.) 44. Genetic counseling is indicated in which of the following clinical scenarios? □Abnormal prenatal quad screen □Infant born with hypoplastic left heart disease
  • 22. □History of multiple miscarriages □Two cousins planning to marry □Child diagnosed with cystic fibrosis ■All of the above description See Table 72-2, Indications for Genetic Counseling. (See Chapter 72, page 378.) 45. Which statement regarding treatment of genetic disorders is NOT true? ■Physiologic therapy for genetic disease such as PKU is curative if started early □Newborn screening is important because early identification of genetic disorders allows early treatment □Enzyme replacement is available for Gaucher disease and Pompe disease □A bone marrow transplant may potentially cure thalassemia major □Gene-transfer vehicles include viruses description Physiologic therapies attempt to ameliorate the phenotype of a genetic disorder by modifying the physiology of the affected individual. The underlying defect itself is not altered by treatment. Physiologic therapies are used in the treatment of inborn errors of metabolism. These include dietary manipulation, such as avoiding phenylalanine by persons with phenylketonuria; coenzyme supplementation for some patients with methylmalonic acidemia and mitochondrial diseases; stimulation of alternative pathways to excrete ammonia for those with urea cycle disorders; bisphosphonate treatment for those with osteogenesis imperfecta to reduce bone fractures; and avoiding cigarette smoking by persons with α1-antitrypsin deficiency. Physiologic treatments can be highly effective, but they usually need to be maintained for a lifetime because they do not affect the underlying genetic disorder. Many of these treatments are most effective when begun early in life before irreversible damage has occurred. This is the rationale for comprehensive newborn screening for inborn errors of metabolism. (See Chapter 72, page 379.) 46. Which statement regarding genetic disorders of metabolism is NOT true? ■In severe disorders, the affected infant may be sick at birth □Most genetic metabolic diseases are treatable □The majority of genetic metabolic disease have autosomal recessive inheritance
  • 23. □Early diagnosis is crucial to good prognosis for most disorders □Tandem mass spectrometry may identify a large number of disorders with just a few drops of blood description In genetic disorders of metabolism, the affected infant is normal at birth and becomes symptomatic later in life. This differentiates these infants from those who appear sick at birth due to birth trauma, intrauterine insults, chromosomal abnormalities, or other genetic diseases. Severe forms of genetic disorders usually become clinically apparent in the newborn period or shortly thereafter. The majority of conditions are inherited as autosomal recessive traits. Most of the genetic metabolic conditions can be controlled successfully by some form of therapy, and a few can be potentially cured by the use of bone marrow or liver transplants. This underlines the importance of early diagnosis, which can be achieved through screening of all newborn infants. Tandem mass spectrometry (MS/MS) requires a few drops of blood to be placed on a filter paper and mailed to a central laboratory for assay. A large number of genetic conditions can be identified by this method. (See Chapter 78, page 416). 47. Signs and symptoms of inborn errors of metabolism include all of the following EXCEPT: □Vomiting ■Diarrhea □Lethargy □Poor feeding □Seizures description Pediatricians should familiarize themselves with early manifestations of genetic metabolic disorders, because (1) severe forms of some of these conditions may cause symptoms before the results of screening studies become available and (2) the current screening methods, although quite extensive, identify a small number of all inherited metabolic conditions. In the newborn period, the clinical findings are usually nonspecific and similar to those seen in infants with sepsis. A genetic disorder of metabolism should be considered in the differential diagnosis of a severely ill newborn infant, and special studies should be undertaken if the index of suspicion is high. Signs and symptoms such as lethargy, poor feeding, convulsions, and vomiting may develop as early as a few hours after birth. (See Chapter 78, page 416 and Fig. 78-1.) 48. Initial laboratory studies to investigate for metabolic disease in an ill infant should include: □Lactate, glucose, bicarbonate □Glucose, calcium, pH □Sodium, glucose, bicarbonate ■pH, bicarbonate, ammonia
  • 24. □Complete blood cell count, sodium, potassium description Measurements of serum concentrations of ammonia, bicarbonate, and pH are often very helpful initially in differentiating major causes of genetic metabolic disorders (see Fig. 78-1). An elevated blood ammonia level is usually caused by defects of urea cycle enzymes. Infants with elevated blood ammonia levels from urea cycle defects commonly have normal serum pH and bicarbonate values; without measurement of the blood ammonia level their defect may remain undiagnosed and they may succumb to their disease. Elevation of serum ammonia levels also is observed in some infants with certain organic acidemias. These infants are severely acidotic because of accumulation of organic acids in body fluids. When blood ammonia, pH, and bicarbonate values are normal, other aminoacidopathies (e.g., hyperglycinemia) or galactosemia should be considered; galactosemic infants also may manifest cataracts, hepatomegaly, ascites, and jaundice. (See Chapter 78, page 417). 49. A 2 day old boy manifests poor feeding, vomiting, and lethargy leading to coma. Laboratory data reveal respiratory alkalosis and hyperammonemia. The urine orotic acid level is also elevated. The most likely diagnosis is: □Methylmalonic acidemia □Carbamoylphosphate synthase deficiency ■Ornithine transcarbamylase (OTC) deficiency □Galactosemia □Reye syndrome description Many genetic disorders of metabolism increase plasma ammonia. In neonatal hyperammonemia, most of the symptoms are related to brain dysfunction due to the elevated ammonia. The affected infant is normal at birth but becomes symptomatic within a few days of protein feeding. Refusal to eat, vomiting, tachypnea, and lethargy can quickly progress to a deep coma. Convulsions are common. See Figure 79-13 for an algorithm to diagnose the cause of hyperammonemia. In the case of OTC deficiency, laboratory studies will not demonstrate acidosis. A marked increase in urinary orotic acid distinguishes OTC deficiency from other disorders. (See Figure 79-13 in Chapter 79, page 449.) 50. For most lysosomal storage disorders, carrier identification and prenatal diagnosis are available; a specific diagnosis is essential to permit genetic counseling. Which of the following disorders is X-linked? □Niemann-Pick disease ■Fabry disease
  • 25. □Tay-Sachs disease □Krabbe disease □Gaucher disease description For all of the lysosomal storage disorders, inheritance is autosomal recessive except for X-linked Fabry disease. (See Chapter 80, page 483.) 51. GM1 gangliosidosis is characterized by all of the following EXCEPT: □Psychomotor retardation □Angiokeratomas □Hepatosplenomegaly □Frontal bossing ■Peripheral neuropathy description The clinical manifestations of the infantile form of GM1 gangliosidosis may be evident in the newborn infant as hepatosplenomegaly, edema, and skin eruptions (angiokeratomas). It most frequently presents in the first 6 mo of life with developmental delay followed by progressive psychomotor retardation and the onset of tonic-clonic seizures. A typical facies is characterized by low-set ears, frontal bossing, a depressed nasal bridge, and an abnormally long philtrum. Up to 50% of patients have a macular cherry-red spot. Hepatosplenomegaly and skeletal abnormalities similar to those of the mucopolysaccharidoses, including anterior beaking of the vertebrae, enlargement of the sella turcica, and thickening of the calvarium, are present. (see Chapter 80, page 483.) 52. A 4 mo old girl presents with developmental delay, an exaggerated startle response to loud noise, and macrocephaly. On physical examination, the child has decreased eye contact and a cherry-red spot in each retina. The most likely diagnosis is: ■GM2 gangliosidosis □Gaucher disease □Fabry disease □Galactosemia □Glycogen storage disease, type I
  • 26. description The GM2 gangliosidoses include Tay-Sachs disease and Sandhoff disease; each results from the deficiency of β-hexosaminidase activity and the lysosomal accumulation of GM2 gangliosides, particularly in the central nervous system. The clinical manifestations of Sandhoff disease are similar to those for Tay-Sachs disease. The diagnosis of infantile Tay- Sachs disease and Sandhoff disease is usually suspected in an infant with neurologic features and a cherry-red spot. Affected infants usually develop normally until 4 to 5 mo of age when decreased eye contact and an exaggerated startle response to noise (hyperacusis) are noted. Macrocephaly, not associated with hydrocephalus, may develop. In the 2nd yr of life, seizures develop that may be refractory to anticonvulsant therapy. Neurodegeneration is relentless, with death occurring by the age of 4 or 5 yr. (See Chapter 80, page 482.) 53. A 15 yr old presents with chronic fatigue and severe bone pain of 1 year's duration. He has hepatosplenomegaly and a normal retinal examination. Laboratory studies reveal normocytic anemia and thrombocytopenia. Radiographs of the distal femur reveal Erlenmeyer flask deformities. The most likely diagnosis is: □Lymphoma ■Gaucher disease □Sandhoff disease □GM3 gangliosidosis □Sickle cell anemia description Gaucher disease is a multisystemic lipidosis characterized by hematologic abnormalities, organomegaly, and skeletal involvement, the latter usually manifesting as bone pain and pathologic fractures. Clinical manifestations of type 1 Gaucher disease have a variable age at onset, from early childhood to late adulthood, with most symptomatic patients presenting by adolescence. At presentation, patients may have bruising from thrombocytopenia, chronic fatigue secondary to anemia, hepatomegaly with or without elevated liver function test results, splenomegaly, and bone pain. Most patients develop radiologic evidence of skeletal involvement, including an Erlenmeyer flask deformity of the distal femur. (See Chapter 80, page 487.) 54. Fabry disease is characterized by all of the following EXCEPT: □Acroparesthesias □Corneal opacities □Cardiomyopathy ■Mental retardation
  • 27. □Angiokeratomas description Fabry disease is an X-linked inborn error of glycosphingolipid metabolism characterized by angiokeratomas (telangiectatic skin lesions), hypohidrosis, corneal and lenticular opacities, acroparesthesias, and vascular disease of the kidney, heart, and/or brain. (See Chapter 80, page 489.) 55. A 14 mo old girl presents with irritability, poor walking, genu recurvatum, and hypotonia. Physical examination reveals absent deep tendon reflexes. Laboratory data reveal an increased cerebrospinal fluid protein level and decreased nerve conduction velocities. The most likely diagnosis is: ■Metachromatic leukodystrophy □I cell disease □GM3 gangliosidosis □Fabry disease □Neuromyopathic Gaucher disease description The clinical manifestations of the late infantile form of metachromatic leukodystrophy, which is most common, usually present between 12 and 18 mo of age as irritability, inability to walk, and hyperextension of the knee, causing genu recurvatum. The clinical progression of the disease relates to the pathologic involvement of both central and peripheral nervous system, giving a mixture of upper and lower motor neuron and cognitive and psychiatric signs. Deep tendon reflexes are diminished or absent. Gradual muscle wasting, weakness, and hypotonia become evident and lead to a debilitated state. As the disease progresses, nystagmus, myoclonic seizures, optic atrophy, and quadriparesis appear, with death in the 1st decade of life. On evaluation, decreased nerve conduction velocities, increased cerebrospinal fluid protein, metachromatic deposits in sampled segments of sural nerve, and metachromatic granules in urinary sediment are all suggestive of the disorder. (See Chapter 80, page 490.) 56. A 15 yr old Ashkenazi Jewish girl is seen because of chronic fatigue. On examination, she seems pale and thin and has a somewhat large abdomen. Her spleen is felt in the iliac fossa. She is mentally alert and has a history of normal development and normal school performance. Her blood cell count shows hemoglobin, 9.0 g/dL and a white blood cell count of 3,000/mm 3 , with normal differential cell count and no abnormal cells. Platelet count is 60,000/μL. The most likely diagnosis is: □Tay-Sachs disease □Niemann-Pick disease type A ■Gaucher disease type I
  • 28. □Mucolipidosis type IV □Canavan disease description Clinical manifestations of type I Gaucher disease have a variable age at onset, from early childhood to late adulthood, with most symptomatic patients presenting by adolescence. Type I, which accounts for 99% of cases, has a striking predilection for Ashkenazi Jews. Gaucher disease should be considered in the differential diagnosis of patients with unexplained organomegaly, who bruise easily, have bone pain, or have a combination of these conditions. Evaluation may show bruising from thrombocytopenia, chronic fatigue secondary to anemia, hepatomegaly with or without elevated liver function test results, splenomegaly, and bone pain. (See Chapter 80, page 487.) 57. A 7 mo old boy has been healthy and developing normally since birth. His mother now reports that he has decreased eye contact with her, even during feedings. The infant also startles very easily when there is a loud noise in the house. Of the following, the most appropriate diagnostic test to confirm the etiology of these findings is the measurement of: ■Leukocyte β-hexosaminidase A activity □Serum concentration of amino acids □Serum concentration of ammonia □Urinary mucopolysaccharides □Urinary organic acids description Patients with the infantile form of GM2 gangliosidoses have clinical manifestations in infancy including loss of motor skills, increased startle reaction, and macular pallor and retinal cherry-red spots. Affected infants usually develop normally until 4 to 5 mo of age when decreased eye contact and an exaggerated startle response to noise (hyperacusis) are noted. Macrocephaly, not associated with hydrocephalus, may develop. Definitive diagnosis is made by determination of β-hexosaminidase A and B activities in peripheral leukocytes. The two GM2 gangliosidoses are distinguished by the enzymatic assay, because in Tay-Sachs disease only the β-hexosaminidase A isozyme is deficient, whereas in Sandhoff disease both the β-hexosaminidase A and B isozymes are deficient. (See Chapter 80, page 485.) 58. A 3 yr old boy was normal at birth but developed progressive coarsening of the face and developmental delay. In addition to coarse facies and mental retardation, the physical examination reveals a cardiac systolic regurgitant murmur, hepatomegaly, joint stiffness, and short stature. CT scan reveals hydrocephalus. There is no corneal clouding. All affected family members have been boys. The most likely diagnosis is: □Hurler syndrome
  • 29. □Galactosemia □Metachromatic leukodystrophy ■Hunter syndrome □Sanfilippo syndrome description Hunter disease (MPS II) is an X-linked disorder and manifests almost exclusively in males. It has been observed in a few females and this is explained by skewed inactivation of the X chromosome carrying the normal gene. Patients with severe MPS II have features similar to those of Hurler disease except for the lack of corneal clouding and the somewhat slower progression of somatic and central nervous system deterioration. Coarse facial features, short stature, dysostosis multiplex, joint stiffness, and mental retardation manifest between 2 and 4 yr of age. (See Chapter 82, page 511.) 59. Hurler disease is characterized by all of the following EXCEPT: □Hepatosplenomegaly □Coarse facies □Dysostosis multiplex ■X-linked inheritance □Corneal clouding description Diagnosis of Hurler disease is usually made between 6 and 24 mo of age with evidence of hepatosplenomegaly, coarse facial features, corneal clouding, large tongue, prominent forehead, joint stiffness, short stature, and skeletal dysplasia known as dysostosis multiplex. Acute cardiomyopathy has been found in some infants younger than 1 yr of age. Inheritance is autosomal recessive. (See Chapter 82, page 511.) 60. According to the International Classification of Diseases, major causes of U.S. infant mortality include all of the following EXCEPT: □Sudden infant death syndrome □Disorders relating to prematurity □Congenital malformations
  • 30. □Disorders relating to maternal complications of pregnancy ■Nonaccidental trauma description In the USA, –50% of infant deaths in 2008 were due to 4 conditions (classified according to the International Classification of Diseases, 10th revision): congenital malformations (20.1%), disorders relating to prematurity and unspecified LBW (16.9%), sudden infant death syndrome (SIDS) (8.2%), and newborns affected by maternal complications of pregnancy (6.3%). LBW (as a result of preterm delivery and/or IUGR) is a major determinant of both neonatal and infant mortality rates and, together with congenital anomalies (cardiac, central nervous system, respiratory), contributes significantly to childhood morbidity. In developing countries, LBW/prematurity, birth asphyxia, and infections are the major causes of infant deaths. (See e87-2.) 61. All of the following increase the risk of pregnancy EXCEPT: □Monochorionic twins □Teenage mother □Mother > 40 years of age ■Long interpregnancy interval □Cigarette smoking description See Table 89-1 for Factors Associated with High-Risk Pregnancy. The lowest neonatal mortality rate occurs in infants of mothers who receive adequate prenatal care and who are 20-30 yr of age. Pregnancies in both teenagers and women older than 40 yr, particularly primiparous women, are at increased risk for intrauterine growth restriction, fetal distress, and intrauterine death. A short interpregnancy interval is associated with increased risk. (See e89-1.) 62. Reproductive technologies are associated with all of the following pregnancy-related risks EXCEPT: □Higher risk for multiple fetuses ■Higher risk for group B streptococcal sepsis □Higher risk for low and very low birthweight □Higher risk for prematurity □Higher risk for congenital anomalies
  • 31. description The use of assisted reproductive technology (in vitro fertilization, intracytoplasmic sperm injection) increases the risk of perinatal mortality, infant morbidity, prematurity, low and very low birthweight, and cerebral palsy, largely because of the increase in multiple-fetus pregnancies with such technology; the risks for birth defects are also increased, in part, because of epigenetic effects on gene expression. (See e89-1.) 63. Oligohydramnios is associated with all of the following EXCEPT: ■Esophageal atresia □Pulmonary hypoplasia □Potter syndrome □Posterior urethral valves □Skeletal contractures description Oligohydramnios is associated with congenital anomalies; intrauterine growth restriction; severe renal, bladder, or urethral anomalies; and drugs that interfere with fetal urination. Oligohydramnios becomes most evident after 20 wk of gestation, when fetal urination is the major source of amniotic fluid. Rupture of the membranes is the most common cause of oligohydramnios and must be ruled out if oligohydramnios is suspected, especially if a normal-sized bladder is seen on fetal ultrasound evaluation. Oligohydramnios causes fetal compression abnormalities such as fetal distress, clubfoot, spadelike hands, and a flattened nasal bridge. The most serious complication of chronic oligohydramnios is pulmonary hypoplasia. Polyhydramnios is associated with premature labor, abruptio placentae, multiple congenital anomalies, and fetal neuromuscular dysfunction or obstruction of the gastrointestinal tract that interferes with reabsorption of the amniotic fluid that is normally swallowed by the fetus. Increased fetal urination or edema formation is also associated with excessive amniotic fluid volume. (See Table 89-4 and e89-2.) 64. Amnion nodosum is associated with all of the following EXCEPT: □Oligohydramnios ■Fetal Candida infection □Renal agenesis □Pulmonary hypoplasia □Flattened facies description Examination of the fresh placenta, cord, and membranes may alert the physician to a newborn infant at high
  • 32. risk and may help confirm a diagnosis in a sick infant. Amnion nodosum (granules on the amnion) and oligohydramnios are associated with pulmonary hypoplasia and renal agenesis, whereas small whitish nodules on the cord suggest a candidal infection. (See Chapter 90, page 552.) 65. Which statement about twins is not true? ■Monovular twinning appears to be an inherited tendency □Most twins are born prematurely □Ninety percent of twins are detected before delivery □Monochorionic twins may be presumed to be monovular □The second born twin is at increased risk for respiratory distress syndrome and asphyxia compared with the first born description The occurrence of monovular twins appears to be independent of genetic influence. Polyovular pregnancies are more frequent beyond the 2nd pregnancy, in older women, and in families with a history of polyovular twins. (See Chapter 91, page 553.) 66. Fetal transfusion syndrome results in which of the following? □Donor twin large and plethoric □Recipient twin small and anemic □Donor twin small and anemic □Recipient twin large and plethoric □A and B ■C and D description In the fetal transfusion syndrome, an artery from one twin acutely or chronically delivers blood that is drained into the vein of the other. The latter becomes plethoric and large, and the former is anemic and small. Generally, with chronicity, 5 g/dL hemoglobin and 20% body weight differences can be noted in this syndrome. (See Table 91-2, Chapter 91, page 554.) 67. Very-low-birthweight (VLBW) infants are best described as:
  • 33. □Predominantly growth restricted ■Predominantly premature □Predominantly post dates □The result of maternal illness □The result of placental infarction description VLBW infants weigh <1500 g and are predominantly premature. In the USA in 2008 the VLBW rates were approximately 1.46% overall, 3.01% among blacks, and 1.18% among whites. The VLBW rate is an accurate predictor of the infant mortality rate. VLBW infants account for over 50% of neonatal deaths and 50% of handicapped infants; their survival is directly related to birthweight, with approximately 20% of those between 500 and 600 g and > 90% of those between 1250 and 1500 g surviving. The VLBW rate has remained unchanged for black Americans but has increased among whites, perhaps because of a rise in multiple births among whites. Perinatal care has improved the rate of survival of VLBW infants. When compared with term infants, VLBW neonates have a higher incidence of rehospitalization during the 1st yr of life for sequelae of prematurity, infections, neurologic complications, and psychosocial disorders. (See Chapter 91, page 556.) 68. Which of the following statements regarding the assessment of gestational age at birth is NOT true? □In the absence of asphyxia, neurologic maturity correlates with gestational age ■The Ballard scoring system is accurate to within 2 days □If a discrepancy exists between the estimation of gestational age by physical examination and fetal ultrasonographic evaluation, the infant is at high risk for morbidity and mortality □Physical features used to assess gestational age include breast buds, ear development, and presence of lanugo description Neurologic maturity (nerve conduction velocity), in the absence of asphyxia, correlates with gestational age despite reduced fetal weight. Physical signs may be useful in estimating gestational age at birth. Commonly used, the Ballard scoring system is accurate to ±2 wk. An infant should be presumed to be at high risk for mortality or morbidity if a discrepancy exists between the estimation of gestational age by physical examination, the mother's estimated date of her last menstrual period, and fetal ultrasonographic evaluation. (See Figure 91-6, Chapter 91, page 557.) 69. A term female is born by spontaneous vaginal delivery to a primiparous woman who received 2 doses of meperidine 30 min and 2 hr before an abrupt delivery. The baby is apneic and limp. The most important, immediate management is to: □Administer naloxone in the umbilical vein
  • 34. ■Perform bag-mask ventilation □Administer naloxone in the endotracheal tube □Begin chest compressions □Obtain a cord pH level description Narcosis results from administration of morphine, meperidine, fentanyl, barbiturates, or tranquilizers to the mother shortly before delivery or from maternal anesthesia given during the 2nd stage of labor. This sequela should be avoided by the use of appropriate analgesic and anesthetic practices. Treatment includes initial physical stimulation and securing of a patent airway. If effective ventilation is not initiated, artificial breathing with a bag and mask must be instituted. At the same time, if the respiratory depression is due to an opiate, naloxone hydrochloride (Narcan), 0.1 mg/kg, should be given intravenously or intramuscularly. Naloxone is contraindicated in infants born to mothers with opiate addiction because it precipitates acute neonatal withdrawal with severe seizures. (See Chapter 94, page 575.) 70. Successful ventilation is determined by all of the following EXCEPT: ■Zero reading of end-tidal CO2 measurement □Pink color □Rising heart rate □Symmetric breath sounds □Good chest rise description Successful ventilation is signified by adequate chest rise, symmetric breath sounds, improved pink color, heart rate > 100 beats/min, spontaneous respirations, presence of end-tidal CO2, and improved tone. (See Chapter 94, page 576.) 71. The biopsychosocial model of development, when applied to the child's height, includes all of the following EXCEPT: □Genetic endowment □Personal eating habits □Access to food
  • 35. □Parents' beliefs ■Differences between breast milk and formula description The biopsychosocial model of medicine recognizes biologic factors, social factors, and psychologic factors that contribute to health and disease. Genetics, environment, economics (cost/access), personal habits, and family beliefs all contribute in this model. (See e6-1.) 72. All of the following statements regarding a child's temperament are true EXCEPT: ■Temperament is absolute and stable throughout the life span □Biology influences temperament □It is a pattern of the child's responses □It is relatively resistant to parents' attempts to modify □It helps parents understand the child's behavior without guilt description Temperament describes the stable, early-appearing individual variations in behavioral dimensions, including emotionality (crying, laughing, sulking), activity level, attention, sociability, and persistence. Temperament has long been described as biologic or "inherited," largely based on parent reports of twins. We now know that genes are dynamic, changing in the quantity and quality of their effects as a child ages and thus, like environment, may continue to change. Longitudinal twin studies of adult personality indicate that personality changes largely result from non-shared environmental influences, whereas stability of temperament appears to result from genetic factors. The concept of temperament can help parents understand and accept the characteristics of their children without feeling responsible for having caused them. (see e6-1). 73. The parents of a 3 yr old girl report that "she ran before she walked," "she is never hungry at the same time," and "she goes from toy to toy." This child is best described as: □Autistic □Having a specific temperament □Having attention-deficit/hyperactivity disorder ■Having developmental pervasive disorder □Being deaf
  • 36. description The classic theory of Thomas and Chess proposes 9 dimensions of temperament (Table 6-1). These characteristics lead to 3 common constellations: (1) the easy, highly adaptable child, who has regular biologic cycles; (2) the difficult child, who withdraws from new stimuli and is easily frustrated; and (3) the slow-to-warm-up child, who needs extra time to adapt to new circumstances. Various combinations of these clusters also occur. (See e6-1, including Table 6-1.) 74. A term baby girl has 2 episodes of bile-stained emesis at 24 hr after birth. There is a history of excessive amniotic fluid volume. The most appropriate diagnostic test is: □Blood culture ■Barium upper gastrointestinal x-ray series with small bowel follow-through □Barium enema □CT scan □Head ultrasound study description Bile-stained emesis suggests intestinal obstruction beyond the duodenum but also may be idiopathic. Abdominal radiographs (kidney-ureter-bladder [KUB] and cross-table lateral views) should be performed in neonates with persistent emesis and in all infants with bile-stained emesis to detect air-fluid levels, distended bowel loops, characteristic patterns of obstruction (double bubble: duodenal atresia), and pneumoperitoneum (intestinal perforation). A contrast swallow radiograph with small bowel follow-through is indicated in the presence of bilious emesis. (See Chapter 96, page 600.) 75. Meconium plug is associated with all of the following EXCEPT: □Hypermagnesemia □Infants of diabetic mothers □Cystic fibrosis □Hirschsprung disease ■Prematurity description Meconium plugs are associated with small left colon syndrome in infants of diabetic mothers and with cystic fibrosis, rectal aganglionosis, maternal opiate use, and magnesium sulfate therapy for preeclampsia. (See Chapter 96, page 600.)
  • 37. 76. Which statement about hemolytic disease of the newborn is NOT true? □RhoGAM administration should be given to the mother within 72 hours of the delivery of an Rh-negative infant; it should also be given after abortion of amniocentesis □ABO incompatibility is the most common cause of hemolytic disease of the newborn ■Most cases of ABO incompatibility require transfusion □Rh and ABO incompatibility account for approximately 95% of cases of hemolytic disease of the newborn description Most cases of ABO incompatibility are mild, with jaundice being the only clinical manifestation. The infant is not generally affected at birth; pallor is not present, and hydrops fetalis is extremely rare. The liver and spleen are not greatly enlarged, if at all. Jaundice usually appears during the 1st 24 hr. Rarely, it may become severe, and symptoms and signs of kernicterus develop rapidly. Treatment of ABO incompatibility relies on phototherapy to lower serum bilirubin levels. In severe cases, IVIG administration can reduce the rate of hemolysis and the need for exchange transfusion. Exchange transfusions with type O blood of the same Rh type as the infant may be needed in some cases to correct dangerous degrees of anemia or hyperbilirubinemia. (See Chapter 97, page 619.) 77. The best approach to prevent congenital anomalies in infants of diabetic mothers is to: □Discontinue insulin and begin glyburide □Provide continuous insulin infusion during labor and delivery □Switch from an oral hypoglycemic agent to insulin until 36-wk of gestation □Do serial glucose tolerance tests ■Maintain periconceptional tight control of maternal blood glucose levels description Periconception glucose control reduces the risk of anomalies and other adverse outcomes, and glucose control during labor reduces the incidence of neonatal hypoglycemia. Women with type 1 diabetes who have tight glucose control during pregnancy (average daily glucose levels < 95 mg/dL) are delivered of infants with birthweights and anthropomorphic features similar to those of infants of nondiabetic mothers. (See Chapter 101, page 629.) 78. Which statement regarding adolescent development is NOT true? □In girls, the first sign of puberty is breast bud development
  • 38. □Adolescents experience asymmetric growth beginning with enlargement of the hands and feet, followed by the arms and legs, and finally, the trunk and chest □Some degree of breast hypertrophy, typically bilateral, occurs in ~50% of boys during Tanner stages 2-3 ■It is abnormal for early adolescents to be preoccupied with their body changes and feel that everyone is staring at them □Adolescents are able to make more complex decisions, but these decisions are highly susceptible to emotion description Self-consciousness increases exponentially in response to the somatic transformations of puberty. Self- awareness at this age centers on external characteristics, in contrast to the introspection of later adolescence. It is normal for early adolescents to be preoccupied with their body changes, scrutinize their appearance, and feel that everyone else is staring at them. (See Chapter 104, page 651.) 79. Papanicolaou smears are indicated in: □Women aged 16 and older □Women aged 21 and older ■All women beginning 3 years after onset of sexual activity or age 21 years, whichever is earlier □All sexually active teenage girls description For sexually active females, the guidelines for Pap smears for cervical cancer screening suggest that annual screening can be delayed safely up to 3 yr after the onset of sexual activity or age 21 yr, whichever is earlier; HPV DNA testing should not be used routinely for females 29 yr and younger since the results would not influence management. (See Chapter 106, page 667.) 80. All of the following may cause gynecomastia EXCEPT: □Klinefelter syndrome □Phenothiazines □Normal puberty □Heroin
  • 39. ■Albuterol □Anabolic steroids description Pubertal gynecomastia, occurring in up to 60% of normal adolescent males, has long been attributed to a transient imbalance of estrogen and androgen concentrations. With typical onset between chronological ages of 10 and 13 yr, it usually regresses over several months Nonpubertal gynecomastia with hypogonadism is associated with Klinefelter syndrome and places a patient at a higher risk of breast cancer. Other conditions associated with nonpubertal gynecomastia are secondary to endocrine disorders, neoplasms, chronic disease, trauma, and medications as well as drugs of abuse. See Table 109-1, Drugs causing Gynecomastia. (See e109-1.) 81. Common presenting complaints of pregnancy include all of the following EXCEPT: □Fatigue □Breast tenderness □Nausea □Irregular menses ■Dysuria description Adolescents may experience the traditional symptoms of pregnancy: morning sickness (vomiting, nausea that may also occur any time of the day), swollen tender breasts, weight gain, and amenorrhea. Often the presentation is less classic. Headache, fatigue, abdominal pain, dizziness, and scanty or irregular menses are common presenting complaints. (See Chapter 112, page 700.) 82. Teenage pregnancy carries increased risk of all of the following EXCEPT: □Prematurity □Low birth weight infant □Gastroschisis ■Multiple birth (twins) □Maternal eclampsia
  • 40. description As expected, teen mothers have low rates of age-related chronic disease (diabetes or hypertension) that might affect the outcomes of a pregnancy. They also have lower rates of twin pregnancies than older women. They tolerate childbirth well with few operative interventions. However, as compared with 20-39 yr old mothers, teens have higher incidences of low birthweight infants, preterm infants, neonatal deaths, passage of moderate to heavy fetal meconium during parturition, and infant deaths within 1 yr after birth. The highest rates of these poor outcomes occur in the youngest and most economically deprived mothers. Gastroschisis, although very rare, has a markedly higher incidence in infants of teen mothers for reasons that are not yet clear. Teen mothers also have higher rates of anemia, pregnancy-associated hypertension, and eclampsia, with the youngest teens having rates of pregnancy-associated hypertension 40% higher than the rates of women in their 20s and 30s. The youngest teens also have a higher incidence of poor weight gain (<16 lb) during their pregnancy. This correlates with a decrease in the birthweights of their infants. (See Chapter 112, page 701.) 83. The X-linked lymphoproliferative (XLP) syndrome is classically associated with overwhelming infection by which of the following agents? ■Epstein-Barr virus □Enteroviruses □Catalase-positive bacteria □Neisseria meningitidis □Pneumocystis jiroveci description X-linked lymphoproliferative (XLP) disease is an X-linked recessive trait characterized by an inadequate immune response to infection with Epstein-Barr virus (EBV). Affected males are usually healthy until they acquire EBV infection. The mean age of presentation is < 5 yr. There are 3 major clinical phenotypes: (1) fulminant, often fatal, infectious mononucleosis (50% of cases); (2) lymphomas, predominantly involving B-lineage cells (25%); or (3) acquired hypogammaglobulinemia (25%). There is a marked impairment in production of antibodies to the EBV nuclear antigen (EBNA), whereas titers of antibodies to the viral capsid antigen (VCA) have ranged from absent to markedly elevated. XLP has an unfavorable prognosis; 70% of affected boys die by age 10 yr. (See Chapter 118, page 727.) 84. A 1 yr old child with recurrent sinusitis is found to have a normal WBC count but no circulating B cells, small tonsils, and no palpable lymph nodes. Serum concentrations of IgG, IgA, IgM, and IgE are below the 95% lower limits for age. The most likely diagnosis is: □Ataxia-telangiectasia □X-linked lymphoproliferative syndrome □DiGeorge syndrome □Common variable immunodeficiency
  • 41. ■X-linked agammaglobulinemia (XLA) description Most boys afflicted with XLA remain well during the 1st 6-9 mo of life by virtue of maternally transmitted IgG antibodies. Thereafter, they acquire infections with extracellular pyogenic organisms, such as Streptococcus pneumoniae and Haemophilus influenzae, unless they are given prophylactic antibiotics or immunoglobulin therapy. Infections include sinusitis, otitis media, pneumonia, or, less often, sepsis or meningitis. Infections with Mycoplasma also are particularly problematic. The diagnosis of XLA should be suspected if lymphoid hypoplasia is found on physical examination (minimal or no tonsillar tissue and no palpable lymph nodes), and serum concentrations of IgG, IgA, IgM, and IgE are far below the 95% confidence limits for appropriate age- and race-matched controls, usually with total immunoglobulins < 100 mg/dL. (See Chapter 118, page 124.) 85. All of the following are prominent features of Wiskott-Aldrich syndrome EXCEPT: □Atopic dermatitis □Thrombocytopenia □Recurrent infections with encapsulated bacteria ■Autosomal dominant inheritance □More frequent occurrence in males description Wiskott-Aldrich syndrome, an X-linked recessive syndrome, is characterized by atopic dermatitis, thrombocytopenic purpura with normal-appearing megakaryocytes but small defective platelets, and undue susceptibility to infection. Patients often have prolonged bleeding from the circumcision site or bloody diarrhea during infancy. The thrombocytopenia is not initially due to antiplatelet antibodies. Atopic dermatitis and recurrent infections usually develop during the 1st yr of life. Streptococcus pneumoniae and other bacteria having polysaccharide capsules cause otitis media, pneumonia, meningitis, and sepsis. Later, infections with agents such as P. jiroveci and the herpesviruses become more frequent. Survival beyond the teens is rare; infections, bleeding, and EBV-associated malignancies are major causes of death. (See Chapter 120, page 734.) 86. All of the following are typically associated with an eosinophilic response and eosinophilia EXCEPT: □Allergic rhinitis □Hypersensitivity drug reactions □Trichinosis (Trichinella spiralis)
  • 42. ■Pinworms (Enterobius vermicularis) □Wiskott-Aldrich syndrome description Many diseases are associated with moderate (1,500-5,000 cells/μL) or severe (>5,000 cells/μL) eosinophilia (see Table 123-1). Allergy is the most common cause of eosinophilia in children in the USA. Eosinophilia is often associated with infection with multicellular helminthic parasites, which are the most common cause in developing countries. Severe eosinophilia in children is most commonly due to visceral larva migrans. The level of eosinophilia tends to parallel the magnitude and extent of tissue invasion, especially by larvae. Eosinophilia often does not occur in established parasitic infections that are well contained within tissues or are solely intraluminal in the gastrointestinal tract, such as Giardia lamblia and Enterobius vermicularis infection. Eosinophilia is observed in many patients with primary immunodeficiency syndromes, especially hyper-IgE syndrome, Wiskott-Aldrich syndrome, and Omenn syndrome. (See Chapter 123, page 740.) 87. A 2 yr old boy refuses to hold his mother's hand when walking across the parking lot of a shopping mall. He attempts to run away from her, but she quickly grabs his hand before he darts out in front of a car. She is exasperated and scared by his actions. Of the following, which technique is most effective in addressing the behavior? □Verbal reprimand □Spanking □Scolding ■Time-out □Slapping his hand description Discipline should be immediate, specific to the behavior, and time limited. Time-out for approximately 1 min/yr of age is very effective. A kitchen timer allows the parent to step back from the situation; the child is free when the timer rings. (See Chapter 10, page 36.) 88. To be effective, time-outs should: □Be accompanied by verbal reprimands □Be accompanied by punishment □Last at least 5-10 min ■Last 1 min/yr of age
  • 43. □Be done at a point in time removed from the incident description Discipline should be immediate, specific to the behavior, and time limited. Time-out for approximately 1 min/yr of age is very effective. A kitchen timer allows the parent to step back from the situation; the child is free when the timer rings. (See Chapter 10 page 36.) 89. Handedness is usually determined by age: □2-4 mo □6-12 mo □15-18 mo □20-24 mo ■36-48 mo description Handedness is usually established by the 3rd yr. Frustration may result from attempts to change children's hand preference. Variations in fine motor development reflect both individual proclivities and different opportunities for learning. Children who are seldom allowed to use crayons, for example, develop a mature pencil grasp later. (See Chapter 10, page 33.) 90. The best approach for parents to help a preschool child overcome monster fears is to: □Rationalize that monsters do not exist □Read books that do not have monsters in them □Have the pediatrician explain that monsters are make-believe ■Use "great power" like monster spray to keep monsters away □None of the above description The active imagination that fuels play and the magical, animist thinking characteristic of preoperational cognition can also generate intense fears. More than 80% of parents report at least 1 fear in their preschool children. Refusal to take baths or to sit on the toilet may arise from the fear of being washed or flushed away, reflecting a child's immature appreciation of relative size. Attempts to demonstrate rationally that there are no monsters in the closet often fail, inasmuch as the fear arises from prerational thinking. However, this same thinking allows parents to be endowed with magical powers that can banish the monsters with "monster spray" or a night light. Parents should acknowledge the fears, offer reassurance and a sense of security, and give the child some sense of control over the situation. (See Chapter 10, page 35.)
  • 44. 91. Between 2 and 5 yr of age, language increases; as a rule, the number of words in a sentence is: □Based on knowledge of numbers ■Equal to the age of the child in years □Independent of the environment □Independent of the number of questions asked the child by adults □Based on the ABCs description Language development occurs most rapidly between 2 and 5 yr of age. Vocabulary increases from 50-100 words to more than 2,000. Sentence structure advances from telegraphic phrases ("Baby cry") to sentences incorporating all of the major grammatical components. As a rule of thumb, between the ages of 2 and 5 yr, the number of words in a typical sentence equals the child's age (2 by age 2 yr, 3 by age 3 yr, and so on). By 21 mo to 2 yr, most children are using possessives ("My ball"), progressives (the "-ing" construction, as in "I playing"), questions, and negatives. (See Chapter 10, page 35.) 92. All of the following statements regarding language development are true EXCEPT: □Deaf children may create their own language □The basics for language may be "hard-wired" in the brain ■Environment and experience have minimal impact on the rate of language development □Delayed language may signify deafness □Delayed language may signify mental retardation description It is important to distinguish between speech (the production of intelligible sounds) and language, which refers to the underlying mental act. Language includes both expressive and receptive functions. Receptive language (understanding) varies less in its rate of acquisition than does expressive language; therefore, it has greater prognostic importance. (See Chapters 14 and 32.) Language acquisition depends critically on environmental input. Key determinants include the amount and variety of speech directed toward children and the frequency with which adults ask questions and encourage verbalization. Children raised in poverty typically perform lower on measures of language development compared with children from economically advantaged families. Although experience influences the rate of language development, many linguists believe that the basic mechanism for language learning is "hard-wired" in the brain. Children do not simply imitate adult speech; they abstract the complex rules of grammar from the ambient language, generating implicit hypotheses. Evidence for the existence of such implicit rules comes from analysis of grammatical errors, such as the overgeneralized use of "-s" to signify the plural and "-ed" to signify
  • 45. the past ("We seed lots of mouses."). Language is linked to both cognitive and emotional development. Language delays may be the first indication that a child has mental retardation, has an autism spectrum disorder, or has been maltreated. Language plays a critical part in the regulation of behavior through internalized "private speech" in which a child repeats adult prohibitions, first audibly and then mentally. Language also allows children to express feelings, such as anger or frustration, without acting them out; consequently, language-delayed children show higher rates of tantrums and other externalizing behaviors. (See Chapter 10, page 35.) 93. Repeated meningococcal infections suggest which of the following types of immune disorder? □B-cell defect □T-cell defect □Combined B- and T-cell defect □Phagocyte function defect ■Complement component deficiency description A defect of complement function should be considered in any patient with recurrent pyogenic infections, disseminated meningococcal or gonococcal infection, or a second episode of bacteremia at any age. A previously well adolescent or young adult with meningococcal meningitis due to an uncommon serotype (not A, B, or C) should undergo screening for a late-component or alternative pathway deficiency with CH50 and AP50 assays. (See Chapter 128, page 753.) 94. A complement defect should be suspected in which disorder? □Recurrent angioedema □Chronic nephritis □Recurrent hemolytic-uremic syndrome □A and B ■All of the above description A defect of complement function should be considered in any patient with recurrent angioedema, autoimmune disease, chronic nephritis, hemolytic-uremic syndrome (HUS), or partial lipodystrophy, or with recurrent pyogenic infections, disseminated meningococcal or gonococcal infection, or a second episode of bacteremia at any age. A previously well adolescent or young adult with meningococcal meningitis due to an uncommon serotype (not A, B, or C)
  • 46. should undergo screening for a late-component or alternative pathway deficiency with CH50 and AP50 assays. (See Chapter 128, page 753.) 95. Signs of allergy on physical examination include all of the following EXCEPT: □Dennie lines (Dennie-Morgan folds) □Purple discolorations beneath the lower eyelids ■Erythematous nasal mucosa with thick yellow secretions □Xerosis □Keratoconus description Parents of allergic children are often concerned about blue-gray to purple discolorations beneath the lower eyelids, attributed to venous stasis and referred to as allergic shiners. They are found in up to 60% of allergic patients and almost 40% of patients without allergic disease. They are often accompanied by Dennie lines (Dennie-Morgan folds), which are prominent symmetric skin folds that extend in an arc from the inner canthus beneath and parallel to the lower lid margin. The nasal mucosa in allergic rhinitis is classically described as pale to purple in comparison with the beefy red mucosa of patients with nonallergic rhinitis. Allergic nasal secretions are typically thin and clear. Purulent secretions suggest another cause of rhinitis. Keratoconus, or protrusion of the cornea, may occur in patients with atopic dermatitis as a result of repeated trauma produced by persistent rubbing of the eyes. (See Chapter 135, page 765.) 96. A 7 yr old boy with asthma has roughness over the extensor surfaces of the upper arms and thighs, which is caused by keratin plugs lodged in the openings of hair follicles. This physical finding is termed: ■Keratosis pilaris □Fibroepitheliosis □Hidradenitis □Xerosis □Acrochordon description Keratosis pilaris, often found on the extensor surfaces of the upper arms and thighs, is characterized by roughness of the skin caused by keratin plugs lodged in the openings of hair follicles. Examination of the skin of the palms and feet reveals exaggerated palmar and plantar creases in some allergic children. (See Chapter 135, page 765.)